Wikipedia:Auskunft/alt37

Wikipedia:Auskunft/alt37/Intro

Fehler bei Vorlage (Vorlage:Autoarchiv-Erledigt): Bei "Zeigen=Nein" können die Parameter Übersicht, aktuelles Archiv und Icon nicht angegeben werden.

18. September 2015

Warum ist Existenz nie notwendig?

Bekanntlich können manche Dinge prinzipiell nicht existieren, nämlich wenn deren Begriff widersprüchlich ist. Zum Beispiel verheiratete Junggesellen. Ihre Nicht-Existenz ist notwendig. Bzw. anders formuliert: Ihre Existenz ist unmöglich. Umgekehrt lassen sich aber anscheinend keine Dinge finden deren Existenz notwendig ist. Existenz scheint immer nur entweder kontingent (weder unmöglich noch notwendig, z.B. Pferd oder Einhorn) oder unmöglich zu sein. Aber wieso? --Cubefox (Diskussion) 21:41, 18. Sep. 2015 (CEST)[Beantworten]

Drittsemesterfrage. Die Existenz der Existenz ist notwendig. --46.253.188.165 21:44, 18. Sep. 2015 (CEST)[Beantworten]
Hm, sicher? Existenz "existiert" ja nicht, zumindest nicht in dem Sinne in dem ein Junggeselle oder ein Naturgesetz in der realen Welt existiert oder nicht. Ich hätte Existenz zu den rein abstrakten Begriffen gezählt. Ähnlich wie z.b. Relationen, Zahlen etc. --Cubefox (Diskussion) 22:12, 18. Sep. 2015 (CEST) PS: Drittsemester von was? --Cubefox (Diskussion) 22:12, 18. Sep. 2015 (CEST)[Beantworten]
Wenn du alles auf Begriffe bringst, existiert nichts notwendig, was nichts anderes sagt, als dass Begriffe nicht notwendig existieren. Will man der Aussage mehr Sinn abgewinnen, muss man anderes als Begriffe betrachten (oder entfalten, was der Unterschied zwischen „rein abstrakten“ und anderen Begriffen ist, was wohl auf ähnliches hinausläuft – aber ich bin offen für andere Ansätze), s. u. --Chricho ¹ ³ 22:15, 18. Sep. 2015 (CEST)[Beantworten]
Aber was könnte man statt Begriffen verwenden? Ich kann mir keine Alternative vorstellen. Mir scheint dass allein schon "denken" nichts anderes bedeutet als "mit Begriffen operieren". Zu den abstrakten Begriffen haben wir ja unten schon geschrieben. (Wobei es auch eine Minderheitenposition gibt, nach der Zahlen, Naturrechte usw. gar nicht abstrakt sind, sondern tatsächlich in einer Art platonischen, unsichtbaren Ideenwelt existieren, ähnlich wie ein Stuhl oder ein Tisch.) --Cubefox (Diskussion) 00:07, 19. Sep. 2015 (CEST)[Beantworten]
Mit "Begriffen" meint ihr beide wohl "das Gemeinte, wie wir es erkannt zu haben meinen und mehr oder weniger verbindlich in Sprache umgesetzt bzw diese Begriffe definiert haben" (Zit. Chricho: "sie existieren nicht notwendig."; Zit.: Cubefox: "Mit Begriffen operieren bedeutet denken.")? Im Gegensatz dazu stünde die tatsächliche Wirklichkeit des Gemeinten "in der realen Welt" (Cubefox) ? Also Genau die epistemologische Hauptfrage ("Was ist ``Sein´´?") ? - Und da hakt die EET ein: Wir erkennen die Welt so gut, daß wir in ihr Bestand haben konnten, aber nicht völlig deckungsgleich, teils auch verzerrt \ gefiltert bis falsch (zB Unwichtiges ganz weggefiltert), weil zweckgebunden. - Das semiotische Dreieck ist nur ein Element des weitaus umfassenderen Kreisels:   | Welt wie sie (an sich) ist |   --> sie wahrnehmen (per Sinne) --&; sie erkennen (per Gehirn), benennen mit Begriffen und für wirklich halten --> in diesen Begriffen denken und wieder Neues wahrnehmen --> usw. .. Sprache, Denken, unsere Wirklichkeit bedingen sich gegenseitig und wachsen an unserer Umwelt, die zweifellos an sich wirklich ist, unabhängig unserer Wahrnehmung ihrer. Das Pikante an diesem Kreisel, daß Veränderung stattfindet, alles abhängig voneinander evoluiert und wir selber mit unserem Denken und Körper und Gehirn, die Denken tun, individuell, wie auch als homo sapiens, wie auch wissenshistorisch über Generationen alles sich mitändert aneinander, kommt in der bloßen Vermittlung zwischen Ding in Welt an sich über Zeichen, Wörter, Grammatik zu den Begriffen, dem Gemeinten des semiotischen Dreiecks (wohl auch nicht kreisläufig, sondern in bestimmten Richtungen) nicht vor. Vom Ding über das Wort zum Gemeinten und zurück ist epistemologisch, erkenntnistheoretisch zuwenig, um zu verstehen, wie unser Bild der Welt entsteht.
Noch zu "Pegasus": Er existiert nicht deshalb nicht, weil es keine geflügelten Pferde gibt, nicht deshalb (gar) nicht, weil er Fiktion ist, es ihn in der `wirklichen´ Welt nicht gibt, weil er die Realität irgendwie transzendieren würde als reines Gedankenobjekt, als `über- oder neben- oder außerwirkliche´ (rein? oder `nur´??) `ausgedachte´ Vorstellung, sondern er existiert als gespeicherte Reize für "Pferd" + "geflügelt" + der ganzen Mythologie um ihn als abrufbares Bild im Gehirn, in Energiezuständen und neuronalen Verknüpfungen .. ganz physisch!! Man darf mE ein abstraktes "Existieren" (von was auch immer), den Begriff "Existenz" (ohne konkreten Zusammenhang, allgemein), nicht auf irgendwelche `Dinge´ (gibt's eh nicht, es sind Wellen, Felder, Zeuch, letztlich) reduzieren. "Existenz an sich" umfaßt Alles, auch solche Ideen, die wirklich werden können, wenn der erste Mensch sich einen "Topf" ausdenkt und dann Palm-Mark-Brei in einer halben Kokusnuss oder in einem Bambusabschnitt auf dem Feuer gart. Da ist der Topf - zwar menschgemacht aber immer noch: - "Natur", wie auch eine Rakete. Wer wöllte entsceiden ab wann die Kokusnuss plötzlich(?) ein Topf ist, doch schon ab dem Moment, wo jemand die bloße Idee hat, daß es funktionieren könnte (vielleicht schon vorher, weil die Zeit reif ist, Töpfe zu erfinden oder es möglich ist, daß er erfunden wird). Was wir uns `ausdenken´, ist ebenso Teil der Welt, wie ein bestimmter Energiezustand, den ein bestimmtes Elektron gerade nicht hat. Es existiert sogar im Gehirn der Plan physisch, während ein Elektron von sich und seinem `anderen´ Energiezustand nichts wissen muß. In der Gentechnik hat die Vorstellung von Chimären Gestalt angenommen (was mit etwas, das es nicht \ unmöglich geben kann, nicht möglich gewesen wäre). Unser Denken transzendiert eben nicht - wie von vielen ~Ismen willkürlich und beliebig postuliert - eine `dinglich-physische´ Realität. -217.84.93.123 13:59, 30. Sep. 2015 (CEST)[Beantworten]
Ich versuche die Frage mal mit einem phänomenologischen Ansatz auf bessere Begriffe zu bringen: Die Haltung, die du in der Frage eingenommen hast, dass du Dinge auf einen „Begriff“ bringst und von dort aus betrachtest, ähnelt wohl der eidetischen Reduktion (aber diese Haltung ist eine vorlogische, geht also nicht von Begriffen, sondern von einem Wesen), eidos aus, nach Husserl wird dabei die Existenz „eingeklammert“, du nimmst eine neutrale, „uninteressierte“ Haltung dem gegenüber ein. Das heißt aber nicht, dass du so die Existenz negieren kannst, sie bleibt erhalten (vgl. die IP-Antwort, wenn man so aber argumentieren will, muss die vorlogische Perspektive eingenommen werden, die Existenz des Begriffs der Existenz ist nicht notwendig). Was das genau heißt, zum Beispiel in Bezug auf die Existenz der Welt, ist eine wichtige Frage der nach-husserlschen Philosophie. --Chricho ¹ ³ 22:09, 18. Sep. 2015 (CEST)[Beantworten]
Okay, jetzt wirds kompliziert. Ich kann dir nicht ganz folgen, mit Phänomenologie und Husserl kenne ich mich wenig aus. Der Begriff der Existenz selbst existiert wohl irgendwie (z.B. als Vorstellung von bewusstseinsfähigen Wesen), aber das gilt ja auch für beliebige andere Begriffe, z.B. Einhorn oder Weihnachtsmann. Dass ein Begriff existiert, heißt also nicht, dass er auf etwas referenziert, das in der Welt existiert, also im physischen oder zumindest physikalischen Sinne. --Cubefox (Diskussion) 22:29, 18. Sep. 2015 (CEST)[Beantworten]
Okay, jetzt führst du Referenz ins Feld und versuchst so den Unterschied zwischen „rein abstrakten“ und anderen Begriffen zu fassen. Der Referent ist notwendig transzendent gegenüber dem Begriff (davon versucht sich die klassische Phänomenologie gerade mit dem Begriff der Intentionalität oder des Noemas zu verabschieden). Wenn dus mit Referenz magst: Neben den abgesonderten „rein abstrakten“ Begriffen müssen wir dann wohl noch eine Unterscheidung einführen: Eigennamen auf der einen Seite, allgemeine, aber nicht abstrakte Begriffe auf der anderen Seite. Bei letzteren musst du alle möglichen Bezugsobjekte in Betracht ziehen, bzw. (was eine Klärung dessen hinausläuft) in allen möglichen Sprechhandlungen. Wenn dus aporetisch magst mit Derrida: Die Möglichkeit der Nicht-Existenz des Referenten ist gerade Bedingung der Möglichkeit der Referenz, die Referenz setzt die unabhängige Existenz des Begriffs/Bezeichners/Eigennamens voraus. Das ist eine Aporie der Referenz – wenn du deine Frage nicht von der Referenz ausgehend fragst, findest du sicher anderes (wenn du vergleichbares bei Husserl finden willst, musst du dir anschauen, wie er Unsinn und Widersinn behandelt). Eine andere beliebte Ansicht ist, die Existenz des Referenten als bedeutungslos abzutun, ihn in einer bestimmten Kant-Interpretation als „als ob“ anzusehen. --Chricho ¹ ³ 22:47, 18. Sep. 2015 (CEST)[Beantworten]
Also noch mal langsam zum mitschreiben. ^^ Mit Referent meinst du das, worauf sich ein Begriff (oder ein Wort? egal?) bezieht, oder? Der Referent existiert entweder (Pferde) oder nicht (Einhörner, verheiratete Junggesellen).
Aber was heißt "Der Referent ist notwendig transzendent gegenüber dem Begriff"?
Zu den "allgemeinen" Begriffen: Meinst du damit etwas wie "Mensch", was sich ja nicht auf einen konkreten Menschen (z.B. "Sokrates") bezieht? "Der Mensch" an sich würde dann tatsächlich nicht existieren (weil das wieder eine Abstraktion wäre), wohl aber "die Menschen", weil sich das auf die Menge aller konkreten, physischen menschlichen Individuen bezieht (Sokrates, Chricho, ...).
Zu deinem als Behauptung formulierten Satz (Die Möglichkeit der Nicht-Existenz des Referenten ist gerade Bedingung der Möglichkeit der Referenz, die Referenz setzt die unabhängige Existenz des Begriffs/Bezeichners/Eigennamens voraus.) kann ich irgendwie keine Begründung in deinem Beitrag finden... Habe ich etwas übersehen?
Und wie könnte man meine "Frage nicht von der Referenz ausgehend" fragen? Meinst du man könnte meine Ausgangsfrage auf eine bestimmte Weise umformulieren, ohne die zentrale Bedeutung zu verändern, und dann ergäbe sich die Antwort gleich von selbst?
Wie man die Existenz der Referenten für bedeutungslos halten kann verstehe ich nicht. Es ist besteht doch offensichtlich ein großer Unterschied zwischen "Angela Merkel existiert" und "Harry Potter existiert". Sonst würde man im Endeffekt sogar Wissenschaften und Pseudowissenschaften auf eine Stufe stellen. (Gibt es einen anthropogenen Klimawandel oder nicht? Bedeutungslos!) Wobei man natürlich annehmen kann, Angela Merkel existiere ebenso wenig wie Harry Potter weil ja alles nur Einbildung sei etc. Aber selbst wenn es auf der Welt nur mich geben und sonst nichts existieren würde, weil ich mir alles einbilde, stellt sich die Frage warum ich mir partout nichts einbilde das notwendig existiert... --Cubefox (Diskussion) 23:51, 18. Sep. 2015 (CEST)[Beantworten]
Siehe semiotisches Dreieck. Die Antwort ist ja oder nein, je nachdem, was du mit „bezieht“ meinst.
Zur besagten Aporie: Setz mal für „Möglichkeit der Nicht-Existenz“ „Möglichkeit der Abwesendheit“ ein. Referenz wäre bedeutungslos, überflüssig, unsinnig, wenn sie nur in Anwesenheit des Referenten möglich wäre. Aber was heißt Abwesendheit anderes, als dass die Gewissheit der Existenz nicht mehr gegeben ist, Möglichkeit der Nicht-Existenz? Das Gorgias-Zitat unten geht in eine ähnliche Richtung, es ist freilich weniger klar und wmgl. eine Parodie.
Wenn wer sagt „bedeutungslos“ kann das verschiedenes heißen. Es kann heißen, etwas nicht als letztes Fundament anerkennen zu wollen. Es kann heißen, dass dazu aufgefordert wird, Fragen wie „Gibt es einen anthropogenen Klimawandel?“ zumindest prinzipiell (wenn auch nicht pragmatisch) als falsch gestellt zurückzuweisen, was nicht heißt, dass die Frage keinen Hintergrund haben kann, vor dem sie dann besser gestellt werden kann, wohl aber, dass die Form der Frage schon zu Irritationen führen kann. Und wer sagt denn, dass die Bestimmung von Wissenschaftlichkeit auf Fragen der Existenz von Referenten fußen soll? Da gibt es doch unzählige andere Ansätze (Wissenschaftstheorie).
Für ein notwendigerweise Eingebildetes wäre das cogito wohl der klassische Kandidat. --Chricho ¹ ³ 00:12, 19. Sep. 2015 (CEST)[Beantworten]
Puh, die Diskussion wird philosophisch immer fortschrittlicher und ich immer müder. Ich les mir das dann morgen in Ruhe durch. Danke schon mal für deine Beiträge! --Cubefox (Diskussion) 01:27, 19. Sep. 2015 (CEST)[Beantworten]
Semiotisches Dreieck: Ja, die Ecke die da unter anderem "Ding" genannt wird ("Gegenstände, Sachverhalte, Ereignisse [...] 'was Sache ist'") entspricht genau meiner vorherigen Interpretation von "Referent". Ich wüsste nicht wie man "bezieht" auch so verstehen könnte, dass dem nicht so wäre.
Kann sein dass ich wieder auf dem Schlauch stehe, aber ich sehe keinen wesentlichen Unterschied wenn ich "Möglichkeit der Abwesendheit" einsetze. "Abwesendheit" ist natürlich nicht deckungsgleich mit Nicht-Existenz, weil auch "nicht-in-der-Nähe-sein" mit abwesend gemeint sein kann. Aber das macht den zitierten Satz für mich nicht plausibler. Inwiefern eine Referenz auf etwas notwendig anwesendes "bedeutungslos, überflüssig, unsinnig" wäre erschließt sich mir deshalb auch nicht. Ich denke übrigens nicht dass nur Abwesendheit heißt, dass die Gewisstheit der Existenz nicht mehr gegeben ist, denn auch wenn etwas anwesend (in der nähe und sinnlich erfahrbar) ist, könnte es rein theoretisch trotzdem eine Einbildung sein. Die Existenz von realen Dingen in der Welt ist deswegen nie völlig sicher, ausgenommen die eigene Existenz ("cogito ergo sum"). Aber auch die ist kontingent, nicht notwendig, weil es mich zwar zweifelsllos gibt, es mich aber genauso gut nicht geben könnte.
Inwiefern die erwähnten Frage nach dem Klimawandel "falsch gestellt" sein könnte weiß ich nicht. Man kann wie gesagt natürlich nie völlig sicher sein, dass nicht die ganze Welt inkl. Klimawandel Einbildung ist. Das heißt aber nicht, dass es irgend einen Grund gibt, die Antwort der Pseudowissenschaften für genauso wahrscheinlich zu halten wie die der Wissenschaften. Die Wissenschaftstheorie führt meines Erachtens genug Kriterien auf, die bestimmte Aussagen als sehr viel sicherer und glaubwürdiger auszeichnet als andere, ohne dabei vollkommene Sicherheit zu fordern. Wobei das IMHO schon von Thema wegführt. --Cubefox (Diskussion) 12:14, 19. Sep. 2015 (CEST)[Beantworten]
„Bezieht“ könnte man auch für das Verhältnis von Signifikant zu Signifikat benutzen.
So wie du von Anwesenheit sprichst, gehst du immer schon von einer Beziehung auf ein „reales Ding in der Welt“ aus. Was das ist, ist hier ja aber erst zu klären. Was ist für dich denn zum Beispiel eine Evidenz (was für die Wissenschaften etwa in der Praxis wichtiger zu sein scheint als die Existenz eines Dings)? Husserl bestimmt das als eine Selbstgegebenheit. Was soll da Eingebildetheit bedeuten? Du gehst ja hingegen bei der Wahl zwischen „Existenz“ und Eingebildetheit immer schon von einer Referenz auf eine transzendente, reale Welt an sich aus (Einbildung von etwas heißt: es ist nicht in dieser realen Welt an sich), die Referenz auf eine solche Welt an sich scheint dein a priori zu sein. Indem du das Schema der Referenz allem zugrunde legst, kannst du schlecht klären, was Referenz ist, und auf keine Notwendigkeit schließen (denn Referenz gelingt eben notwendigerweise nicht notwendig, was nur außerhalb deiner Grundlagen irgendwie zu klären ist, aber auch deiner Intuition entspricht, wenn du die Frage so stellst). Um auf eine Aporie bei dir aufmerksam zu machen: Zuerst bestimmst du als eingebildet, was nicht auf etwas in der Welt verweist. Dann aber sprichst du auch von der Möglichkeit, dass die Welt selbst Einbildung ist. Was soll das dann bedeuten?
Eine Evidenz, die sich zugleich in ihrer eigenen Notwendigkeit zeigt, nennt Husserl übrigens apodiktisch. Und da wäre ein Beispiel das cogito. Nicht im Sinne eines Schlusses, vom „ego cogito“ zum „ego sum“, in dem das „ego“ eine eigene geistige Sphäre, eine Person oder dergleichen bezeichnet, sondern die denkende Reflexivität des cogito selbst zeigt sich als notwendig. Solche Notwendigkeit ist aber wohl etwas anderes, als was du dir unter Notwendigkeit der Existenz im Sinne einer Referenz auf ein reales Ding in der transzendenten Welt vorstellst, sondern transzendentale Notwendigkeit (was aber auch mehr ist als eine logische Notwendigkeit).
Du kannst natürlich alle philosophischen Ansätze kritisieren, aber bedenke zumindest den Wert, den diese Ansätze haben, um deine Gedanken etwas zu ordnen und ihr Voraussetzungen und Alternativen zu klären. Nicht von einer ursprünglichen und zugleich fragwürdigen festen Instanz einer Referenz auf eine Welt an sich ausgehen zu müssen, ist meines Erachtens durchaus etwas wert. Insb. wenn wir von Wissenschaften reden: Gerade eine solche Instanz liefert überhaupt nichts, um das wissenschaftliche Vorgehen und wmgl. Wahrheit dabei klären zu können.
Zu der Klimawandel-Frage: Was zum Beispiel an der Frage schlecht sein könnte, ist, dass sie verhüllt, dass es sich um ein ethisch-pragmatisches Problem einer Verantwortung für die Zukunft, für andere handelt und nicht um eines der Setzung „anthropogener“ und „nicht-anthropogener“ Wesenheiten, dass sie ebenso das wissenschaftliche Vorgehen verhüllt. Die Rhetorik der Klima-Skeptiker beruht nicht selten genug auf Vokabular „nicht-bewiesener Existenz“ etc., das eben den Anschluss an das, was Wissenschaft praktisch ausmacht, verschließt. Grüße --Chricho ¹ ³ 13:58, 19. Sep. 2015 (CEST)[Beantworten]
Was ist für dich denn zum Beispiel eine Evidenz (was für die Wissenschaften etwa in der Praxis wichtiger zu sein scheint als die Existenz eines Dings)? - Den Ausdruck Evidenz benutze ich normalerweise nicht, insofern kann ich darüber nur sagen was unter Evidenz steht. Es scheint da um Aussagen zu gehen, die keiner Begründung bedürfen, etwa wenn sie die Introspektion betreffen. So etwas gibt es aber in der Wissenschaft eher selten, zumindest in den Nturwissenschaften. Dass die Erde um die Sonne kreist ist zum Beispiel alles andere als evident, darauf musste man erst über Umwege (mittelbar) schließen. Die einzige wissenschaftliche Methode, die versucht sich nur auf evidente Tatsachen zu verlassen, scheint die Phänomenologie zu sein. Und deren mögliches Anwendungsgebiet ist natürlich auf bestimmte Gebiete begrenzt.
Husserl bestimmt das als eine Selbstgegebenheit. Was soll da Eingebildetheit bedeuten? - Nichts, ich habe "eingebildet" auch gar nicht auf evidente Tatsachen bezogen. Wenn eine Tatsache evident ist, dann braucht man keine (möglicherweise falsche) Begründung dafür, und dann gibt es auch keine Möglichkeit sich zu irren, und demnach auch keine Einbildung, die ja immer einen Irrtum darstellt. Wenn zum Beispiel die Empfindung von Schmerz evident ist, dann kann man sich Schmerz nicht "einbilden", er ist dann immer echt. Die Antwort auf die Frage ob etwas evident ist oder nicht, ist allerdings selbst nicht unbedingt evident. Manche Leute halten die Existenz Gottes für evident, aber das bezweifelten selbst viele gläubige Menschen und suchten nach Gottesbeweisen (die überflüssig wären, wenn die Existenz Gottes evident wäre.).
Du gehst ja hingegen bei der Wahl zwischen „Existenz“ und Eingebildetheit immer schon von einer Referenz auf eine transzendente, reale Welt an sich aus (Einbildung von etwas heißt: es ist nicht in dieser realen Welt an sich), die Referenz auf eine solche Welt an sich scheint dein a priori zu sein. - Ich denke das ist einfach eine Definitionsfrage, bzw. eine Frage der Bedeutung des Wortes "existieren". Was ein Wort bedeutet, bestimmt die Mehrheit der Sprecher, und die scheint damit meines Erachtens in der Mehrheit eine Referenz auf reale Objekte zu meinen, "außerhalb" und unabhängig vom eigenen Geist. Wenn man kommuniziert, muss man sich an den üblichen Bedeutungen von Wörtern orientieren oder selbst Definitionen mitliefern, ansonsten wird man mit Sicherheit missverstanden, und es geht einem wie diesem Herrn. Wörter möglichst gemäß ihrer üblichen Verwendung zu benutzen ist eben nicht a priori, sondern a posteriori, es beruht auf dem Erlernen von Sprache. Natürlich könnte es sein dass man sich irrt und die meisten Leute etwas ganz anderes mit diversen Wörtern meinen. Es könnte sein dass die Leute, wenn sie "Tisch" sagen, das meinen, was ich meine, wenn ich "Teppich" sage. Das halte ich aber für eher unwahrscheinlich, denn sollche Irrtümer fallen relativ schnell auf wenn jemand Wörter unerwartet verwendet.
Indem du das Schema der Referenz allem zugrunde legst - Ich lege Referenz ja nicht allem, sondern nur Begriffen zugrunde. Und nicht, weil es mir Spaß macht, sondern weil das anscheinend dem üblichen Begriff von "Begriff" entspricht (was meines Erachtens auch der Artikel zum semiotischen Dreieck bestätigt). Man kann es vermutlich anders machen (keine Ahnung wie, evtl. müsste ich mal Husserl lesen), wobei dann zumindest die Gefahr besteht, einfach nur andere Begriffe für die gleichen Wörter zu verwenden. Es könnte dann sein dass man mit den gleichen Worten von etwas anderem spricht, und ein anderes Problem behandelt ohne es zu merken.
denn Referenz gelingt eben notwendigerweise nicht notwendig, was nur außerhalb deiner Grundlagen irgendwie zu klären ist - Dass Referenz "notwendigerweise nicht notwendig" sei ist aber erst mal nur eine Behauptung, für die ich eine Begründung bräuchte. Zumindest kann ich aktuell keine erkennen. Beides zusammen wäre dann die Antwort auf die Frage meines Ausgangsbeitrags.
Zuerst bestimmst du als eingebildet, was nicht auf etwas in der Welt verweist. Dann aber sprichst du auch von der Möglichkeit, dass die Welt selbst Einbildung ist. Was soll das dann bedeuten? - Letzteres war schlampig formuliert. Ich meine nicht, dass die Welt eingebildet sein könnte, sondern dass es sein könnte, dass das, was wir für die Welt (für existierend) halten, nicht die Welt ist. Siehe z.B. Solipsismus oder Gehirn im Tank.
sondern die denkende Reflexivität des cogito selbst zeigt sich als notwendig - Ich weiß nicht wie Husserl das begründet, aber ich würde wie die Evolutionäre Erkenntnistheorie argumentieren, dass die Existenz von denkenden Wesen wahrscheinlich eine notwendige Bedingung von transzendentalen Urteilen ist.
Gerade eine solche Instanz liefert überhaupt nichts, um das wissenschaftliche Vorgehen und wmgl. Wahrheit dabei klären zu können. - Wie würdest du dann Sätze wie "Angela Merkel existiert" von Sätzen wie "Harry Potter existiert" abgrenzen? Wenn man sie nicht abgrenzen kann, wenn man alle Existenzsätze für gleichermaßen falsch hält, könnte man kaum noch angemessen auf die Anforderungen des Alltags reagieren. Man müsste alles mögliche bezweifeln und könnte tatsächlichen Unsinn nicht mehr erkennen. Ähnlich wie beim Verfolgungswahn, wo man hinter jeder Harmlosigkeit eine Bedrohung vermutetet, und so auch nicht mehr in der Lage ist, echte Bedrohungen zu erkennen.
Was zum Beispiel an der Frage schlecht sein könnte, ist, dass sie verhüllt, dass es sich um ein ethisch-pragmatisches Problem einer Verantwortung für die Zukunft - Verstehe ich nicht. Wenn es tatsächlich keinen von Menschen verursachten Klimawandel gäbe (z.B. weil die Klimaforscher Teil einer Verschwörung sind), dann gäbe es auch tatsächlich kein ethisch-pragmatisches Problem. Klimawandelleugner bezweifeln ja nicht, dass ein anthropogener Klimawandel gewissen Umfangs ein Problem wäre auf das man reagieren müsste, sondern dass es diesen Klimawandel gibt.
Die Rhetorik der Klima-Skeptiker beruht nicht selten genug auf Vokabular „nicht-bewiesener Existenz“ etc., das eben den Anschluss an das, was Wissenschaft praktisch ausmacht - Stimmt, sichere Existenzbeweise gibt es in den empirischen Wissenschaften nicht, was aber nicht heißt dass alles Wissen gleichermaßen unsicher wäre. --Cubefox (Diskussion) 19:01, 19. Sep. 2015 (CEST)[Beantworten]
Dass der Begriff der Existenz durch eine Mehrheit der Sprecher (Sprache funktioniert übrigens durchaus nicht durch Abstimmung und Überstimmung), durch den alltäglichen Sprachgebrauch hinreichend geklärt wäre – glaubst du das wirklich?
Evidenz hast du, wenn du dich für das Konzept stark machen willst, in den Naturwissenschaften in den Erfahrungen, auf denen sie aufbauen, in den Beobachtungen und Experimenten. Dass Evidenz nicht unmmitelbar ist, stets schon Vermittlung drin steckt, macht das Konzept noch nicht zwangsläufig hinfällig.
Bzgl. Welt: Jetzt hast du auf einmal eine Dreiteilung, die noch über Referenz hinausgeht: Den Begriff der Welt, das, was wir für die Welt halten, und die Welt. Was heißt das denn nun? Hast du dir nicht schon die Antwort gegeben, dass in deinem Weltbild die Existenz der Welt notwendig ist? Mit evolutionärer Erkenntnistheorie setzt du wohl einen wissenschaftlichen Realismus voraus, das klappt ohne Welt sicher nicht. Wie du denn mit der Möglichkeit des Solipsismus vereinbaren willst, weiß ich aber nicht.
Angela Merkel und Harry Potter weiß ich dadurch zu unterscheiden, dass diese Vorstellungen, die ich habe, ganz unterschiedlich gebildet sind, im einen Fall durch bestimmte politische Konstellationen, im anderen Fall durch eine fest bestimmte Zahl fiktiver (und Fiktion ist etwas anderes als irgendeine Nichtexistenz) Texte. Nur mal eine Liste von Personen: Sokrates, Jack the Ripper, Homer, Subcomandante Marcos, Menelik I. … und jetzt überleg mal, ob „Existenz“ das sinnvollste Kriterium ist, um ihre Bedeutung zu verstehen?
Die Klima-Skeptiker spielen mit den Verwirrungen, die aus der Vermenung von alltagsmetaphysischen Vorstellungen von Existenz und wissenschaftlichen Ausdrucksweisen entstehen. Und doch, die Klima-Skeptiker sagen mitunter auch, dass ein Klimawandel gar kein Problem wäre (und argumentieren etwa damit, dass das ja auch nicht anthropogen vorkommt). --Chricho ¹ ³ 21:47, 19. Sep. 2015 (CEST)[Beantworten]
Dass der Begriff der Existenz durch eine Mehrheit der Sprecher (Sprache funktioniert übrigens durchaus nicht durch Abstimmung und Überstimmung), durch den alltäglichen Sprachgebrauch hinreichend geklärt wäre – glaubst du das wirklich? - Ich weiß nicht was du mit "hinreichend geklärt" genau meinst, jedenfalls bin ich mir wie gesagt ziemlich sicher, dass damit meist ein Verweis auf etwas subjektunabhängiges gemeint ist. Glaubst etwa du dass dem nicht so ist? Das würde wohl wieder etwas vom Thema wegführen, aber ich könnte einige Beispiele anführen die dem widersprechen. (Eine repräsentative Umfrage dazu hat vermutlich noch niemand gemacht.)
Den Begriff der Welt, das, was wir für die Welt halten, und die Welt. Was heißt das denn nun? - Das heißt, dass Irrtümer möglich sind. Hast du dich etwa noch nie geirrt? Man kann einen Begriff von etwas haben, und man kann annehmen, dass dem etwas Reales (Ding-Ecke) entspricht, z.B. ein Pferd -- oder nicht, z.B. Pegasus. Dabei kann man sich natürlich irren. Der Glaube an feuerspeiende Drachen war zum Beispiel mal verbreitet, er hat heute aber nur noch wenige Anhänger.
Hast du dir nicht schon die Antwort gegeben, dass in deinem Weltbild die Existenz der Welt notwendig ist? - Ich habe nicht gesagt dass die Existenz der Welt notwendig ist, im Gegenteil. Sogar der Titel dieses Abschnitts widerspricht dem.
Mit evolutionärer Erkenntnistheorie setzt du wohl einen wissenschaftlichen Realismus voraus, das klappt ohne Welt sicher nicht. - Aber die Welt klappt umgekehrt ohne "wissenschaftlichen Realismus". Der ist nämlich nur eine Theorie den sich irgendwelche Leute ausgedacht haben. Wenn sie richtig ist, dann ist sie auch ohne diese Leute richtig. Wenn sich die Planeten auf Keplerschen Bahnen bewegen, liegt das auch nicht an Kepler. Oder hängst du der Theorie an, dass der Mond nur da ist, wenn einer hinschaut?
Wie du denn mit der Möglichkeit des Solipsismus vereinbaren willst, weiß ich aber nicht. - Solipsismus und wissenschaftlicher Realismus (wovon die Evolutionäre Erkenntnistheorie ausgeht) schließen sich tatsächlich gegenseitig aus. Aber ich sage ja nicht, dass beides stimmt, sondern dass eines von beidem stimmen könnte. Darüber hinaus halte ich die Evolutionäre Erkenntnistheorie für sehr viel wahrscheinlicher richtig als den Solipsismus, weil erstere vieles von dem, wie ich die Welt wahrnehme, erklären kann, der Solipsismus jedoch nicht. (Bei Interesse kann ich das ausführen.)
Angela Merkel und Harry Potter weiß ich dadurch zu unterscheiden, dass diese Vorstellungen, die ich habe, ganz unterschiedlich gebildet sind, im einen Fall durch bestimmte politische Konstellationen, im anderen Fall durch eine fest bestimmte Zahl fiktiver (und Fiktion ist etwas anderes als irgendeine Nichtexistenz) Texte. - Und woran macht man fest dass die Texte fiktiv sind? Würde es Harry Potter wirklich geben, würde man die Bücher nicht unter Fiction (wie die Angelsachsen sagen) oder Fantasy einsortieren, sondern bei den Biographien. (Es könnte natürlich theoretisch auch sein, dass es Harry Potter incl. Hogwarts tatsächlich gibt, ohne dass es jemand außer J.K. Rowling weiß. Dann wären die Bücher unter Fiction natürlich falsch einsortiert.)
Sokrates, Jack the Ripper, Homer, Subcomandante Marcos, Menelik I. … und jetzt überleg mal, ob „Existenz“ das sinnvollste Kriterium ist, um ihre Bedeutung zu verstehen? - Es stimmt, bei historischen Personen ist das oft wirklich nicht so wichtig. Es könnte auch sein, dass Platon sich Sokrates ausgedacht hat. Bei alltäglichen Situationen ist es schon ganz anders. Es macht z.B. unter Umständen einen erheblichen Unterschied, ob es die heruntergefahrene Bahnschranke oder den herannahenden Zug gibt oder nicht. Wer bloß nicht an Türschwellen glaubt, dürfte noch mit einigen gebrochenen Zehen davon kommen.
Die Klima-Skeptiker spielen mit den Verwirrungen, die aus der Vermenung von alltagsmetaphysischen Vorstellungen von Existenz und wissenschaftlichen Ausdrucksweisen entstehen. - Ich denke sie gehen einfach nur von einer Verschwörung fast aller Klimaforscher aus, ohne sich der Unwahrscheinlichkeit dieser Theorie bewusst zu sein.
Und doch, die Klima-Skeptiker sagen mitunter auch, dass ein Klimawandel gar kein Problem wäre (und argumentieren etwa damit, dass das ja auch nicht anthropogen vorkommt). - Dann sagen sie aber, dass die Auswrkungen gering sind. Deswegen schrieb ich ja "gewissen Ausmaßes". --Cubefox (Diskussion) 17:05, 20. Sep. 2015 (CEST)[Beantworten]
Siehe Existenz: In der Existenzphilosophie ist Existenz ganz und gar nicht subjektunabhängig (auch wenn es Versuche gibt, sie davon zu lösen), ebenso wenn du von deiner Existenzgrundlage sprichst, und in der formalen Logik hat es auch nichts mit „realen Dingen“ zu tun. Ich seh ja, wie du es benutzt und versuche die Ganze Zeit mich an deinen Gebrauch des Wortes anzupassen, bloß solltest du dich dafür nicht auf irgendeine Mehrheit berufen.
Der wissenschaftliche Realismus ist keine Theorie (wie etwa ein Modell für die Planetenbewegung), sondern eine philosophische Position, in der die Existenz der Welt durchaus notwendig ist, die du anscheinend immer wieder annimmst (widersprechende allerdings auch, wie die Möglichkeit des Solipsismus). Insb. ist deine Haltung gegenüber philosophischen Positionen ja nun äußerst voraussetzungsvoll – du brauchst irgendeine Instanz, in der sich die Gültigkeit einer Position bestimmt, und irgendwie lassen sich über diese noch Wahrscheinlichkeitsaussagen machen? Mit einem statistischen Test? Was ist die Nullhypothese? Und wie soll eine solche Metaposition dann überhaupt vereinbar sein mit der Gültigkeit etwa des wissenschaftlichen Realismus oder des Solipsismus?
Der Artikel Fiktion hier ist ganz gut. Fiktion definiert sich eben nicht über das übereinstimmen mit Realität, sondern ist eine sehr spezifische Kulturtechnik. Wenn ich nun eine Person treffe, die mir anvertraut, dass Rowling die Harry-Potter-Romane in Bezug auf ihn geschrieben hat, der Name „Harry Potter“ auf diese Person referiert, mit ein paar Abweichungen (vllt. bei den Zauberfertigkeiten) und ein paar Übereinstimmungen (vllt. beim Aussehen), oder wenn Daniel Radcliffe sich in Harry Potter umbenennt, entsprechend im Alltag herumläuft und Geschichten von seiner Schulzeit in Hogwarts erzählt, dann ändert das in beiden Fällen nichts daran, dass die Romane Fiktion sind (und auch am Status Harry Potters ändert es nichts, vorausgesetzt, ich kann die anonyme Person bzw. Daniel Radcliffe auch gut ignorieren).
Wieso mich deine Ausführungen über Irrtümer irritiert haben: Es ging um die Angelegenheit, dass der Begriff der Welt auf eine Welt an sich referieren soll. Entweder gelingt die Referenz oder sie gelingt nicht. Du kannst dich jetzt entscheiden, ob es Irrtümer gibt, bei denen aber die Referenz im Grunde bestehen bleibt, oder nicht. Grüße --Chricho ¹ ³ 17:53, 20. Sep. 2015 (CEST)[Beantworten]
Siehe Existenz: In der Existenzphilosophie ist Existenz ganz und gar nicht subjektunabhängig (auch wenn es Versuche gibt, sie davon zu lösen), ebenso wenn du von deiner Existenzgrundlage sprichst, und in der formalen Logik hat es auch nichts mit „realen Dingen“ zu tun. Ich seh ja, wie du es benutzt und versuche die Ganze Zeit mich an deinen Gebrauch des Wortes anzupassen, bloß solltest du dich dafür nicht auf irgendeine Mehrheit berufen. - Dass es auch anders verwendet wird bestreite ich ja nicht, für weitaus üblicher halte ich aber weiterhin die von mir verwendete Bedeutung. Aber wie gesagt, sicher könnte man das nur mit einer Umfrage klären. (Anscheinend würdest du annehmen, dass dort "Pegasus existiert" genauso häufig als wahre Aussage angekreuzt wird wie "Angela Merkel existiert". Auch wenn ich mir immer noch nicht recht vorstellen kann dass du das wirklich annimmst. Aber es ist ja wie du schon sagst egal, solange du verstehst welche Bedeutung ich in der Ausgangsfrage meinte.)
Der wissenschaftliche Realismus ist keine Theorie (wie etwa ein Modell für die Planetenbewegung), sondern eine philosophische Position, in der die Existenz der Welt durchaus notwendig ist - Nenn es Theorie oder "Position", jedenfalls kann sie genauso falsch oder richtig sein wie eine Theorie. Eine zentrale Annahme des wissenschaftliche Realismus ist, dass es eine subjektunabhängige Realität gibt. Annahmen sind kontingent, nicht aber notwendig (und widersprüchliche Annahmen wären unmöglich, d.h. notwendig falsch).
die du anscheinend immer wieder annimmst - Nein, ich nehme nicht an dass irgend etwas notwendig existiert. Keine Ahnung wie oft ich das noch wiederholen soll. :( Die Frage ist ja gerade, warum es offenbar nichts notwendig gibt.
(widersprechende allerdings auch, wie die Möglichkeit des Solipsismus) - Nicht widersprüchlich, ich hatte in meinem letzten Beitrag dazu noch etwas eingefügt, vielleicht hast du das noch nicht gelesen.
du brauchst irgendeine Instanz, in der sich die Gültigkeit einer Position bestimmt, und irgendwie lassen sich über diese noch Wahrscheinlichkeitsaussagen machen? Mit einem statistischen Test? Was ist die Nullhypothese? - Es gibt Prinzipien, nach denen manche Aussagen a priori plausibler ("wahrscheinlicher richtig") sind als andere. Am bekanntesten ist wohl das Sparsamkeitsprinzip, besser bekannt als Ockhams Rasiermesser. Die Gültigkeit dieses Prinzips ist aber natürlich auch nur eine Annahme, die man bezweifeln kann, auch wenn das kaum jemand tut (du vielleicht?). Ob und wie man dieses Prinzip begründen könnte, ist übrigens eine interessante Fragestellung. Einer kurzen Google-Suche nach zu urteilen gibt es anscheinend schon ein paar Ansätze. Vielleicht wäre Husserls Phänomenologie eine geeignete Methode? Ich erinnere mich dunkel dass jemand versucht hat mit letzterer die Gültigkeit der klassischen Aussagenlogik (oder der Deduktion überhaupt?) zu begründen.
Der Artikel Fiktion hier ist ganz gut. Fiktion definiert sich eben nicht über das übereinstimmen mit Realität - Vielleicht nicht nur, aber auch. Schon in der Einleitung steht "Zur Erklärung von Fiktion werden in der Literatur- und Kunsttheorie unter anderem fehlender Wahrheitsanspruch und mangelnde Übereinstimmung mit der Realität herangezogen." Der Duden sieht es ähnlich.
Zu deinen Ausführungen zu Harry Potter: So wie ich es sehe bestätigt der Artikel Fiktion meine Annahme, dass die Bücher durchaus nicht als Fiktion bezeichnet würden, wenn Rowling darin reale Verhältnisse beschreiben würden (und wenn das allgemein bekannt wäre natürlich). Natürlich gibt es auch mit erfundenen Elementen vermischte Biographien, wo die beschriebenen Dinge nur grob der Wirklichkeit entsprechen. Wenn sich Daniel Radcliffe in Harry Potter umbenennen würde, würde das natürlich nichts daran ändern, dass er ein Schauspieler ist und nicht der Zauberer, den Rowling meinte. Ich denke der Unterschied zwischen Fiktion und Wahn/Halluzination/Täuschung ist, dass man sich bei der Fiktion jederzeit darüber im Klaren ist, dass sie nicht mit der Wirklichkeit übereinstimmt.
Ich weiß nicht genau was du mit "gelingen" oder "bestehen" einer Referenz meinst. Ich würde lieber bei der Formulierung meines vorherigen Beitrags bleiben, dann braucht man auch den meines Erachtens eher unklaren Begriff der Referenz nicht: "Man kann einen Begriff von etwas haben, und man kann annehmen, dass dem etwas Reales (Ding-Ecke) entspricht, z.B. ein Pferd -- oder nicht, z.B. Pegasus." Wenn eine Annahme falsch ist, nennt man das auch einen Irrtum. Was ist daran unklar? --Cubefox (Diskussion) 22:51, 20. Sep. 2015 (CEST)[Beantworten]
Das Wort Existenz: Mit Ungeklärtheit meinte ich insb., dass die verschiedenen aufgeführten Bedeutungsvariationen eben nicht klar getrennt sind. Dass du für dich das recht klar trennst, mag sein, wenn du dich auf allgemeinen Gebrauch beruhst, kannst du diese Trennung da aber nicht unbedingt voraussetzen.
Fiktion: „Keine der gängigen Fiktionstheorien sieht Erfundenheit als notwendigen Bestandteil fiktionaler Darstellungen an. Es wird sogar die Auffassung vertreten, dass es fiktionale Darstellungen geben kann oder gibt, die völlig ohne Erfundenheit auskommen.“
„Nein, ich nehme nicht an dass irgend etwas notwendig existiert. Keine Ahnung wie oft ich das noch wiederholen soll.“ Deine Frage kenne ich, ich habe bloß versucht, dich davon zu überzeugen, dass du eben eine notwendige Existenz von etwas implizit annimmst. Es tut mir Leid, dass ich dafür Dinge rausgreifen und verschieben muss, aber einfach als ein fertiges Etwas lässt sich deine Ursprungsfrage eben nicht nehmen, man muss schon reingehen, wie sie gestellt ist. Also bitte nicht sauer werden, wenn ich dir etwas unterjuble, was dir zugleich widerspricht, liegt das nicht daran, dass ich nicht gelesen hätte. Unter einfachem Hinnehmen könnte ich gar nichts zur Angelegenheit sagen. Was natürlich nicht ausschließt, dass ich manches falsch verstanden habe, in dem Fall darfst du natürlich zurecht darauf hinweisen.
Solipsismus als Widerspruch: Es ging mir nicht mehr um den Widerspruch zwischen Solipsismus und wissenschaftlichem Realismus, sondern zwischen einem möglichen Widerspruch zwischen deiner Metaposition (von der aus du Positionen als möglicherweise falsch, nicht notwendig, plausibel etc. einordnest) und Solipsismus bzw. wissenschaftlichem Realismus. Von deiner gemütlichen Metaposition aus sind dir alle Dinge und alle Positionen nicht notwendig, dass diese Metaposition von den einzelnen Positionen, die du reflektierst (etwa Solipsismus), allerdings erschüttert werden könnte, reflektierst du nicht, als würde sich alle Philosophie nur auf eine Spielzeugwelt beziehen und nicht immer schon die Aussagen, die du über sie machst, selbst mit umschließen. Kann es sein, dass das Fortkommen der Diskussion hier dadurch blockiert ist, dass du jede philosophische Position, die ich anführe, insb. solche, die ich in deinen Argumentationsstrukturen auszumachen meine, deiner Metaposition zur Beurteilung und als bloße Möglichkeit unterstellst, die dabei unangetastet bleibt, obwohl es mir egtl. auch genau um die geht? Natürlich gibt es die Möglichkeit, eine solche Metaposition einzunehmen, immer. Ich versuche nur, bei dir eine Verunsicherung auszulösen, von der ich glaube, dass sie innerhalb deiner Äußerungen schon zwischen manchen Stellen besteht.
„jedenfalls kann sie genauso falsch oder richtig sein wie eine Theorie“: Was hast du denn da für einen Wahrheitsbegriff? --Chricho ¹ ³ 00:18, 21. Sep. 2015 (CEST)[Beantworten]
Danke schon mal für deinen Beitrag. Meine Antwort kommt noch, ich bin noch etwas angeschlagen von einem VolksBierfest. --Cubefox (Diskussion) 20:27, 22. Sep. 2015 (CEST)[Beantworten]
Zur Metaposition: Wenn man über irgendetwas redet, das irgendjemand anderes gesagt hat, das kann auch irgend etwas Banales, völlig unphilosophisches sein, dann hat man relativ dazu schon eine "Metaposition" eingenommen. Wenn das problematisch wäre, wäre fast die ganze Auskunftsseite hier problematisch. Du nimmst natürlich auch eine "Metaposition" ein, wenn du über meine Aussagen sprichst. Ich kann aber nicht erkennen, was daran problematisch wäre. Das einzige "Problem" das ich sehe, ist, dass man sich auch bei den Überlegungen über andere Überlegungen irren kann, aber das ist wohl weniger ein Problem als eine triviale Tatsache. Die einzige Alternative dazu wäre, sich nie gedanklich mit widersprechenden Positionen zu befassen, nie ihre Möglichkeit und den eigenen Irrtum in Betracht zu ziehen. Das nennt man Dogmatismus. Ich wüsste nicht welche Vorteile Dogmatismus haben könnte. (Ich wüsste aber sehr viele Nachteile.) Wenn ich religiöser Dogmatiker wäre, und felsenfest glauben würde, dass Gott notwendig existiert, was würde das schon über die Richtigkeit meiner Überzeugung sagen? Warum sollte Glaubenstärke irgendetwas mit der Richtigkeit des Geglaubten zu tun haben? Umgekehrt hat die Vermeidung von Dogmatismus meines Erachtens mehrere Vorteile. Unter Anderem: Wenn ich eine Möglichkeit in Betracht ziehe, dann kann ich mich in etwas hineinversetzen. Ich kann ein Gedankenexperiment machen, um zu sehen, welche Implikationen es hätte, wenn ich etwas bestimmtes annehmen würde. Ich kann mir etwa vorstellen ein dogmatischer Solipsist zu sein. Würde daraus folgen, dass für mich dann die Richtigkeit des Solipsismus notwendig wäre? Ich denke nicht. Ich würde nur annehmen, er sei notwendig richtig, aber das ist ja etwas anderes. Man könnte natürlich prüfen, ob die Überzeugung richtig ist, indem man sie begründet. Im Falle von Notwendigkeit ist die einzige Möglichkeit ein Beweis. (Willkürliches Beispiel: P1: Alle Junggesellen sind männlich und unverheiratet. P2: Hans ist Junggeselle. C: Also ist Hans männlich und unverheiratet.) Ein solcher Beweis kann zwar unplausible Prämissen haben, aber der Beweis selbst ist davon unabhängig. Das heißt jeder kann ihn nachvollziehen, egal was er konkret annimmt, also z.B. auch unabhängig davon, ob man selbst Solipsist ist oder nicht.
„jedenfalls kann sie genauso falsch oder richtig sein wie eine Theorie“: Was hast du denn da für einen Wahrheitsbegriff? - Einen Intuitiven. Allerdings mir ist bekannt, dass es verschiedene Wahrheitsbegriffe gibt (deren Definitionen man bescheidenerweise Wahrheitstheorien nennt, weil sie meist irgendwelche "Macken" haben, d.h. vom intuitiven Wahrheitsbegriff abweichen.) Ich kenne aber keinen Wahrheitsbegriff, der einen prinzipiellen Unterschied zwischen wahren "Positionen" und wahren Theorien machen würde. Der einzige (graduelle, nicht grundsätzliche) Unterschied den ich hier sehen kann: "Position" verwendet man eher für einzelne Hypothesen, "Theorie" dagegen meist für mehrere Hypothesen, die zusammen einen größeren Zusammenhang beschreiben. Dabei kann nur ein Teil der Hypothesen der Theorie falsch sein, und die Theorie kann die Wirklichkeit trotzdem halbwegs gut beschreiben. Zum Beispiel beschreibt die Einsteinsche Gravitationstheorie die Wirklichkeit anscheinend besser als die Newtonsche, aber weder liegt Newtons Theorie komplett daneben noch ist Einsteins Theorie perfekt. Theorien sind also wohl weniger dichotom als Hypothesen. Wobei man natürlich auch Hypothesen beliebig schwammig formulieren kann, sodass sie desto weniger dichotom werden. --Cubefox (Diskussion) 22:40, 23. Sep. 2015 (CEST)[Beantworten]

Alle Antworten weder hier abschließend beantwortet. ;-) --Gonzo.Lubitsch (Diskussion) 22:44, 18. Sep. 2015 (CEST)[Beantworten]

Äh ... okay. ^^ --Cubefox (Diskussion) 22:49, 18. Sep. 2015 (CEST)[Beantworten]
Naja, wurde als Abschluss eher nicht hingenommen. Heidegger spricht in der Tat vom „Vorrang der ‚existentia‘ vor der essentia“ (SZ 58) in Bezug aufs Dasein, Sartre hat daraus gemacht, dass „die Existenz dem Wesen vorausgeht“. --Chricho ¹ ³ 22:54, 18. Sep. 2015 (CEST)[Beantworten]
Wenn wir uns mal ganz von den Fragen nach Begriff und Existenz lösen: hilft a priori? Oder ist das zu allgemein? --Chricho ¹ ³ 23:28, 18. Sep. 2015 (CEST)[Beantworten]
"A priori" ist mir bekannt, ich wüsste nicht wie das weiterhelfen könnte. --Cubefox (Diskussion) 01:09, 19. Sep. 2015 (CEST)[Beantworten]
Aber egtl. ist die Antwort ganz einfach: Nichts existiert. Siehe Gorgias:

„Wenn sich nun jemand Wagen denkt, die auf dem Meere fahren, so muß er, auch wenn er es nicht sieht, glauben, daß es Wagen gibt, die auf dem Meere fahren. Dies ist aber ungereimt. Folglich wird das Seiende nicht gedacht oder begriffen.“

ein lächelnder SmileyVorlage:Smiley/Wartung/:)  --Chricho ¹ ³ 23:44, 18. Sep. 2015 (CEST)[Beantworten]
Der Denkfehler der Frage ist die Einschränkung von Existenz nicht nur auf Eigenschaften von Dingen, sondern sogar auf sprachlich abstrakte Eigenschaften von Dingen. Kann man machen, aber dann sprechen wir von ein paar Logik-Fingerübungen. "Existenz" kann aber soviel mehr bedeuten. Und natürlich liegt sie auch vor dem Denken. Also was soll es sein? --Gamma γ 00:24, 19. Sep. 2015 (CEST)[Beantworten]
Will ich nicht abstreiten, aber man kann sich auch ausgehend von den sprachlich-abstrakten und logischen Angelegenheiten fragen, was denen zugrunde liegt, was da passiert – dann kann auch da etwas passieren, was vllt. nicht nur Fingerübung ist. --Chricho ¹ ³ 00:43, 19. Sep. 2015 (CEST)[Beantworten]
Soso, und was meinst du liegt dem zugrunde? Als Übungsaufgabe heute: "Gibt es Amerika wirklich?" Oder das mit Merkel und Harry Potter da oben. Was "existiert" und was kennt man "nur" vom Hörensagen? Oder ist alles nur Hörensagen mithin sprachlich-abstrakt? Das Ende vom Lied ist immer: man kommt da nicht mehr raus und dem liegt auch nichts zugrunde, was man nicht selbst eingebracht hat. Also will man Aussagen über die Existenz machen (passt auf einen Bierdeckel und viel Sinnvolles ist nicht dabei) oder Aussagen über Aussagen über die Existenz, also z.B. über Eigenschaften von Dingen. Es ist erstaunlich wie schwer es oft ist den Unterschied zwischen "Wirklichkeit" und der Vorstellung von der Wirklichkeit zu sehen/verstehen. --Gamma γ 01:24, 19. Sep. 2015 (CEST)[Beantworten]
Solche Frage zu stellen, kann nicht heißen, einen Grund darunter freilegen zu können, in dem das zu Erklärende (etwa die Logik-Fingerübungen) noch nicht vorausgesetzt sind. Das heißt aber nicht, dass bei der Frage nichts passieren würde, das Hineinlegen auf „meine Freiheit“ reduzierbar wäre. Und ja alle Sätze sind nur Hörensagen, aber dabei passiert etwas. Eine Existenz, die das alles erst möglich machen soll, ist nicht die Existenz, die durch das Wort „Existenz“ benannt werden kannn, und zugleich ist es nur die Sprache, in der sich solche Unmöglichkeit formulieren lässt. Insofern von der Sprache aus fragen, den Grund nicht einfach setzen können. Die Übungsaufgaben fallen mir wohl nicht leichter als dir. --Chricho ¹ ³ 01:51, 19. Sep. 2015 (CEST)[Beantworten]
@Gamma: Um mal mit „Gamma erklärt die Welt“ zu antworten: Du sprichst dort von der philosophischen Exoterik und der Mystik als zwei entgegengesetzten Haltungen, Seiten, die aber auf selbige Erkenntnis hinauslaufen. Ich wollte schlicht die Exoterik verteidigen. Sie hat zudem den Vorteil, dass sie beim Schweigen nicht halt macht, sondern dieses weiter in sich einfügen kann, die Bewegung noch einmal von vorne mit zusätzlichem Inhalt anfangen kann. Dafür macht sie allerdings auch nicht zufrieden. Und auch du bekennst dich, indem du schreibst, zur Exoterik, und auch du scheinst ja nicht ganz zufrieden zu sein. Gut, du sprichst von Kunst, die geht wohl eher von der anderen Seite aus als die Philosophie, aber ebenso ohne „sofort, überall und von Jede[n] direkt“ zufrieden zu stellen (und zudem kauf ich es dir nicht ab, „Gamma erklärt die Welt“ ist keine Kunst, so wenig wie andere Philosophie/Exoterik auch). --Chricho ¹ ³ 14:36, 19. Sep. 2015 (CEST)[Beantworten]
Von Eigenschaften hatte ich eigentlich gar nicht gesprochen, nur von Dingen. Was meinst du mit "sprachlich abstrakten Eigenschaften von Dingen"? So wie ich das Wort "abstrakt" verstehe, können nur Begriffe abstrakt sein. Siehe auch wiktionary:abstrakt. Zur Bedeutung von Existenz: Ich meine Existenz hier hauptsächlich im physischen oder physikalischen Sinne. Gefühle, Begriffe etc. würde ich aber wohl auch einschließen. Dass abstrakte Begriffe wie Zahl oder Existenz außer als Begriff auch noch irgendwie in einer platonischen Ideenwelt existieren könnten (siehe mein Kommentar weiter oben), würde ich mal außen vor lassen. Wenn du mit "Existenz liegt vor dem Denken" meinst, dass Dinge auch existieren können ohne dass sie jemand bedenkt oder beobachtet, dann stimme ich dir zu. Ich wüsste aber nicht wie das meine Ausgangsfrage beantworten könnte. (Kann aber sein dass ich auf dem Schlauch stehe. ;) --Cubefox (Diskussion) 01:07, 19. Sep. 2015 (CEST)[Beantworten]
Mit „sprachlich abstrakten Eigenschaften“ meint Gamma wohl das, was du oben „Begriffe“ nanntest. „Abstrakt“ also etwas weiter verstanden als bei dir, auch Begriffe wie „Pferd“ einschließend. --Chricho ¹ ³ 01:15, 19. Sep. 2015 (CEST)[Beantworten]
Sorry wg. BK, mein editfenster war ewig lang offen. Der Schlauch ist wohl, dass du Existenz als Eigenschaft verstehst. A existiert oder A schläft. Alles Abstrakta und dann kommt noch dazu, dass Dinge nicht einfach so existieren. "Jungesselle existiert". Was bedeutet das genau. Wer ist wann, warum genau "Junggeselle". Das ist nicht einfach so "physisch oder physikalisch". Sowas gibt (sic! :-)) es garnicht, auch die Physiker einigen sich oft lang und beschwerlich welches Teilchen jetzt "existiert". Das ist alles Denken. Das juckt die Existenz nicht, also die Existenz, die alles erst möglich macht. --Gamma γ 01:38, 19. Sep. 2015 (CEST)[Beantworten]
Um doch noch die Frage in dem Sinn, den du vermutlich gemeint hast zu beantworten: ja, es gibt in diesem sprachlogischen Sinn auch Tautologien und es gibt eine Reihe von Abstufungen. Z.B. "Das ist wahr oder nicht wahr"; "Eine Rose ist eine Rose". Oder selbstbezüglich: Dies ist ein Gedanke. Oder metamäßig: Das ist ein Satz. Oder meinst du sowas wie: „HIV-Virus“? Dann gibt es einfach: „Wort“ oder dadaistisch: „Dada“ – weil ja „Dada“ nun existiert, wenn ich es ausspreche oder denke. Das ist auch der einzige Punkt wo gedankliche Abstraktion einfach ist/existiert und nicht Denkinhalte Aussagen über Dinge, die auch Inhalte des Denkens sind, formulieren. Kontradiktionen sind da viel einfacher, weil die Existenz der Aussagenteile schon postuliert wurde, einfach durch die Verwendung des Begriffes. Ich hoffe das war jetzt verständlich. Wenn es nicht leicht zu verstehen ist, dann weil der Mensch auch gern so ein präexistentes Ding wäre, also ein Ding, dem man posteriori die Eigenschaft "existiert" zuschreiben kann und damit Kontrolle gewinnen kann, obwohl das "Ding" dazu ja schon existieren muss. Für den Alltag und seine Logik und/oder für Wikipedia hat diese Unterscheidung kaum Relevanz, weil es hier aus diesen guten Gründen eh nie um die Wirklichkeit (was immer das ist) geht, sondern und die Auswertung relevanter Quellen gedanklich-abstrakter Inhalte. --Gamma γ 10:35, 19. Sep. 2015 (CEST)[Beantworten]
Ich hoffe mal, ich gehe nicht ganz falsch, wenn ich als hoffentlich entfernt zum Thema gehörig einwerfe, was ich an dieser Stelle mal geschrieben habe: Diskussion:Ontologie#Wesenslehre. --79.198.2.187 11:15, 19. Sep. 2015 (CEST)[Beantworten]
Ich denke nicht dass man Existenz sinnvoll als Eigenschaft verstehen kann, weil eine Eigenschaft einen Ding entweder zukommt oder nicht. Es existieren aber keine Dinge die eine hypothetische Eigenschaft "Existenz" nicht hätten. Die Aussage "Ein Ding hat die Eigenschaft Existenz" wäre eine Tautologie und bedeutungslos, sie würde nur heißen "was existiert das existiert". Ich würde stattdessen nur sagen, die Aussage "x existiert" ist abhängig von x entweder falsch oder eine Tatsache, nicht aber eine "Eigenschaft" von x. Oder wie weiter oben geschrieben: Ein Begriff kann sich auf etwas beziehen das existiert, oder auf etwas das nicht existiert (eine Fiktion).
Zu "Junggeselle existiert": Wie ich weiter oben über "Mensch" geschrieben hatte, gibt es natürlich nicht einen prototypischen "Junggesellen an sich", das ist nur eine Abstraktion, es gibt aber konkrete Junggesellen aus Fleisch und Blut. Verheiratete Junggesellen dagegen gibt es nicht, und es kann sie auch nicht geben. Weshalb es aber offenbar nichts gibt, das notwendig existiert, weiß ich nicht.
Zu Physikern und Teilchen: Natürlich wird in der Physik auch von hypothetische Teilchen gesprochen, bei denen fraglich ist ob es sie gibt, die vorerst nur "auf dem Papier existieren" weil sie nicht empirisch bestätigt wurden. Die Existent von vielen anderen Teilchen ist aber unter Physikern unstrittig. Als Solipsist könnte man natürlich trotzdem die Existenz von allem und jedem anzweifeln (von Teilchen inkl. Physikern), denn vollkommen sicher kann man sich auch bei aller Empirie nicht sein. Das mag eine erkenntnistheoretisch berechtigte Position darstellen, für sonderlich wahrscheinlich hält sie aber meines Wissens kaum jemand.
Zu den Tautologien: Die sind immer wahr, aber mit Existenz haben sie meines Erachtens nichts zu tun. (Im Gegensatz zu Widersprüchen, die Nicht-Existenz notwendig implizieren.) Dass eine Rose eine Rose ist, heißt nicht, dass es Rosen gibt. Die Aussage "Pegasus hat Flügel", könnte man vermutlich mit gutem Willen auch als wahr (sogar notwendig wahr) bezeichnen, aber Pegasus existiert deswegen noch lange nicht. Dass ein Begriff des Pegasus (oder von Dada) in gewissem Sinne tatsächlich in meiner Vorstellung existiert, ist demgegenüber etwas anderes. Nämlich der Unterschied zwischen Begriff und Ding/Referent, auf den ich oben mit Chricho eingegangen bin. --Cubefox (Diskussion) 13:16, 19. Sep. 2015 (CEST)[Beantworten]
Wer die „Existenz nur auf dem Papier“ abgrenzt von der „unstrittigen Existenz“, verkennt die physikalische Praxis, verkennt sowohl, was „unstrittige Existenz“ in der Praxis von empirischer Überprüfung von Theorien bedeutet, als auch, was ein Teilchen in theoretischer physikalischer Betrachtung ist. Da hält sich die Physik nicht einfach an irgendein olles (alltags)metaphysisches Schema – wenn, dann nur indirekt und notwendigerweise indirekt. Oder anders ausgedrückt: Das Papier ist notwendig für die Existenz. --Chricho ¹ ³ 14:41, 19. Sep. 2015 (CEST)[Beantworten]
Ich denke eher, dass jemand, der eine empirisch völlig ungeprüfte Theorie mit einer empirisch ausgiebig getesteten Theorie gleichsetzt, die physikalische Praxis verkennt. Du wirst keinen Physiker finden, der die Existenz von SuSy-Teilchen für so sicher hält wie die Existenz von Elektronen. --Cubefox (Diskussion) 19:42, 19. Sep. 2015 (CEST)[Beantworten]
Du wirst Physiker finden, die keine wissenschaftlichen Realisten sind. Wie du darauf kommst, dass ich „die Eistenz von SuSy-Teilchen für so sicher [halte] wie die Existenz von Elektronen“, weiß ich nicht. --Chricho ¹ ³ 21:47, 19. Sep. 2015 (CEST)[Beantworten]
Du weißt es nicht, aber bestreitest du es auch nicht? Das war nämlich das, was ich oben sagen wollte, was du dann aber wohl missverstanden hattest. Nur "auf dem Papier" war natürlich salopp (deswegen auch in Anführungszeichen), gemeint war unsichere bzw. zweifelhafte Existenz im Vergleich z.B. zu Elektronen. --Cubefox (Diskussion) 17:18, 20. Sep. 2015 (CEST)[Beantworten]
Es gibt nicht einfach verschiedene Teilchenarten, für deren Existenz sich Wahrscheinlichkeiten angeben lassen. Es gibt verschiedene Theorien mit verschiedenen Konzepten, die sich Elektronen oder Selektronen nennen, die sich miteinander in ihrer Anwendbarkeit vergleichen lassen. Selbst eine Spielart von wissenschaftlichem Realismus wie der Entitätenrealismus entscheidet den unterschiedlichen Status solcher verschiedenen Teilchen nicht mit irgendeiner Wahrscheinlichkeit. --Chricho ¹ ³ 18:01, 20. Sep. 2015 (CEST)[Beantworten]
Gefällt dir das Wort "plausibler" besser? Ich habe den Eindruck, dass "wahrscheinlich" und "plausibel" häufig synonym verwendet werden, selbst wenn man keine genaue Wahrscheinlchkeit angeben kann. Jedenfalls denke ich, dass die allermeisten Physiker die Aussage "Elektronen existieren" für sehr viel wahrscheinlicher/plausibler halten als "Selektronen existieren". Eben weil die Theorie, in deren Rahmen sie postuliert werden, bisher empirisch nicht gestützt wurde. Wenn du meine Annahme ob der Überzeugung von Physikern nicht teilst, dann ist das eben so. Wie gesagt kann ich hier keine repräsentativen Meinungsumfragen machen und du auch nicht. Da gibts dann auch nicht mehr viel zu diskutieren. --Cubefox (Diskussion) 22:41, 20. Sep. 2015 (CEST)[Beantworten]
90% von dem was hier geredet wurde ist nicht mal wissenschaftlich empirisch/logisch definiert, das ist der typische Konflikt zwischen der Philosophie und der Physik. Die Philosophen reden von Dingen die wissenschaftlich gesehen nicht mal eine empirische Definition haben. Genauso könnte ich Fragen was die Farbe des Grundgesetzes ist oder eine zufällig Reihe von Buchstaben angeben. Wenn man Wissenschaft machen will, dann muss man sehr vorsichtig auf seine Sprache achten. Hier ein Musterbeispiel von Carnap: Cogito_ergo_sum#Carnaps_Sprachanalyse--Lexikon-Duff (Diskussion) 17:06, 21. Sep. 2015 (CEST)[Beantworten]
Carnaps Philosophie ist Wissenschaft und andere Philosophie nicht? --Chricho ¹ ³ 18:53, 21. Sep. 2015 (CEST)[Beantworten]
Logischer Positivismus ist eine Fundamentalphilosophie, und Grundlage und Prinzip der modernen Wissenschaften. Metaphysik dagegen ist, wie Carnap meint, eine Art von Kunst die zur Analyse dient.--Lexikon-Duff (Diskussion) 20:14, 22. Sep. 2015 (CEST)[Beantworten]
Nehmen wir das „Musterbeispiel“: Steht das irgendwie außerhalb der Metaphysik? Grundlage der Wissenschaften zu sein, behaupten indes so manche. Und deine Äußerung trägt nichts dazu bei, die Verwirrung zu erklären. Was eine „empirische Definition“ ist, weiß ich indes nicht. Definieren ist indes nicht der einzige Weg, sehr sorgfältig auf die Sprache zu achten. Solch eine Sorgfalt jenseits des Definierens brauchst du selbst in der Metamathematik. --Chricho ¹ ³ 23:34, 22. Sep. 2015 (CEST)[Beantworten]
Die großen Paradigmenwechsel in der Physik hatten immer auch etwas mit der Anwendung des logischen Positivismus zu tun, indirekt oder direkt.
  • Einstein: Der Unterschied ob die Kraft durch die Gravitation ausgelöst wird oder durch eine Beschleunigung des Systems ist unmessbar und darum gleichwertig. (Äquivalenzprinzip in der allgemeinen Relativitätstheorie), die Äthertheorie ist bedeutungslos. Einstein besuchte übrigens auch den Wiener Zirkel.
  • Quantenmechanik: Die Frage nach der wahren Natur der Wellenfunktion ist bedeutungslos und nie erfahrbar, aus diesem Grund sind alle Interpretationen der QM gleichwertig.
  • Eichtheorien: Eine Theorie ist nicht abhängig von ihrer Eichung. Der Unterschied ist nicht erfahrbar und somit kann ich eine beliebige Eichung benutzen und habe eine gleichwertige Theorie.
  • Black hole complementarity (holographisches Prinzip): (stark vereinfacht ausgedrück) Der Unterschied zwischen dem inneren eines schwarzen Loches und dem äußeren ist nicht erfahrbar, darum sind beide Bereiche gleichwertig.
  • S-Matrix Theorie. Alle beobachtbaren Größen in der Physik werden durch eingehende und ausgehende Streuamplituden dargestellt die aus der Unendlichkeit kommen und in die Unendlichkeit gehen. Der Rest ist bedeutungslos (extremste Form des logischen Positivismus). Dies beantwortet auch die Frage die von der stringtheorie gestellt wird. Was ist ein Entfernungsmaß in einer Raumzeit die mathematisch nicht definiert ist?.

Und das ist auch kein Wunder, empirisch undefinierbare Objekte existieren nicht in der Wissenschaft. Was ich mit empirisch definiert meine hat Carnap gut erklärt in seinem Aufsatz "Überwindung der Metaphysik durch logische Analyse der Sprache". Man braucht erst mal einen Fundamental satz wie z.B. "x ist ein Gott", dann wird x definiert durch Eigenschaften die man mit empirischen Mitteln identifizieren kann. Nehmen wir mal Arthropode. "x ist ein Arthropode" nun kann man anhand empirischer Methoden festlegen (Verifikationsmethoden) was ein Arthropode ist z.B. "x ist ein Tier, x hat einen gegliederten Körper, x hat einen Panzer aus Chitin, daraus folgt "x ist ein Arthropode", weitere Wörter müssen ähnlich auf primitivere Wörter die empirisch identifiziertbar sind zurückgeführt werden. Carnap hat gezeigt das alle klassischen Fragen der Metaphysik wie Gibt es einen Gott?, Was ist der freie Wille?, Was ist das nichts? usw. alle bedeutungslos und Rahmen abhängig sind, wie man z.B. die natürlichen Zahlen durch arabisch oder griechisch Ausdrücken kann. Oder eben wie in einer Eichtheorie in der ich die Eichung beliebig wechseln kann ohne das die Physik sich ändert. Das logischer Positivismus selbst nicht durch seine Anforderungen definiert werden kann ist übrigens nicht wichtig. Da er eine Fundamentalphilosophie ist, wie ein Axiom das keiner Begründung bedarf.--Lexikon-Duff (Diskussion) 17:14, 23. Sep. 2015 (CEST)[Beantworten]

Nun gut, wenn du dein Denken durch Ziehen eines Tickets gründen willst, kann ich dagegen nicht anreden. Indes bietet deine Ausführung Cubefox keinen Ansatz für irgendeine Einsicht darin, wie seine Frage zustande kommt (auch für Kunst reicht es nicht, sie eben als Kunst abzutun). Aber auf das Physikbild will ich doch eingehen: Indem du das alles unter das Schlagwort logischer Positivismus bringst, verkennst du die Paradigmenwechsel. Die Quantenmechanik zeichnet nicht aus, dass es mehrere gleichwertige Interpretationen gibt, sondern, dass eine lokale Beschreibung eben so etwas wie einen Kollaps der Wellenfunktion erfordert. Es gibt nämlich auch ungleichwertige Interpretationen, nämlich unmögliche, falsche, und durch deren historisches Scheitern hat sich die Quantenmechanik als Paradigmenwechsel herausgestellt. Eine „S-Matrix Theorie“ gibt es nicht, geschweige denn, dass die Berechnung von Streuamplituden einen solchen Absolutheitsanspruch hätte, selbst wenn wir uns auf Kern- und Teilchenphysik beschränken, ist dem nicht so, schließlich sollen ja auch Bindungszustände beschrieben werden, da hilft die S-Matrix zwar auch, dennoch ist er etwas anderes als eine Streuamplitude, diese beschreibt zunächst einmal (asymptotisch) eine Wahrscheinlichkeit eines Übergangs von einem Zustand nicht-wechselwirkender Teilchen in einen anderen. Was die zahlreichen mathematisch nicht oder kaum definierten Sachen in QFT und Stringtheorie sind, wird nicht dadurch gelöst, zu sagen, das sind die Verifikationsmethoden, da die Frage gerade die ist, was die sein sollen, wie man zu denen kommt, und das ist bei ungeklärten mathematischen Grundlagen eben nochmal verwickelter als etwa in der Quantenmechanik. Übrigens überrascht mich, dass du Galilei nicht nennst. Grüße --Chricho ¹ ³ 14:30, 24. Sep. 2015 (CEST)[Beantworten]
@Chricho: Ich hab auch nicht gesagt das sich die QM dadurch auszeichnet, sondern das das einfach ein Fakt ist und auch die Wahrscheinlichkeitsinterpretation auf diesem Wege zu den erfolgen geführt hat indem man einfach nicht mehr den Sinn der Wellenfunktion hinterfragt hat, sondern nur eben das beste framework. Und klar gibt es inkonsistente QM Interpretationen. Und das beste framework der Physik ist heutzutage S-Matrix + Regge Theorie vereint in der stringtheorie, das es natürlich noch Probleme gibt ist ja klar.--Lexikon-Duff (Diskussion) 14:16, 26. Sep. 2015 (CEST)[Beantworten]
(Beitrag an die Stelle verschoben auf die Bezug genommen wurde. Sonst kenn sich hier niemand mehr aus.) --Cubefox (Diskussion) 20:22, 26. Sep. 2015 (CEST)[Beantworten]
Lieber Lexikon-Duff, zu den Definitionen: Was genau ist denn an meiner Fragestellung unklar? Was ich hier mit "existieren" meine habe ich zum Beispiel an mehreren Stellen präzisiert. Und wenn du die Frage für ein Scheinproblem hältst, dann solltest du zeigen können was genau mit der Fragestellung nicht stimmt. Der allgemeine Verweis auf den logischen Positivismus bringt mir wenig. (Der ist mir zwar sympathisch, wobei er vor allem dafür bekannt ist, dass er nicht halten konnte was man sich von ihm versprach.) Ich weiß zwar nicht genau was du mit einer "logischen Definition" meinst, aber z.B. kontingent, notwendig, möglich und unmöglich sind tatsächlich Begriffe aus der Logik. Und nein, Fragen lassen sich natürlich nicht immer nur mit Empirie beantworten. Auf eine empirische Antwort hatte ich hier auch gar nicht gehofft (ich bezweifle dass sie es in diesem Fall geben kann), sondern auf eine begrifflich-logische. --Cubefox (Diskussion) 20:41, 26. Sep. 2015 (CEST)[Beantworten]

[ad hoc brinstorm, ohne thread gelesen zu haben:] Junggeselle und verheiratet paßt einfach nicht zueinander, .. es sind Eigenschaften \ Attribute [edit, 26,09, 14:50 :] \ Akzidenzien Sein#Kategoriale_Bestimmung_des_Seienden [end edit --217.84.99.168 14:52, 26. Sep. 2015 (CEST)] , die sprachlich-logisch nicht kombinierbar sind. Daraus zu schließen, daß ein Etwas: "verheirateter Junggeselle" etwas [edit, 26,09, 14:51 :] (subtanziell) [end edit --217.84.99.168 14:54, 26. Sep. 2015 (CEST)] 'Nicht-Existentes' ist, bedeutet vielleicht ein Paradoxon - der Sinn von Sprache und Kommunikation wird (nihilistisch gefärbt) hintergangen - die Logik der Sprache erfordert mögliche Kombinationen, damit sie Sinn ergibt - Man kann für einen "Ball" ausschließen, daß er eckig ist .. das heißt nicht, daß es einen "eckigen Ball" gibt, der "nicht-existent" ist.[Beantworten]

Wir Menschen haben die Fähigkeit entwickelt kausale Zusammenhänge zu erkennen und in Zukunft und Vergangenheit zu blicken. Die so teils vorhersehbaren Wechselwirkungen der Natur waren unserem Bestandhaben förderlich. Der Mensch hatte nun mit möglichen Entwicklungen in der Zukunft, basierend auf Ausgangswissen, zu tun, dem inneren Vorstellungsraum [ Hoimar von Ditfurth ]. Damit war auch der Unsinn geboren, wenn Bilder \ Zusammenhänge \ Schlußfolgerungen \ Entwicklungen schlicht in falschen Kombinationen gedacht \ gesehen \ sich vorgestellt wurden .. zB Trial and Error mußten noch filtern. Daß es keine "Eckige Kugel" gibt, ist Dadaismus pur .. "Ceçi n'est pas une pipe" .. sprachlich-logischer Unsinn (es ist keine Schlußfolgerung aus Bekanntem, sondern Voraussetzung für die Begriffe "Kugel" und "eckig" und rund" und "kantig", daß sie richtig benutzt werden innerhalb des Sprachsystems Geometrie und deren Begriffe), sobald man daraus eine "Nicht-Existenz" postuliert und "Notwendigkeit" sucht. Sich selbst negierende Sprachanwendung jenseits plausibler Realität.

Von der dunklen Materie ist nur die Eigenschaft der umliegenden Materie bekannt, mehr zu gravitieren, als nach bekannten Gesetzen zu erwarten wäre .. offenbar existiert (notwendigerweise?) etwas, das diesen Effekt bewirkt oder es fehlt einfach noch die (eine andere) Erklärung.

Mich mutet daher befremdlich an, daß aus irgendeinem Dialoguniversum (System von mit Sprache Gemeintem in Kontexten), das notwendigerweise immer von menschlichem Wahrnehmungsapparat wahrgenommen und vom Verstand auf vermeintliche Realität angewandt \ interpretiert \ausgelegt \ gedeutet wird - .. daß sich aus einer solchen Sprachanwendung irgendwelche Aussagen \ Rückschlüsse auf "Existenz" im weiteren Sinne \ ontologisch \ universell treffen lassen. Möglicherweise ist die bloße Tatsache, daß das Universum existiert, notwendig, weil es damit jemals nachweislich existiert (hat). Es gibt dann dazu, daß es nicht endloses ewiges Nichts gibt keine Alternative. Wie sehr irgendetwas in oder an diesem Universum notwendig ist, könnte wenn überhaupt nur mit zB Simulationen der Ergebnisse ontologischer Kalküle (mit verschiedenen - gar veränderlichen Axiomensätzen auf unseren Begriffen, wie "Entstehung, Möglichkeit, Beständigkeit, Eigenschaften, Träger letzterer, Unterscheidungsmerkmale", (Wechsel-)Wirkung, usw.) ohne(!) Anfangsbedingungen wissenschaftlich untersucht werden. ;o]) Danke für die Frage lol. --217.84.103.91 14:41, 25. Sep. 2015 (CEST) "nicht von der Referenz ausgehend fragen" o.s.ä. : Im System Sprache schließen sich "verheiratet sein" und "Junggeselle sein" aus. "Junggeselle" und "verheiratet" sind mehr oder weniger verbindlich definierte Begriffe innerhalb des Systems Sprache. Da ist dann auch nix mehr `notwendig´ oder `unmöglich´ oder `existent´ im ontologischen Sinne, auf Realität der Welt und des Universums bezogen, sondern einfach "nicht definiert", "kommt nicht vor", "ist nicht Teil der mit Sprache bezeichenbarem Inhalt \ Gemeintem". So, wie man nicht durch "0" teilen kann .. da ist auch nix `notwendig´ oder auch nicht: " ein Ergebnis durch ``0´´ ist `nicht-existent´ " .. --217.84.103.91 14:55, 25. Sep. 2015 (CEST) .. und noch .. Es gibt keine universell gültige Logik, weder als Grundlage von (auch `exakter´) Wissenschaft, noch in der Realität. Wir sind (letztlich im Erkennen von Realität) immer befangen in unserer Wahrnehmung und unserer Deutung wissenschaftlicher Ergebnisse im Gesamtzusammenhang Welt \ Universum. Begriffe, wie "notwendig", "möglich / unmöglich" im ontologischen Sinne, rühren an die Grenzen unserer Fähigkeit, Welt zu erkennen. Der mE beste Ansatz diese Wirren zu umschiffen, ist, die Evolution des Verstandes, der diese Wirren hervorbringt zu erforschen (Evolutionäre_Erkenntnistheorie). --217.84.103.91 15:07, 25. Sep. 2015 (CEST)[Beantworten]

Modallogik#Die_zugrundeliegende_Intuition [erster Absatz, letzte Sätze:] " Außerdem gibt es Aussagen, die in jeder vorstellbaren Situation wahr sind. Solche Aussagen bezeichnen wir als notwendig. Notwendige Aussagen sind z. B. „Kreise sind rund“ und „Junggesellen sind unverheiratet“. " --217.84.65.106 04:10, 26. Sep. 2015 (CEST)[Beantworten]

Beitrag nach links gerückt, sonst sieht das wie eine Antwort auf die IP aus. --Cubefox (Diskussion) 20:22, 26. Sep. 2015 (CEST)[Beantworten]
Junggeselle und verheiratet paßt einfach nicht zueinander - Eben, es handelt sich um einen Widerspruch. Hatte ich in der Ausgangsfrage auch schon angemerkt.
das heißt nicht, daß es einen "eckigen Ball" gibt, der "nicht-existent" ist. - Natürlich existiert nichts das nicht existiert. Es hat meines Erachtens auch auch niemand etwas anderes behauptet.
Daß es keine "Eckige Kugel" gibt, ist ... sprachlich-logischer Unsinn ..., sobald man daraus eine "Nicht-Existenz" postuliert - Dass es keine eckigen Kugeln geben kann ist kein "Postulat", sondern eine begriffsanalytisch beweisbare Tatsache. Wenn es Kugeln gibt sind sie rund, das folgt aus dem Begriff der Kugel, und wenn es etwas eckiges gibt ist es nicht rund, das folgt aus dem Begriff der "Eckigkeit". So weit so trivial. Aber in meiner Ausgangsfrage ging es ja nicht darum, warum es begrifflich Widersprüchliches nicht geben kann (= notwendig nicht gibt), sondern warum es offenbar auch nichts notwendig gibt. Das ist nämlich offensichtlich nicht so einfach zu beantworten.
Möglicherweise ist die bloße Tatsache, daß das Universum existiert, notwendig, weil es damit jemals nachweislich existiert (hat). - Wenn die Existenz des (oder irgendeines) Universums notwendig wäre, müsste sich dafür ein logischer Grund angeben lassen, man müsste soagr beweisen können warum dessen Existenz notwendig ist. Im Halbsatz "weil es damit jemals nachweislich existiert (hat)" sehe ich keinen solchen Beweis, ich weiß nicht einmal was er mit der Hypothese zu tun hätte. Dass etwas existiert heißt ja noch nicht dass dessen Existenz auch notwendig wäre. Es scheint ja eher umgekehrt so zu sein, dass alles was existiert, auch nicht existieren könnte. Dass es etwas gibt ist also offenbar nur eine kontingente, keine notwendige Tatsache. Es könnte natürlich auch sein dass ich mich damit irre (dann gäbe es tatsächlich etwas das notwendig existiert) oder ich irre mich nicht, und es gibt notwendig nichts, das notwendig existiert (d.h. es kann nichts notwendig geben). Dass letzteres der Fall ist erscheint mir deutlich plausibler, aber ich wüsste nicht wie das zu begründen (geschweige denn zu beweisen) wäre. Daher die Ausgangsfrage.
Der mE beste Ansatz diese Wirren zu umschiffen, ist, die Evolution des Verstandes, der diese Wirren hervorbringt zu erforschen (Evolutionäre_Erkenntnistheorie). - Ich wüsste nicht wie die Evolutionäre Erkenntnistheorie irgendetwas Relevantes zur Ausgangsfrage beitragen könnte (ich hatte sie weiter oben in anderem Zusammenhang übrigens schon mal erwähnt).
Modallogik#Die_zugrundeliegende_Intuition [erster Absatz, letzte Sätze:] " Außerdem gibt es Aussagen, die in jeder vorstellbaren Situation wahr sind. Solche Aussagen bezeichnen wir als notwendig. Notwendige Aussagen sind z. B. „Kreise sind rund“ und „Junggesellen sind unverheiratet“. " - Ja und? Das ist richtig. Aber Existenzsätze zählen offenbar grundsätzlich nicht zu den notwendigen Aussagen. Dass Junggesellen notwendig unverheiratet sind impliziert nicht, dass es Junggesellen notwendig gibt. "Pegasus ist geflügelt" ist auch notwendig, trotzdem gibt es Pegasus nicht. --Cubefox (Diskussion) 23:08, 26. Sep. 2015 (CEST)[Beantworten]
Teilen durch Null ist nicht widersprüchlich (auch nicht notwendig oder unmöglich oder kontingent), sondern nicht definiert. Teilen durch Null ist keine gültige Operation im System: Mathematik. Es ist auch kein Begriff und "verheirateter Junggeselle" ist auch kein Begriff, sondern eine unsinnige Kombination von Akzidenzien. Und "verheiratete Junggesellen" sind keine "Dinge, deren Begriff widersprüchlich ist". "Verheiratete Junggesellen" sind im System Sprache nicht definiert, sie sind keine gültige Kombination von Eigenschaften einer Person. Derlei unerlaubte Konstrukte irgendwie weiterzudenken zB sie auf eine Substanz: das `Ding´ "verheirateter Junggeselle", das nicht-existent sein soll, oder zB Notwendigkeit oder Möglichkeit oder sonstwas einer solchen Existenz oder Nicht-Existenz (wovon denn jetzt genau?) darin zu suchen, ist unlogisch.
"Sein, Existenz (an sich)" (das, was allem Seienden gemeinsam ist, nach Abzug sämtlicher individueller Eigenschaften) und (tatsächlich \ konkret \ beschreibbar \ bezeichenbar) "Seiendes" wird im ganzen thread unwidersprochen vermischt. Eine (idealisierte, mehr oder weniger wohldefinierte) Kugel in der Geometrie und eine Kugel als Begriff für allerlei Kugeliges und eine (oder allerlei) Kugel(n) in der Wirklichkeit sind völlig verschiedene Dinge. Das Wesen einer Kugel wird nicht dadurch nicht-existent, daß man sich ein "eckiges Kugel" ausdenkt, .. ebensowenig ist dieses "verheiratetes eckiges Kugelgesell" irgendwie bestimmbar \ näher bezeichenbar, auch nicht ``nicht-existent´´ als es näher bezeichnende Eigenschaft, denn da ist nichts zum bezeichnen: es ist überhaupt nicht definiert, es kommt gar nicht vor. Deshalb kann es auch nicht `notwendig-nicht-existent´ oder `notwendig-unmöglich´ sein. Es ist überhaupt nichts.
Da unser Universum existiert, gibt es "nie und nirgendwo ein Universum" bzw "kein Universum" nicht. Da Existenz von irgendetwas überhaupt, wie auch von allem in einem Universum, praktisch dasselbe ist, wie ein Universum, ist die sinnvolle Feststellung, daß es existiert zugleich die Erkenntnis, daß es das notwendig tut, da es im Begriff "Existenz" enthalten ist, .. der Begriff "Existenz" ist so definiert, dafür wurde er `erfunden´. Oder salopp: Da es das Universum nunmal gibt, wie könnte es anders sein? Sollte es schon einmal `gefehlt´ haben, .. mitten im `Nichts´ (das es, wie man rundum sieht ja nicht gibt)??
Im Sinne der EET läßt sich aufzeigen, wie wir "Existenz" wahrnehmen, warum wir ein Wort dafür haben, welchen Begriff wir uns davon machen, wierso unser Gehirn nicht-Seiendes überhaupt denken kann, und wie gut unser Bild von "Existenz" auf tatsächlich Existierendes zutrifft. Alle irreführenden teufelskreisigen ~Ismen (und etwaige Wirren um Notwendigkeit von nicht-Seiendem oder vielerlei Kopfgeburten) sind durchbrochen und vom Tisch. Sogar, was Logik eigentlich ist, kann sie uns lehren.
"Notwendige Aussagen sind z. B. „Kreise sind rund“ und „Junggesellen sind unverheiratet“. " - Ja und? Das ist richtig. Aber Existenzsätze zählen offenbar grundsätzlich nicht zu den notwendigen Aussagen. " ..
Hä? A=B, aber B=!A ??
Verheiratete Junggesellen sind Dinge, deren Begriff widersprüchlich ist. Pegasus gibt es nicht. - Ausgangsfrage ist ungültig. --217.84.83.134 03:14, 27. Sep. 2015 (CEST)[Beantworten]

Dein Problem ist wohl vor allem in der Sprache zu suchen. „Es gibt keine verheiratete Junggesellen“ sollte man erstmal formalisieren zu: . Es existiert also kein mit disen Eigenschaften.

Der Trick ist, dass es nicht möglich ist, eine tautologische Aussage durch Verneinung des Quantors zu erhalten, sehr wohl aber durch Verneinung des Inhalts: .

Aussagen können tautologisch oder kontradiktorisch sein (oder normal, was hier uninterressant ist), und dadurch kann man zeigen, dass alle x oder alle x nicht (bzw. kein x) diese Eigenschaft erfüllen können. Sowas gilt aber stets für alle Individuen nicht für mindestens eine. Wenn etwas für alle Individuen gilt, kann es immer auch sein, dass es für gar keins gilt, wenn keins existiert. 100% befriediegend ist das vielleicht nicht, ich konnte meine Gedanken nciht rihtig ausrücken, aber vielleicht wird es dadurch ein bissachen klarer… -- David23x (Diskussion) 18:18, 27. Sep. 2015 (CEST)[Beantworten]

"Verheiratete Junggesellen" sind im System Sprache nicht definiert, sie sind keine gültige Kombination von Eigenschaften einer Person. - Dass irgendwelche Aussagen im System x "nicht definiert" seien gilt nur für formale Systeme, wo alle "definierten" Aussagen aus Axiomen abgeleitet werden können. Bei natürlicher Sprache handelt es sich aber nicht um ein formales System. Hier können beliebige Widersprüche ausgedrückt werden. Da gibt es kein "ist nicht definiert", da gibt es aber sehr wohl widersprüchliche Begriffe und Definitionen, die trotzdem Teil der Sprache sind. Bei vielen widersprüchlich Begriffen ist auch gar nicht sofort klar, dass sie widersprüchlich sind (bei verheirateten Junggesellen ist es natürlich sofort offensichtlich), etwa wenn es sich um komplexere Begriffe handelt, dessen Definition andere relativ komplexe Begriffe verwendet. Weil uns nicht für jeden Begriff unmittelbar eine perfekte Definition bewusst ist, fallen Widersprüche oft nicht auf. (Die Aufdeckung solcher verdeckter Widersprüche ist übrigens ein Hauptthema der Analytischen Philosophie.)
... oder zB Notwendigkeit oder Möglichkeit oder sonstwas einer solchen Existenz oder Nicht-Existenz (wovon denn jetzt genau?) darin zu suchen, ist unlogisch. - Im Gegenteil, das ist sehr logisch. "Unmöglich" ist nämlich ein Begriff aus der Modallogik, der genau für solche Fälle verwendet wird. "Es gibt verheiratete Junggesellen" ist ein Standardbeispiel einer unmöglichen Aussage. Ich glaube du verwendest den Begriff der Unmöglichkeit gemäß der Umgangssprache, wo zum Beispiel die falsche Aussage "Es gibt etwas das sich schneller bewegt als Licht" unmöglich ist. Nach der Modallogik ist diese Aussage zwar weiterhin falsch, aber sie ist hier nicht unmöglich, sondern kontingent. Ich dachte es sei aus der Ausgangsfrage ersichtlich, dass ich hier modallogische Begriffe verwende, weil ich ja Kontingenz und Notwendigkeit entsprechend verlinkt hatte. Aber vielleicht hätte ich das tatsächlich deutlicher machen sollen.
Das Wesen einer Kugel wird nicht dadurch nicht-existent - Ich bin weiter oben schon mehrmals darauf eingegangen, dass ich mit "existieren" tatsächlich physische oder physikalische Existenz meine, nicht aber Zahlen, Relationen, idealisierte geometrische Formen oder dergleichen. (Physische Kugeln aber schon.)
Da unser Universum existiert, gibt es "nie und nirgendwo ein Universum" bzw "kein Universum" nicht. - Ich sehe keine hinreichende Begründung für die Behauptung, die Existenz des Universums sei notwendig. Wie gesagt: Die Tatsache dass etwas existiert, heißt nicht, dass es notwendig existiert. Mit "Universum" meint man üblicherweise (jedenfalls ich) so viel wie "Alles was existiert". Die Menge allen Existierenden könnte aber auch leer sein; dass sie es nicht ist, ist anscheinend nur eine kontingente Tatsache. Jedenfalls wüsste ich von nichts, das nicht nur existiert sondern auch notwendig existiert. (Es gab zwar mal Versuche zu beweisen, dass die Existenz Gottes notwendig sei, diese Beweise haben sich aber recht schnell als fehlerhaft herausgestellt.)
Zur Evolutionären Erkenntnistheorie: Ich weiß sie mindestens so zu schätzen wie du, aber in Bezug auf die aktuelle Frage sehe ich nicht wie sie weiterhelfen könnte. Vgl. auch meine Erwähnung weiter oben bei Chricho.
Hä? A=B, aber B=!A ?? - Äh nein, wie kommst du darauf?
Verheiratete Junggesellen sind Dinge, deren Begriff widersprüchlich ist. - Richtig, deswegen ist ihre Existenz unmöglich (im Sinne der Modallogik).
Pegasus gibt es nicht. - Richtig, die Existenz von Pegasus ist aber nicht unmöglich, sondern kontingent. Genau wie die Existenz eines konkreten Pferdes, das durchaus existiert. Dass ein Begriff (Pegasus, Pferd) widerspruchsfrei ist lässt nicht auf dessen Existenz schließen, sondern nur darauf, dass dessen Existenz nicht unmöglich ist. Weiter oben bei Gamma bin ich auch schon mal darauf eingegangen. --Cubefox (Diskussion) 21:04, 27. Sep. 2015 (CEST)[Beantworten]
Wir reden aneinander vorbei (weiß nur nicht wo und warum genau):
a) Der Junggeselle (resp. Kreis \ Kugel) kann Subjekt sein, etwas, das zweifellos existiert. "verheiratet \ eckig" sind dann jeweils Eigenschaft, eine solche, die J'gges. \ Kugel nicht haben kann.
  Es ist aber in jedem Fall ein Junggeselle eine "Person", ein "Mensch", ein "Mann", und Kugel und Kreis jeweils ein "geometrisches Objekt". Eine Person kann nun aber nicht gleichzeitig die beiden Eigenschaften: "verheiratet", wie auch "Junggeselle" haben, ebenso, wie ein geometrisches Objekt nicht gleichzeitig "kugelrund" und "eckig" sein kann.
Das eigentlich Gemeinte, dessen Existenz zur Diskussion steht (Person; geometrisches Objekt), das durch eine `unerlaubte´ nicht definierte Kombination von Eigenschaften ausgedacht wurde \ regelwidrig zusammengebaut wurde, ist nicht Element des jeweiligen Systems ( selbstverständlich ist Sprache ein formales System! .. zwar hochkomplex und dynamisch, aber ihr Zweck, Sinn und Gemeintes zu vermitteln mittels zugrunseliegender allgemeinverbindlich definierter Begriffe, Grammatik, usw, (Duden), ist Leitfaden und verhindert, daß sprache beliebig oder widersprüchlich wird ((Deshalb ist Dadaismus Kunst und kein Dialekt)) )
b) derart das, was Du `Widersprüche´ nennst, was aber nur falsch angewandte Sprache ist, da zwei(!) Begriffe (nicht einer) - seien es nun zwei Eigenschaften oder ein Subjekt mit einer Eigenschaft, die es (definitionsgemäß!) nicht haben kann - unsinnig zusammengesetzt sind .. derart betrachtet als unmögliche (unsinnige) Kombination ist es hinfällig, nach einer `Notwendigkeit´ oder `Unmöglichkeit´ oder `Kontigenz´ von etwas schließlich überhaupt nicht Definiertem zu fragen ("Teilen durch Null" ist nicht `notwendig unmöglich´ oder `unmöglich´ - es ist schlicht ungültig, nicht definiert, nicht vorgesehen, nicht Teil des Systems).
c) Da sehe ich es auch als problematisch an, "gibt es schlicht nicht" oder "existiert einfach nicht" - (im Falle der verheirateten Junggesellen oder der eckigen Kugeln) - zu Substantivieren zu einer "Nicht-Existenz", die dann `notwendig´, `möglich´ oder `unmöglich´ oder `sonstwas´ sein kann oder soll. Ein Etwas "Nicht-Existenz" daraus zu machen, wo einfach nur gemeint ist, daß es diese Kombinationen ebensowenig gibt, wie ein Phantasiewort, mit dem gar nichts gemeint ist oder etwas, das den Naturgesetzen widerspricht (Fällt etwa ein Hammer `notwendig´ oder `unmöglich´ nicht nach oben, sondern nach unten?! - so zu fragen macht keinen Sinn, wenn ein Vorgang entweder den Naturgesetzen entsprechen kann oder nicht vorkommt). Da ist nix zum Analysieren.
d)? zurück zur Ausgangsfrage? [gemischt?] : Ich vermute immer noch, daß Du von einer Widersprüchlichkeit in einem System (Sprache, Geometrie, Zahlen \ Abstrakta) - in dem Fall dem nicht-Existieren [Substantivierung lol] der Kombination "verheiratet" + "Junggeselle" im System Sprache - auf ontologische Existenz "etwas im Universum oder von allem, was existiert" beliebig eine ganze Bibliothek voller Aspekte von "Existenz" (Sein; Fehlen von etwas Bestimmtem; wirklich sein (Realität) \ seiend; Sein an sich; unsere Fähigkeiten Seiendes wahrzunehmen, zu benennen und zu denken; Leere, wo etwas sein könnte(?); noch unentdecktes \ unbekanntes Existierendes; .. ).
Natürlich existiert Pegasus! Du meinst, "es gibt ihn nicht wirklich", aber zu "allem, was existiert" gehören Fabelwesen selbstverständlich dazu. Sie sind fiktiv zusammengebaute \ ausgedachte Entitäten, die irgendwo in unseren Hirnen Gestalt angenommen haben, als geistiges Bild gespeichert sind, Teil der Kultur geworden sind, eine Bedeutung haben, Assoziationen wachrufen. Nicht weniger existent, als zB die Modallogik selbst:
Modallogik, Fuzzylogik, Logik überhaupt ist nicht die höchste Instanz in Sachen "was existiert und was nicht existieren kann". Sie selber beruht auf Definitionen, Axiomen, Sätzen, Operatoren, Zeichen, Gemeintem, das letztlich aus unserer Sprache(!), unserer Wahrnehmung der Welt, dann der Begriffsbidung und schließlich unserem auf diesen Begriffen beruhendem Denken (das wiederum unsere Wahrnehmung der Welt mitbestimmt und die neuen Begriffe erfindet - ein Wechselspiel zwischen --> Umwelt --> ihre Wahrnehmung durch uns --> Sprache --> Denken --> Wahrnehmung der Welt - deshalb die EET) kommt im Ursprung. Deshalb wehre ich mich instinktiv dagegen, daß etwas, das es überhaupt nicht gibt, es dies "modallogisch notwendig" oder "modallogisch unmöglich" nicht tut. Außerdem beißt sich da die Katze in den eigenen Schwanz: Ich stelle fest, daß etwas oder das Universum existiert und nenne es "existieren". Ich stelle fest, daß ich mir Sachen ausdenken kann, die es nicht geben kann und nenne sie "unmöglich" oder "notwendig unmöglich" oder "notwendig nicht-existent", andere Sachen, die es geben kann, die "nicht ausgeschlossen" ( hatten wir noch gar nicht ;o]=) ) sind und nenne sie "(nicht-existent, aber) möglich". Ich stelle fest, das etwas existiert, das genausogut nicht existieren könnte (`in dieser oder einer einer anderen möglichen Welt´??? - offenbar sind nichtmal die eigenen Grundbegriffe (("notwendig, (un)möglich, kontingent, usw")) anständig definierbar!?) und nenne es "kontingent". Mit diesen Begriffen entwickle ich nun eine formale Modallogik, die `ausrechnen´ (Aussagen überprüfen) können soll, ob etwas existiert, ob es das `notwendig oder (un)möglich oder kontingent´ tut. Einer ontologischen Existenz, einem Sein jenseits individueller Eigenschaften, jenseits meiner Begriffe(!) (und jenseits meiner Wahrnehmung, meinem Denken) bin ich dann keinen Schritt näher gekommen .. eine ordentliche Modallogik müßte das "Unbekannte", das "noch nicht Entdeckte", das "völlig Andere, als wir es kennen", das "alles bisher Be- und Gekannte auf den Kopf stellende" uvm. mitberücksichtigen, gar unrestriktiv maximal- oder unendlich-wertig sein (mindestens so `wertig´, wie die Zusammenhänge in der Realität es tatsächlich sind) und imstande sein, die eigenen Grundbegriffe zu überprüfen in einem evolutionären Algoritmus .. selbst dann stünde man noch vor der Aufgabe, die Ergebnisse sinnvoll \ richtig zu erkennen undoder zu deuten und nicht das Offensichtliche zu übersehen .. [außerdem hatten Aristoteles und Carnap noch kein Handy mit Display ;o]P ]
"Notwendige Aussagen sind z. B. „Kreise sind rund“ und „Junggesellen sind unverheiratet“. " - Ja und? Das ist richtig. Aber Existenzsätze zählen offenbar grundsätzlich nicht zu den notwendigen Aussagen." - Geht nich', ein' Quantor davormachen - dann ist es eine "Existenzaussage": "Es gibt einen Kreis, der rund ist." .. Nehme an, Du wirst sagen, es muß keinen Kreis überhaupt (notwendig) geben. - Wenn es aber keine Kreise gibt, dann ist das Wort "Kreis" überflüssig, sinnlos. Dabei ist die Existenz von Kreisen als gedankloiches Modell eingeschlossen (daß es die auch nicht gäbe). Dann haben wir da stehen: "Es gibt nichts, was es nicht gibt." O,o (egal, ob rund oder verheiratet, notwendig oder möglich .. wenn es keine Kreise - auch in der Vorstellung - nicht gibt und das Wort "Kreis" so sinnlos ist, wie ein Phantasiewort für gar nichts Gemeintes, dann erübrigen sich alle weiteren Aussagen darüber).
Kann es sein, daß Du , was "existieren" angeht, zu sehr an "real existierenden `Dingen´", also `Sachen´ klebst (die tatsächlich überprüfbar real wirklich in echt existieren, wo man "notwendig, (un)möglich etc" sagen kann)? --217.84.102.98 13:56, 28. Sep. 2015 (CEST)[Beantworten]
Nochmal das mit dem Universum: ..
  • Das Universum existiert.
  • Es existiert jetzt, und, da Zeit erst in einem Universum entsteht, wenn sich ein einförmiges undifferenziertes Quantenschaum(?)-Universum zu einem solchen mit differenzierbaren unterscheidbaren Wellen(?) \ Energie(-Einheiten)(?) \ Vakuumenergie(?) entwickelt(?) \ ensteht(?) \ diese `erste´ `Mindest´-Differenzierung schon hat(?) \ schon haben muß, um existieren zu können(?), .. jedenfalls, da es keine `Hyperzeit´ gibt mit lange nichts oder `Nichts´, dann plötzlich ein Universum, sondern Zeit, irgendein Zeit-Begriff erst in einem Universum Sinn macht .. deshalb existiert es mit und in seiner Zeit, aber nicht in einer Hyperzeit irgendwann erst nicht, dann wohl, dann platzt es, wie eine Seifenblase, .. sondern, wenn es `jetzt´ existiert, existiert es `immer´ .. es existiert hyperzeitlos. Die bloße Tatsache, daß es (jetzt) existiert schließt also aus, daß es "kein Universum", "universelles Nichts" möglich wäre.
  • Damit existiert es notwendig, a) weil es da ist, und "nichts", "universelles Nichts" und "kein Universum" ausschließt, b) weil "Existenz von irgendetwas" alternativlos ist, sobald man feststellen kann, daß irgendwetwas existiert, weil dies der Beleg dafür ist, daß nicht Nichts existiert. "Existenz von irgendetwas" und "Universum" sind dann dasselbe. c) Auch die Vorstellung "jemals kein Universum" (vorher oder nachher oder irgendwann) ist zunichte. d) Sollte ein Universum in einer Ur- bzw Endknall-Singularität `zu 0-Energiebilanz´ zerplatzen können, wie eine Seifenblase einfach weg sein können, dann hätten wir ein Universum mit einer "0-Energiebilanz, das jeden Augenblick (wieder) urknallen kann".
Ich wüßte nicht, wie nachdem einmal festgestellt werden kann, daß dieses Universum existiert, .. wie dann noch "gar kein Universam" irgendwie möglich sein können soll. Wie das tatsächliche ganze allumfassende Universum im Détail ist oder nicht ist in seinen Elementen \ Zusammensetzung \ Ausdehnung \ Funktionieren \ lokal \ kausal \ Entwicklung, sind dann Akzidenzien zB den Naturgesetzen entsprechend. Es ist mein Verständnis des Begriffs "existieren ( von irgendetwas überhaupt ((erstmals oder jemals)) )", angewandt auf das Ganze, das Universum, Alles, was ist in seiner ganzen Gesamtheit. --217.84.102.98 14:52, 28. Sep. 2015 (CEST)[Beantworten]
Mit reiner Logik - so gut sie auch sein mag und sich selbst zu überprüfen imstande sein sollte - ist der Wirklichkeit des Seins, einem Begriff, wie "Existenz" nicht beizukommen, da immer unbewiesene Ausgangsannahmen gesetzt werden müssen, die sich schwerlich ohne "ist dies oder jenes" oder "soll ``..´´ bedeuten (sein)" formulieren lassen. - Deshalb sehe ich wirklich nur die Möglichkeit, sich dieser Welt der Regelmäßigkeiten für Seiendes ganzheitlich, wie Chaostheorie und Evolutionstheorie zu nähern unter der mächtigen Zuhilfenahme von Simulationen und Ausprobieren ontologischer Kalküle .. es könnten dann Erkenntnisse über Regelmäßigkeiten \ Bedingungen Voraussetzungen \ über Seiendes (Eigenschaften; Möglichkeiten; Bestandhaben; mit/ohne Raum/Zeit; mit/ohne `Gegenstück´; Raumaufteilung; Platz blockieren versus Überlappung; Arten von "seiend" (diffus, geformt, gekoppelt, verstreut, ganz & unteilig oder zusammengesetzt & kombiniert, dinglich / nicht dinglich; .. ?); raum-/zeitliche Abstände; Einfluss auf Anderes (Wirkung); .. alles jeweils verknüpfbar habend/nichthaben-könnend/brauchend/usw. .. allerlei erdenkliche Abstrakta; .. ein Game of Life auf abstrakten `Zutaten´ für Seiendes an sich? Eine Erforschung des Manifestierens \ Gestalt \ Sein annehmend von Möglichem, besierend auf Definitionen \ Axiomen von "Sein" und "seiend" ((zB 1."unterscheidbar", zB 2."hat mindestens eine Eigenschaft", zB 3."muß beständig sein")) .. ) ganz allgemein vielleicht gewonnen werden. --217.84.102.98 15:29, 28. Sep. 2015 (CEST)[Beantworten]
Eine Person kann nun aber nicht gleichzeitig die beiden Eigenschaften: "verheiratet", wie auch "Junggeselle" haben, ebenso, wie ein geometrisches Objekt nicht gleichzeitig "kugelrund" und "eckig" sein kann. - Ich weiß, es hat aber auch niemand etwas anderslautendes behauptet. Keine Ahnung warum du dich so an diesem Thema aufhängst.
selbstverständlich ist Sprache ein formales System! .. zwar hochkomplex und dynamisch - Da irrst du dich. In einem Formalen System sind alle Sätze des Systems eindeutig aus präzisen vordefinierten Axiomen und Schlussregeln ableitbar. Das ist bei natürlicher Sprache schlicht und einfach nicht der Fall. Siehe z.B. Formales System oder Natürliche Sprache. Ich kann dazu auch das Buch Gödel, Escher, Bach von Douglas R. Hofstadter empfehlen.
... und verhindert, daß sprache beliebig oder widersprüchlich wird - In natürlicher Sprache finden sich nur allzu oft Widersprüche. Dass man meist versucht diese zu vermeiden ist nicht immer erfolgreich. Auch weil man oft gar nicht bemerkt wenn man widersprüchliche Begriffe verwendet. Siehe meinen Hinweis zur Analytischen Philosophie im vorigen Beitrag.
derart das, was Du `Widersprüche´ nennst, was aber nur falsch angewandte Sprache ist - Dass "verheirateter Junggeselle" durchaus ein Widerspruch ist, und zwar nicht nur wenn es nach mir geht, kannst du zum Beispiel unter Contradictio in adiecto nachlesen.
derart betrachtet als unmögliche (unsinnige) Kombination ist es hinfällig, nach einer `Notwendigkeit´ oder `Unmöglichkeit´ oder `Kontigenz´ von etwas schließlich überhaupt nicht Definiertem zu fragen - Du kannst Widersprüche in der Sprache gerne privat "unsinnig" nennen, das ändert aber nichts daran, dass solche widersprüchlichen Sätze in der Modallogik als "unmöglich" bezeichnet werden. Ob du sie so bezeichnen magst oder nicht ist dabei irrelevant. Das hatte ich aber schon im letzten Beitrag gesagt.
"Teilen durch Null" - Hat mit der ganzen Angelegenheit nichts zu tun.
Da sehe ich es auch als problematisch an, "gibt es schlicht nicht" oder "existiert einfach nicht" ... Ein Etwas "Nicht-Existenz" daraus zu machen, wo einfach nur gemeint ist, daß es diese Kombinationen ebensowenig gibt - Ob man nun "Nicht-Existenz" sagst oder "gibt es nicht" oder "existiert nicht" ist gleichgültig. Eine Substantivierung ist eine rein grammatische Operation die nichts an der Bedeutung ändert, insbesondere postuliert sie nicht, wie du unterstellst, plötzlich ein "Etwas" wo vorher keines gemeint war.
wie ein Phantasiewort, mit dem gar nichts gemeint ist - Einem Phantasiewort entspricht gar keinen Begriffen, es gibt daher auch keinen Widerspruch. Dass beidem auch kein reales Ding entsprechen kann stimmt natürlich.
Fällt etwa ein Hammer `notwendig´ oder `unmöglich´ nicht nach oben, sondern nach unten?! - so zu fragen macht keinen Sinn, wenn ein Vorgang entweder den Naturgesetzen entsprechen kann oder nicht vorkommt - Naturgesetze sind selbst kontingent. D.h. sie sind so wie sie sind, aber sie könnten logisch betrachtet auch anders sein. Es ist zum Beispiel ein Universum denkbar in dem es keine Gravitation gibt, oder eines in dem sie nicht mit der zweiten Potenz der Entfernung abnimmt sondern mit der dritten, oder eines in dem die Lichtgeschwindigkeit kaum schneller als der Schall ist (Terry Pratchetts Scheibenwelt lässt grüßen) etc. pp. Ein Universum mit sehr einfachem Naturgesetzen wäre zum Beispiel ein zellulärer Automat, etwa das auch von dir erwähnte Game of Life.
Natürlich existiert Pegasus! Du meinst, "es gibt ihn nicht wirklich", aber zu "allem, was existiert" gehören Fabelwesen selbstverständlich dazu. - Wie ich in den anderen Beiträgen weiter oben mehrmals geschrieben hatte, meinte ich in der Ursprungsfrage "existieren" im physischen/physikalischen Sinne, was wohl dem entspricht das du hier mit "wirklich geben" meinst. Dass der Begriff von Pegasus "in der Vorstellung" (oder so) existiert, heißt nicht, dass Pegasus selbst existiert. Das ist der Unterschied zwischen Begriff und Ding, denen zwei verschiedene Ecken im Semiotischen Dreieck entsprechen. Weiter oben bei Chricho sind wir näher darauf eingegangen.
Logik ... ist nicht die höchste Instanz ... - Natürlich ist die Logik (wie die Mathematik) in gewissem Sinne "nur" eine Sprache die der Mensch erdacht/erfunden hat. Gäbe es keine Menschen, gäbe es diese "Sprachen" nicht. Trotzdem sind sie auf beliebige denkbare Welten anwendbar, egal ob sie nun real sind oder nicht. Bzw. sie wären anwendbar, egal ob es ob es uns gäbe oder nicht. Anders gesagt: Ein Stein und eine Pflanze waren auch nicht das gleiche als noch keine Menschen gab, die das entsprechend erkennen und benennen konnten. Und es ist zwar eine Welt möglich (genauer: kontingent) in der die Lichtgeschwindigkeit doppelt so hoch ist wie bei uns, aber keine, in der sie sowohl doppelt als auch halb so hoch ist. Die Evolutionäre Erkenntnistheorie besagt ja gerade, dass es die Welt eben nicht nur gibt, weil wir sie wahrnehmen, sondern dass unsere Wahrnehmung eine Folge davon ist, dass es bereits eine Welt gab, in der sich erst etwas entwickelte, das diese Welt wahrnahm. Die prinzipielle Gültigkeit der Logik ist nicht davon abhängig, dass es jemand gibt, der die Logik "anwenden" kann.
Ich stelle fest, daß ich mir Sachen ausdenken kann, die es nicht geben kann und nenne sie "unmöglich" oder "notwendig unmöglich" - Letzteres sagt keiner der sich mit den Begrifflichkeiten auskennt, das "notwendig" ist hier redundant.
andere Sachen, die es geben kann, die "nicht ausgeschlossen" ( hatten wir noch gar nicht ;o]=) ) sind und nenne sie "(nicht-existent, aber) möglich" - Die nennt man nur "möglich", ob sie existieren oder nicht ist dabei nicht festgelegt.
Ich stelle fest, das etwas existiert, das genausogut nicht existieren könnte (`in dieser oder einer einer anderen möglichen Welt´??? - In mindestens einer möglichen Welt, egal ob in der tatsächlichen oder nicht. Wobei man Leibnizens Rede von den "möglichen Welten" für die Definition der modallogischen Begriffe nicht mehr braucht. Kontingent ist etwa das, was weder begriffsanalytisch wahr (tautologisch, =notwendig) noch widersprüchlich (=unmöglich) ist.
offenbar sind nichtmal die eigenen Grundbegriffe (("notwendig, (un)möglich, kontingent, usw")) anständig definierbar!? - Die sind anständig definiert, so weit ich sehe scheinst du nur die Definitionen nicht besonders gut zu kennen.
Einer ontologischen Existenz, einem Sein jenseits individueller Eigenschaften, jenseits meiner Begriffe(!) (und jenseits meiner Wahrnehmung, meinem Denken) bin ich dann keinen Schritt näher gekommen - So so. Du denkst also, die Modallogik würde ohne dich und deine Begriffe plötzlich ungültig werden. Gäbe es kein denkendes Wesen, könnte es also plötzlich eckige Kreise geben und so. Nein, eben nicht.
das "noch nicht Entdeckte", das "völlig Andere, als wir es kennen", das "alles bisher Be- und Gekannte auf den Kopf stellende" uvm. mitberücksichtigen - Anscheinend postulierst du einfach, dass man irgendetwas entdeckt, auf das die Modallogik nicht anwendbar sei (natürlich ohne das erklären zu können), und schließt daraus, dass die Modallogik unzureichend sei. Das nennt man eine Petitio principii.
Nehme an, Du wirst sagen, es muß keinen Kreis überhaupt (notwendig) geben. - Wenn es aber keine Kreise gibt, dann ist das Wort "Kreis" überflüssig, sinnlos. - Es gibt in der realen Welt auch keine 15-dimensionalen Hyperwürfel und keine Osterhasen, das heißt aber nicht dass die Begriffe irgendwie "sinnlos" wären. Diesen Begriffen entsprechen einfach keine realen Dinge.
Dabei ist die Existenz von Kreisen als gedankloiches Modell eingeschlossen (daß es die auch nicht gäbe). - Wie bereits gesagt, die Existenz von Begriffen ("gedankliches Modell") ist etwas anderes als die physische Existenz von dem damit Bezeichneten. Letzteres meinte ich in der Ausgangsfrage. Darüber hatten ich und andere Leute weiter oben übrigens schon gesprochen.
Kann es sein, daß Du , was "existieren" angeht, zu sehr an "real existierenden `Dingen´", also `Sachen´ klebst (die tatsächlich überprüfbar real wirklich in echt existieren, wo man "notwendig, (un)möglich etc" sagen kann)? - Ich "klebe" nicht daran, ich "meine" einfach reale Dinge. (Was übrigens nicht gerade unüblich ist. Wenn jemand sagt "Der Weihnachtsmann existiert nicht", dann meint er wohl kaum, dass er keinen Begriff davon hätte. Und wenn ein Wissenschaftler sagt "Es gibt wahrscheinlich flüssiges Wasser auf dem Mars" dann meint er wohl kaum "... auf dem Mars in meiner Phantasie.")
jedenfalls, da es keine `Hyperzeit´ gibt mit lange nichts oder `Nichts´, dann plötzlich ein Universum, sondern Zeit, irgendein Zeit-Begriff erst in einem Universum Sinn macht - Dass die Existenz des Universums, inklusive seiner Raum- und Zeitdimensionen kontingent ist, erfordert keine "Hyperzeit". Die wäre nur erforderlich damit man "vor" und "nach" dem Universum definieren kann, aber darum geht es ja gar nicht. (Außerdem ist mit "Universum" ohnehin "alles, was ist" gemeint, es würde also das "davor" und "danach" inkl. der "Hyperzeit" miteinschließen.) Zeit ist nicht weniger kontingent als Raum. Ein Universum mit 100 Raumdimensionen ist genauso kontingent wie eines ohne Zeitdimensionen.
Ich wüßte nicht, wie nachdem einmal festgestellt werden kann, daß dieses Universum existiert, .. wie dann noch "gar kein Universam" irgendwie möglich sein können soll. - Das ist genauso möglich kontingent wie die Tatsache, dass die Welt zwar so ist, wie sie ist, dass sie logisch betrachtet aber auch anders sein könnte. Die Schwerkraft könnte stärker sein. Oder schwächer. Es könnte sie auch gar nicht geben. Genau wie es die Schwerkraft nicht geben könnte, könnte es auch alles andere nicht geben. Das Universum ist die Menge von allem was es gibt. Diese Menge könnte größer, kleiner und offenbar auch ganz leer sein. Letzteres hieße so viel wie "nichts existiert" oder "das Universum existiert nicht".
Mit reiner Logik - so gut sie auch sein mag und sich selbst zu überprüfen imstande sein sollte - ist der Wirklichkeit des Seins, einem Begriff, wie "Existenz" nicht beizukommen - Ich denke meiner Frage ist nur mit Begriffsanalyse und Logik beizukommen. Man könnte entweder beweisen, dass es A) tatsächlich etwas gibt, das notwendig existiert (z.B. Gott, Urknall etc.), dann wäre meine Frage hinfällig. Oder B) man beweist, dass es notwendig nichts gibt das notwendig existiert, was die Ausgangsfrage beantworten würde. Ich halte es für sehr viel wahrscheinlicher dass eine Antwort nach dem Muster B) machbar ist, deswegen hatte ich die Ausgangsfrage auch darauf ausgerichtet. --Cubefox (Diskussion) 23:45, 28. Sep. 2015 (CEST)[Beantworten]
[ad hoc - Antwort noch nicht gelesen] -- EN:Modal_logic nennt in der Einleitung weitere Kriterien (andere als, Wahrheitskriterien "möglich, notwendig, unmöglich") zB "man nimmt an" (doxastisch), "es ist bekannt \ man weiß" [epistemisch), "es ist obligatorisch" (wohl mehr "Voraussetzung", als "notwendig", deontisch) und "war ~", "ist immer ~", "wird ~", "wird immer der Fall sein" (temporal). Mir fällt auf, daß in allen Formulierungen das "ist" offenbar nicht genauer definiert werden mußte, es als bekannt vorausgesetzt wird. Fragen bzw Aussagen, die modallogisch auf jedwed modulierte Existenz ("möglich, notwendig, usw") geprüft werden sollen geraten so zwingend in Konflikt mit den dem modallogischen System eigenen Axiomen \ Definitionen ihrer Formalen Sprache (Operatoren; Doppelpunkte; der `Bedeutung´ ihrer Zeichen, die eigene Semantik) in Konflikt .. oder schafft die Modallogik es zB ihre eigneen Voraussetzungen zu prüfen: "Der Doppelpunkt bedeutet (notwendig, möglicherweise, unmöglich [dies oder jenes].", "Eine Aussage links vom Pfeil ist wahr, wenn rechts .." ?   |   Auch die Definitionen um `allerlei mögliche Welten´ erscheinen mir ziemlich abenteuerlich und beliebig für ein System, daß Wahrheit, Realität, Gültigkeit erst überprüfen können soll .. [gez. 217.84.° = Benutzer:RoNeunzig ] --217.84.101.163 14:16, 29. Sep. 2015 (CEST)[Beantworten]
Mal ein Versuch ohne Existenzialismus und Quantenphysik: Bei den "verheirateten Junggesellen" hat man eine "Binnen-Bedingung" für die Nicht-Existenz formuliert, in der Art "Ein Objekt, das gleichzeitig die (sich ausschließenden) Eigenschaften A und B hat, existiert nicht." Wie es um die Existenz von A oder B selbst steht, ist im Prinzip egal. Wenn man aber beweisen will, das ein Objekt existiert, reicht so eine einfache Bedingung nicht aus. Zum einen muss man das Objekt, um das es geht, genau definieren und von anderen abgrenzen. (Was genau ist denn "der Weihnachtsmann": der Weihnachtsmanndarsteller aus dem Kaufhaus; die Schokoladenfigur im Regal; das Ding, das vom Balkon meines Nachbarn runterhängt; der Typ, der angeblich am Nordpol wohnt...) Zum anderen läuft das wohl auf Cogito ergo sum hinaus, um eine feste Grundlage zu haben, von der aus man alle anderen Sachen definieren kann. - Für die Definition der Nicht-Existenz mussten wir die Objekte also nicht kennen, für die Definition der Existenz müssen wir eigentlich alle Objekte kennen. Dann können wir alle Nicht-Weihnachtsmänner aussortieren und sehen ob der Weihnachtsmann übrig bleibt. --Optimum (Diskussion) 19:43, 29. Sep. 2015 (CEST)[Beantworten]
@Cubefox: Touché. ( Habe mich nie erschöpfend mit Logik \ Modalogik beschäftigt, nicht mehr freiwillig seit ich erkannte, daß selbst Logik auf unüberprüfbaren ersten Vorausannahmen ((Axiomensatz, Definitionen, Schlußregeln, usw)) gründet und keine Wahrheit enthalten kann, die über sie selbst hinausgeht. )
@thread & Ausgangsfrage:   Also, etwas das modallogisch notwendig existiert, müßte unmöglich nicht existieren. Vielleicht die Naturgesetze? Ist ein seiendes Universum denkbar, das nicht funktioniert \ obwohl es gar nicht funktionieren kann, weil es keine Gesetzmäßigkeiten gibt, nichtmal Zufall oder Fluktuation, irgendwas, die seine Existenz in irgendeiner Form `regeln´ - und nichtmal sogenanntes `Chaos´ (das ontologisch eh fraglich ist, da `Unordnung´ bestenfalls verbindlich mit "Entropie" untersucht werden kann und `Vorhersagbarkeit´ ((durch wahrnehmende Subjekte)) kein Kriterium ist) .. nichtmal Chaos ist frei von ``Inseln der Ordnung´´, wäre also auch in einem U. mit chaotischen Naturgesetzen wären es welche. - Es könnte dann höchstens noch die rein theoretische Möglichkeit bestehen (aber um solche Unwägbarkeiten scheint es in der Modallogik ja zu gehen), daß zwar kein Universum existiert, aber `in allen möglichen Welten´ die Naturgesetze auch ohne, daß ein U. existiert meta-gültig wären für falls ein U. entstehen sollte. Oder es gibt kein U. und keine Naturgesetze, womit sie nicht notwendig exitieren würden, wenn es kein U. gibt. - Muß nicht etwas, das existiert mindestens eine (vielleicht mindestens eine komplementär zweite?) Eigenschaft haben? Muß es nicht Bestand haben, um von ihm zu sagen, es existiert? Also: "Es existiert notwendig mindestens eine Eigenschaft, da ein Universum (unseres) existiert." ?
Ansonsten lassen sich sicher in der Modallogik Aussagen konstruieren, in denen eine bestehende Voraussetzung zwingend die Existenz von Anderem nach sich zieht: "Wenn es die Erde gibt, dann existiert (notwendig auch) ihre Schwerkraft." ("Zu einer Masse gibt es unmöglich keine Schwerkraft.")
@Optimum: .. und cogito führt mich wiederum zur EET, die von allen PHilosophien und ~Ismen am besten zu erklären imstande ist, wo dieses cogito seine Ursprünge hat und demnach einzuschätzen ist in seinen Fähigkeiten, Welt wie zutreffend zu erkennen. --217.84.93.123 13:16, 30. Sep. 2015 (CEST)[Beantworten]
[s.a. derzeit 6. post von oben, "was außer Begriffe"] --217.84.93.123 14:11, 30. Sep. 2015 (CEST)[Beantworten]

Liebe Philosophierer, ihr seid jetzt hier seit bald zwei Wochen am Werke und inzwischen nimmt der Abschnitt ein Viertel der Auskunft ein (114 KB von 444 KB). Könnten wir das auf eine Benutzer-Unterseite verlagern? Benutzer:Cubefox? --Eike (Diskussion) 14:04, 30. Sep. 2015 (CEST)[Beantworten]

Wurde Ausgangsfrage denn beantwortet? Sonst bin ich dagegen, den Besuchern der Auskunft diese schwierige Ausnahmefrage vorzuenthalten. --217.84.93.123 14:14, 30. Sep. 2015 (CEST) Vielleicht kann jemand darin Versiertes, Teilthreads zusammenklappen und oder über-Untertiteln? --217.84.93.123 14:18, 30. Sep. 2015 (CEST)[Beantworten]

24. September 2015

Ist Betrug in Deutschland kein Offizialdelikt?

Oder warum wurde gegen Martin Winterkorn kein Strafverfahren von Amtes wegen eröffnet?--Muroshi (Diskussion) 13:30, 24. Sep. 2015 (CEST)[Beantworten]

Straftaten im Ausland müssen nicht verfolgt werden, § 153c StPO -- Liliana 13:31, 24. Sep. 2015 (CEST)[Beantworten]
Die Autos gab's auch hier. --Eike (Diskussion) 13:34, 24. Sep. 2015 (CEST)[Beantworten]
In Deutschland ist es aber nicht verboten. -- Liliana 13:37, 24. Sep. 2015 (CEST)[Beantworten]
Betrug ist in Deutschland nicht verboten? --Eike (Diskussion) 14:40, 24. Sep. 2015 (CEST)[Beantworten]
Es wurden Anzeigen erstattet, es besteht also eh keine Notwendigkeit, von Amts wegen etwas anzuleiern. --Eike (Diskussion) 13:36, 24. Sep. 2015 (CEST)[Beantworten]
Siehe: https://de.wikipedia.org/wiki/Offizialdelikt_%28Deutschland%29 : Da wird Betrug explizit im Lemma aufgeführt.--Muroshi (Diskussion) 13:59, 24. Sep. 2015 (CEST)[Beantworten]

Auch wenn es die Medien heute so darstellen ist es doch keineswegs so, daß ein Vorstandsvorsitzender für jede Entscheidung in seinem Unternehmen strafrechtlich verantwortlich ist. Allein der Vorstand der Volkswagen AG besteht aus 10 Mitgliedern. Dazu kommen diverse Manager der Zwischenebenen, Projektleiter und wohl auch Verantwortliche in der USA. Die moralische Verantwortung rechtfertigt da noch lange keine staatliche Ermittlungen, und auch die Anzeigen gegen unbekannt dürften nur eingeschränkt wirksam werden, wenn wie bei anderen Fällen auch die vermeintlich Geschädigte und Täterin mit der VW AG identisch ist. Übrigens sehen auch nicht alle Experten die Situation von VW so schwarz. Denn auch was umgangssprachlich als Betrug bezeichnet wird, ist noch lange nicht als solcher juristisch zu betrachten. Denn über den "Laborcharakter" der Verbrauchsangaben werden sowohl die Aktionäre als auch Kunden ausführlich informiert. Genauso ist es bekannt, daß sich die Motoren individuell per Sensoren auf äußere Einflüsse einstellen. Es dürfte nicht nur 1 Situation programmiert worden sein, auf den der Motor mit Verhalten X reagiert, also auch nicht automatisch unrichtige Angaben.Oliver S.Y. (Diskussion) 13:48, 24. Sep. 2015 (CEST)[Beantworten]

Außerdem ist in Deutschland die Manipulation von Emissionswerten bei Tests rechtlich zulässig. --194.113.41.2 13:53, 24. Sep. 2015 (CEST)[Beantworten]
(BK) Ähm, bitte Betrug (Deutschland) lesen. Betrug ist ein Vermögensdelikt und der materielle Schaden durch einen Autokauf bei VW ist (im Gegensatz zu den Strafsummen, die in den USA zu zahlen sind) wohl kaum bezifferbar. Die offizielle Lesart ist außerdem, Winterkorn habe von allem nichts gewußt. Auch das müßte erstmal widerlegt sein, vor allem wenn man ihn hier auf der Auskunft als Schuldigen vorführt. Bitte auch Offizialdelikt (Deutschland) lesen. Betrug ist ab einem bestimmten Umfang des Schadens kein relatives Antragsdelikt mehr (vgl. Strafantrag (Deutschland)). Die Entscheidung, "von Amts wegen etwas anzuleiern" oder eben nichts hängt also nicht von bereits gestellten Anzeigen (deren Zielrichtung ich nicht kenne) ab sondern die Staatsanwaltschaft muß - falls es sich überhaupt um Betrug handelt - auf jeden Fall ermitteln (Offizialdelikt). Richtig ist natürlich, dass mit einer Anzeige bereits ein Vorgang besteht und dann weitere Anzeigen und die Initiative von Staatsanwaltschaften zusammengefaßt werden. --91.44.83.12 13:55, 24. Sep. 2015 (CEST)[Beantworten]
Wir reden hier offenbar von verschiedenen Vorfällen. Die Käufe, um die es bisher geht sind in der USA vorgenommen worden. Und das nicht zwischen der Volkswagen AG und Kunde XYZ, sondern einem Vertriebspartner mit Rechtsstand USA. Der "Betrug" um den es hier geht ist die Anweisung, Organisation, Umsetzung und ggf. das Verbergen dieses Programmteils. Auch da ergibt sich nicht automatisch ein Bezug zur Volkswagen AG. Bestenfalls kann hier eine Staatsanwaltschaft Vorermittlungen aufnehmen, aber auch das nicht aufgrund von Zeitungsmeldungen gegen den Willen der Geschädigten. Es gibt aktuelle innerhalb des Konzerns 600 verschiedene IT-Projekte, dazu die Bereiche IT-Steuerung und IT-Standards. Dazu kommt das Problem, daß 2 Modellreihen betroffen sind, die auch eigenverantwortlich parallel wirken. Darum fiel mir eigentlich als erstes der § 129 StGB ein - Bildung einer kriminellen Vereinigung, und nicht Betrug durch Herrn Winterkorn.Oliver S.Y. (Diskussion) 14:28, 24. Sep. 2015 (CEST)[Beantworten]
Es ist schon mal zumindest Europa: http://www.tagesschau.de/wirtschaft/regierung-vw-103.html Mit "hier" war ich oben zu voreilig, aber es würde mich wundern, wenn nicht.--Eike (Diskussion) 14:39, 24. Sep. 2015 (CEST)[Beantworten]
Große Überraschung: http://www.n-tv.de/newsletter/breakingnews/2-8-Millionen-Volkswagen-in-Deutschland-manipuliert-article16012786.html --Eike (Diskussion) 15:15, 25. Sep. 2015 (CEST)[Beantworten]
Ich schätze, die Sammelklagen in den USA richten sich nicht gegen dortige Vertriebspartner. IT-Projekte zur Absatzsteigerung betreffen in der Tat Volkswagen, näher deren Management, aber betreib ruhig weiter Verbrämung.--Muroshi (Diskussion) 14:58, 24. Sep. 2015 (CEST)[Beantworten]
Bisher habe ich den Eindruck, dass du hier eher tendenziell tribunalartige Verhältnisse inszenieren möchtest statt einfach nur unaufgeregt eine Sachfrage zu untersuchen und zu klären. --91.44.83.12 15:15, 24. Sep. 2015 (CEST) [Beantworten]
Ich stelle einfach fest, dass du die Fakten bewusst schön redest und nicht besonders objektive Antworten gibst (ob für die entsprechende PR-Abteilung oder aus Markenliebe lasse ich jetzt einmal offen): http://www.zeit.de/wirtschaft/2015-09/abgas-manipulation-europa-volkswagen-bmw --Muroshi (Diskussion) 15:48, 24. Sep. 2015 (CEST)[Beantworten]
Aha. Nun, es können hier ja alle lesen, was ich geschrieben und wie ich argumentiert habe und sich dann selbst eine Meinung bilden, was von deiner "Feststellung" zu halten ist. --91.44.83.12 15:57, 24. Sep. 2015 (CEST) [Beantworten]
Hm. An den § 129 StGB kann man sicherlich oft denken. Aber was ist denn nun ganz konkret die Straftat? Und wie könnte das zusammenpassen mit § 129 Abs. 1 Nr. 2 StGB („wenn die Begehung von Straftaten nur ein Zweck oder eine Tätigkeit von untergeordneter Bedeutung ist“). Die Frage der betriebsinternen Verantwortlichkeiten ist ja noch nicht geklärt. Die Organisation und Umsetzung des fraglichen Programmteils wäre nur strafbar, wenn die damit verbundene Absicht einer Täuschung bekannt und als Handlungsziel geteilt war. Das halte ich für undenkbar und auch für nicht nachweisbar. Es geht aus meiner Sicht vor allem um die grundsätzliche Diskussion, Zielbestimmung und Planung des Vorgangs und die daraus folgenden Anweisungen. Einem ausführenden Programmierer oder Ingenieur mag es sogar sinnvoll erscheinen, für den Alltagsbetrieb und den Laborbetrieb unterschiedliche Werte zu ermitteln. Teile des Programms wären also für sich gesehen unverdächtig. Mir ist auch nicht klar: Nach welcher Vorschrift in der StPO muß die Staatsanwaltschaft angeblich bei Vorermittlungen auf den Willen der Geschädigten Rücksicht nehmen? Das kann ich nicht nachvollziehen. --91.44.83.12 15:11, 24. Sep. 2015 (CEST)[Beantworten]
Betrug ist ein Offizialdelikt, und ob ein Strafverfahren eröffnet wird, ist noch nicht entscheiden. <= Ist damit die Frage vollständig und korrekt beantwortet? --Eike (Diskussion) 15:19, 24. Sep. 2015 (CEST)[Beantworten]
Betrug ist ein Offizialdelikt, richtig. Aber für einen Betrug ist es gem. § 263 StGB erforderlich, dass (objektiv) eine Täuschungshandlung, ein entsprechender Irrtum und dadurch ausgelöst eine (selbstschädigende) Vermögensverfügung des Getäuschten sowie (subjektiv) eine Absicht des Täters vorliegt, sich durch diese Vermögensverfügung zu bereichern. Selbst wenn man unterstellt, dass Vorstandsmitglieder von den Manipulationen wussten, kann man nicht annehmen, dass sie selbst täuschten oder als "Garanten" zur Aufklärung der Kunden verpflichtet waren. Außerdem kommt das Geld, das die Kunden zahlen, nicht unmittelbar dem Vorstand zugute. Ich halte deshalb einen Betrug für offensichtlich nicht gegeben. Es würde mich allerdings nicht wundern, wenn eine Staatsanwaltschaft - wie schon öfters geschehen - den Straftatbestand der Untreue kreativ interpretiert und Anklage erhebt, weil der Täter dem VW-Konzern Schaden zugefügt hat. --Zerolevel (Diskussion) 15:36, 24. Sep. 2015 (CEST) PS zu Betrug: bereichert könnte auch der VW-Konzern als "Dritter" sein, aber die Absicht (mehr als Kenntnis) der rechtswidrigen Bereicherung müsste nachgewiesen werden. --Zerolevel (Diskussion) 15:50, 24. Sep. 2015 (CEST)[Beantworten]
+1 - In Strafverfahrensrecht (Deutschland) sind die Stufen eines Strafverfahrens dargestellt. Das Präsidium des VW-Ausichtsrats hat beschlossen, Strafanzeige zu erstatten. Es wird also mit aller Wahrscheinlichkeit ein Ermittlungsverfahren eröffnet und damit ist die erste Unterstellung in der Frage geklärt: Es wurde kein Strafverfahren von Amtes wegen eröffnet, weil es bereits oder absehbar besteht oder aber die Staatsanwaltschaft sich einfach noch nicht entschieden hatte. Ob die Staatsanwalt von sich aus die Initiative ergriffen hätte läßt sich damit nicht mehr klären. Die ungeklärte Kernfrage ist, um welche Straftaten es eigentlich geht. Das bleibt abzuwarten, bis VW die Strafanzeige tatsächlich gstellt hat und der Inhalt bekannt ist. Betrug kommt, so weit ich das sehe, als Straftat nicht in Betracht. Die zweite Unterstellung in der Frage, ein Strafverfahren müsse sich gegen Winterkorn richten, hat ebenfalls keine ausreichende Grundlage. Die Anzeige von VW richtet sich absehbar selbstverständich gegen unbekannt. --91.44.83.12 15:45, 24. Sep. 2015 (CEST)[Beantworten]
Ob die nun von vielen Unternehmensethikern kritisierte Anreizstruktur von Boni hier einer Bereicherungsabsicht nicht in die Hände spielen könnte, scheint mir gemäss Ausführung etwas fraglich. Aber selbst wenn das Verhalten nicht dahingehend verstanden wird, dass selbst getäuscht wurde, gäbe es wohl auch noch Strafbarkeit wegen Unterlassen, nicht? --Muroshi (Diskussion) 16:08, 24. Sep. 2015 (CEST)[Beantworten]
Hm. Was meinst du mit „Strafbarkeit wegen Unterlassen“? Zur Auswahl stehen die §§ 8 und 9 StGB und natürlich § 13 Abs. 1 StGB („Wer es unterläßt, einen Erfolg abzuwenden, der zum Tatbestand eines Strafgesetzes gehört, ist nach diesem Gesetz nur dann strafbar, wenn er rechtlich dafür einzustehen hat, daß der Erfolg nicht eintritt, und wenn das Unterlassen der Verwirklichung des gesetzlichen Tatbestandes durch ein Tun entspricht.“) Auch hier wäre es hilfreich, die mutmaßliche Straftat endlich mal zu benennen. Der oben verlinkte Tagesspiegel meldet über das Präsidium des VW-Ausichtsrats ja nur sehr allgemein: „Das Gremium habe "den Eindruck, dass strafrechtlich relevante Handlungen eine Rolle" gespielt hätten.“ --91.44.83.12 16:33, 24. Sep. 2015 (CEST) --91.44.83.12 16:33, 24. Sep. 2015 (CEST)[Beantworten]
Ich bin kein Jurist. In der Schweiz ist der jeweilige Tatbestand in vielen Fällen grundsätzlich als Unterlassungsdelikt strafbar, hier wird das Beispiel mit dem CEO dann auch explizit erwähnt. (Vielleicht ist das Deutsche Recht hier grosszügiger): http://studunilu.ch/wp-content/uploads/faju/zusammenfassungen/bachelor/1_jahr/stgb_ii.pdf (S.4)--Muroshi (Diskussion) 18:06, 24. Sep. 2015 (CEST)[Beantworten]
Es ist doch wohl so, dass die Software im Steuergerät des Motors bei stillstehenden Lenkrad den Abgasfilter zuschaltet und wenn das Lenkrad bewegt wird - das Auto also fährt - der Abgasfilter für höhere Leistung abgeschaltet wird. Solche Ideen werden nicht in einem Vorstand geboren und schon gar nicht im großen Kreis. Das hat eine überschaubare Ingenieursebene aus vielleicht sogar nur 2 Mann+ in der Motorentwicklung ausgeheckt und damit auch die Gesamtfirma getäuscht. Zweites Indiz ist, dass die Verkäufer- und Managementebene seit Anfang 2014 von Vorwürfen der Amerikanischen Behörden weiß, selbst ahnungslos über Umfang und mögliche Wirkungen das abgewimmelt hat. Ich mag nicht glauben, dass Winterkorn und Co. derart kriminell waren und nicht bestimmend reagiert hätten, wenn sie gewusst hätten was da läuft. Die Firma ist den Kunden natürlich für den Schaden ersatzpflichtig, für die Strafe muss erst der Schuldige gefunden werden. VW hat richtigerweise Strafanzeige gegen Unbekannt gestellt.--2003:68:ED13:4900:B0DC:8089:944A:5061 19:16, 24. Sep. 2015 (CEST)[Beantworten]
@Muroshi: Im deutschen Recht ist es so, dass es einerseits "echte Unterlassendelikte" gibt, die ausdrücklich so formuliert sind (zB unterlassene Hilfeleistung), andererseits "unechte Unterlassensdelikte", bei denen ein verbotener Erfolg durch Unterlassen eintritt (z.B. jemanden verhungern lassen). Bei diesen "unechten" Unterlassensdelikten macht man sich nur strafbar, wenn man verpflichtet ist, diesen Erfolg zu verhindern (sogenannte "Garantenstellung"), z.B. als Elternteil gegenüber den eigenen Kindern, als Krankenpfleger gegenüber den Patienten etc. Diese Verpflichtung kann sich aus dem Gesetz, aus beruflicher Stellung, Vertrag oder aus "vorangegangenem Tun" ergeben: Wenn ich jemanden einsperre und ihn dort verhungern lasse, habe ich nicht nur eine Freiheitsberaubung, sondern auch ein Tötungsdelikt begangen. -- Zerolevel (Diskussion) 11:33, 25. Sep. 2015 (CEST)[Beantworten]
Okay. Aber dann kann ich dann bei allfälligem Mitwissen über gesundheitsschädigenden Stickstoffdioxidausstoss eben keinen strukturellen Unterschied zum präsentierten Fall feststellen. Das wird letztlich die Strafrechtsuntersuchung etwas ergründen (es ist ja keine exakte Wissenschaft, falls man der Juristerei diesen Status mit viel gutem Willen überhaupt zubilligen möchte). Die obige 2 Mann-These ist bei einem hierarchisch geführten Wirtschaftsunternehmen, das sich Märkte mit Strategien erschliesst, jedenfalls grobe Schönfärberei. Ingenieure hecken aus, was das Management vorgibt oder das Management segnet es zumindest willentlich ab, insbesondere bei 11 Mio. betroffenen Fahrzeugen.--Muroshi (Diskussion) 12:55, 25. Sep. 2015 (CEST) [Beantworten]
Ich habe den Eindruck, dass du Strafrecht und den Aspekt der moralischen Verwerflichkeit nicht klar trennst. Deine Frage oben zielt auf die juristische Situation ab. Das - wie du es formulierst - „allfällige Mitwissen über gesundheitsschädigenden Stickstoffdioxidausstoss“ hat so allgemein gesehen überhaupt keine Relevanz. Die Fragen sind aus meiner Sicht: Wodurch ist wem ein materieller Schaden entstanden und welche geltenden Gesetze wurden verletzt? Konkret: Wurden durch die Manipulation deutsche Vorschriften über zulässige Abgaswerte unterlaufen? Ist Käufern, die ein Auto unter vorrangiger Berücksichtigung der angeblichen Abgaswerte erworben haben, durch die Täuschung ein materieller Schaden entstanden? Betriebsintern steht darüber hinaus die Frage im Raum, wer konkret womit den Konzern (und die Aktieninhaber) geschädigt hat. --91.44.83.12 13:48, 25. Sep. 2015 (CEST) [Beantworten]
Naja, gerade das Strafrecht strebt aber doch danach, die moralischen Normen abzubilden. Wenn das gegenwärtige Recht keine Handhabe für solche Verfehlungen bietet, muss es angepasst werden. Ich denke "Mitwissen über gesundheitsschädigenden Stickstoffdioxidausstoss“ könnte evtl. Relevanz haben. Zumindest statistisch lässt sich da ein materieller Schaden wegen Atemwegerkrankungen relativ gut belegen.--Muroshi (Diskussion) 17:54, 25. Sep. 2015 (CEST)[Beantworten]
Ja, der user M... ist im positiven Sinne sehr schriftgewaltig, aber er verallgemeinert und wirft vieles durcheinander. Vorstände und Industrieführer muss man wahrlich auch nicht lieben, sie bekommen viel Geld und leben von der Leistung der anderen im Betrieb. Kapitalwirtschaftlich sind sie den Geldgebern und Eigentümern auch für vieles verantwortlich, besonders der Geldvermehrung, da sitzen sie auch auf einem Schleuderstuhl, wenn was schief läuft. Vor der Justiz haben sie Anspruch auf Gleichbehandlung wie Otto Normalverbraucher. Da ist es nun mal so, um ein verständliches Beispiel zu nennen, dass der verursachende Lokführer bei einem Unfall bestraft wird und nicht der Herr Grube. So ist das auch bei Winterkorn und die irgendwo im Betrieb entstandene geheime Schummelsoftware. Das haben nur ganz wenige gewusst, sonst wäre das schon lange in den 2 Jahren an die Öffentlichkeit gekommen, wenigstens als Gerücht oder Hinweise aus der Konkurrenz. Und man hätte dann sicher erschreckt reagiert, wenn man gewusst hätte, was da besonders im Wirtschaftsstrafwütigen USA gemogelt wird. --2003:68:ED12:500:A0CF:7019:7DE9:678 14:52, 25. Sep. 2015 (CEST)[Beantworten]
Leute, ihr diskutiert hier alle eigentlich nur um die einzig und zentral relevante Tatsache herum, daß Betrug bei Abgasemission in Deutschland per Sondergesetz absolut legal ist: [1] Die auch schon weiter oben angesprochen wurde. Also Pustekuchen wegen jeglicher strafrechtlicher Untersuchungen. --2003:48:2E4C:B115:459:94B6:930D:8ECE 04:24, 26. Sep. 2015 (CEST)[Beantworten]
Öhhhmm - Du hast bemerkt, dass es im ZEIT-Artikel nur um Schall-Emissionen geht und nicht um CO, NOXe, Feinstaub etc.? --Zerolevel (Diskussion) 10:31, 26. Sep. 2015 (CEST)[Beantworten]
+1 Und du benutzt irreführend ein Verständnis von Betrug, das mit dem strafrechtlich verwendeten Begriff absolut nichts zu tun hat. Warum? Das ist in diesem Thread ellenlang ausgeführt, diskutiert und verlinkt. Hast du das nicht gelesen? Warum nicht? Geht's hier um Sachklärung oder nur um imkompetentes Geschwätz? --91.44.83.12 14:00, 26. Sep. 2015 (CEST)[Beantworten]
Das Management war durchaus informiert: http://bazonline.ch/wirtschaft/unternehmen-und-konjunktur/vwtechniker-warnte-bereits-vor-jahren-vor-abgasbetrug/story/22477802 --Muroshi (Diskussion) 13:13, 27. Sep. 2015 (CEST)[Beantworten]
Nein Muroshi, du lügst. Der Bericht der Basler Zeitung vom 27. September 2015, den du als Beleg verlinkst, schreibt eindeutig: „Wer davon wusste und warum die Warnung folgenlos blieb, ist unklar.“ Deshalb kann man nur, wie die Zeitung das auch macht, schreiben, dass unter Berufung auf die interne Revision des Konzerns sowie auf Informationen aus Aufsichtsratskreisen die Frankfurter Allgemeine Sonntagszeitung meldet, „der Auto-Konzern [soll] bereits vor Jahren vor dem Einsatz der Abgas-Software gewarnt worden sein. 2011 soll ein VW-Techniker auf die illegalen Praktiken aufmerksam gemacht haben“. Es ist aber völlig unverantwortlich und inakzeptabel, in diesem Stadium der Untersuchung wie du zu behaupten: „Das Management war durchaus informiert.“ Denn (nochmal): „Wer davon wusste und warum die Warnung folgenlos blieb, ist unklar.“ --91.44.83.12 14:49, 27. Sep. 2015 (CEST)[Beantworten]
Völliger Unsinn: "Bei den Untersuchungen in der Abgas-Affäre soll die interne Revision des Fahrzeugherstellers auf ein Schreiben des Zulieferers Bosch gestoßen sein. Nach einem Bericht der Bild am Sonntag habe Bosch VW bereits 2007 darauf hingewiesen, dass die von ihm gelieferte Software nur für Testzwecke und nicht für den normalen Fahrbetrieb vorgesehen gewesen sei. Demnach teilte er den Wolfsburgern mit, dass der geplante Einsatz gesetzeswidrig sei." Offenbar existiert also ein Schreiben vom Softwareentwickler Bosch an VW. http://www.zeit.de/wirtschaft/unternehmen/2015-09/volkswagen-skandal-zeitplan-umruestung --Muroshi (Diskussion) 16:57, 27. Sep. 2015 (CEST)[Beantworten]
Liest du manchmal spaßeshalber die Beiträge, auf die du antwortest? Hint: "Management" --Eike (Diskussion) 17:07, 27. Sep. 2015 (CEST) [Beantworten]
Muroshi liest scheinbar nichtmal die Belege, die er anführt. Auch die von ihm verlinkte ZeitOnline vom 27. September 2015 schreibt: „Es sei jedoch nicht geklärt worden, warum diese Warnung seinerzeit folgenlos blieb und wer darüber informiert gewesen war.“ Das ist so eindeutig, dass zumindest ich bei Muroshis Behauptung nicht mehr von einem Irrtum oder einen Mißverständnis ausgehen kann. Ich denke, er lügt. --91.44.83.12 17:30, 27. Sep. 2015 (CEST) [Beantworten]
Doch, doch. Bei einem offiziellen Warn-Schreiben von Bosch an VW kann man nicht behaupten, das Management wäre nicht über ein solches Schreiben informiert worden. Eine dahingehende Behauptung ist einfach nur absurd. "Es sei jedoch nicht geklärt worden, warum diese Warnung seinerzeit folgenlos blieb und wer darüber informiert gewesen war" bezieht sich auf den Techniker. Deutsche Sprache, schwierige Sprache.--Muroshi (Diskussion) 18:37, 27. Sep. 2015 (CEST)[Beantworten]

Der Bericht der Bild am Sonntag, auf den sich ZeitOnline bezieht, spricht (bei allen berechtigten Vorbehalten bei dieser Quelle) von einem „Schrei­ben des Auto­zu­lie­fe­rers Bosch an den VW-Konzern aus dem Jahr 2007“ Nirgendwo ist präzisiert, wer von Bosch an wen bei VW geschrieben hat. Nach Angaben von Bild handelt es sich um eine Warnung: „Darin warnt Bosch vor der ille­ga­len Ver­wen­dung sei­ner Tech­nik zur Abgas­nach­be­hand­lung. (...) Bosch hatte die Soft­ware an VW gelie­fert, die aller­dings nur für Test­zwe­cke und nicht für den nor­ma­len Fahr­be­trieb vor­ge­se­hen war. Nach BamS-Informationen teilte der Zulie­fe­rer damals den Wolfs­bur­gern mit, dass der geplante Ein­satz geset­zes­wid­rig sei.“ Weil es bis jetzt keine gesicherte Information der internen Revi­sion bei VW zum konkreten Empfänger des Schreibens gibt und was mit dem Schreiben geschehen ist, ist die Behauptung, das VW-Managemet sei informiert gewesen, grundlagenlos, spekulativ und den tatsächlichen Ergebnissen der Untersuchung vorausgreifend. Oder eben ein plumper Versuch, Sachverhalte und Tatsachen zu entstellen und grob verzerrt darzustellen, sprich zu lügen. --91.44.83.12 19:25, 27. Sep. 2015 (CEST) [Beantworten]
So, es läuft jetzt auch ein Ermittlungsverfahren.[2] Können wir die Frage damit schließen? --Eike (Diskussion) 13:00, 28. Sep. 2015 (CEST)[Beantworten]

Neues Meldegesetz ab 1. November

Verstehe ich das neue Meldegesetz richtig, dass man sich nirgends melden kann, wenn der Auszug aus der vorherigen Meldeadresse nicht bestätigt wird? Heißt das dann, ich soll mich noch schnell im Oktober ohne festen Wohnsitz melden, damit ich später keine Probleme bekomme, wenn ich eine Wohnung bekomme? -- Liliana 23:03, 24. Sep. 2015 (CEST)[Beantworten]

Nein, du sollst nicht. Der Vermieter ist in Zukunft verpflichtet, den Ein- und Auszug eines Mieters innerhalb von zwei Wochen schriftlich zu bestätigen. Um dieser Verpflichtung des Vermieters Nachdruck zu verleihen, hat der Gesetzgeber die Androhung eines Bußgeldes bis zu 1.000 Euro vorgesehen, wenn die Bescheinigung nicht rechtzeitig ausgestellt wurde. --88.130.78.149 23:15, 24. Sep. 2015 (CEST)[Beantworten]
Das ist das wirklich Schöne an der WP-Auskunft: Man erfährt (durch Frage und Antwort) Dinge, von denen man nicht einmal ahnte, daß es sie gibt. - Davon mal ab: Willkommen im Wohlfühlwunderland der Deregulierung! 188.109.67.241 23:30, 24. Sep. 2015 (CEST)[Beantworten]
Vor 40 Jahren hätte der Staat das brauchen können. Was es heute soll, kann nur vermutet werden.[3] --Hans Haase (有问题吗) 23:37, 24. Sep. 2015 (CEST)[Beantworten]
Das heißt dann, wenn der Vermieter gegen das Bußgeld Einspruch einlegt, den Rechtsweg ausschöpft, womöglich noch zum Bundesverfassungsgericht geht um das Meldegesetz als verfassungswidrig einstufen zu lassen, dann vergehen Jahre und in all diesen Jahren kann ich keine Wohnung beziehen? Das bestärkt mich ja eher darin, mich schnell jetzt noch abzumelden. -- Liliana 20:26, 25. Sep. 2015 (CEST)[Beantworten]
Liliana, komm wieder auf den Teppich. Wenn ein Vermieter gegen diese Pflicht verstoßen hat, dann hat er dagegen verstoßen. Ich sehe nicht, wo da ein Einspruch, eine Klage, der Gang zum Bundesverfassungsgericht, zum Europäischen Gerichtshof für Trallafitti und schließlich der zum lieben Gott weiterhelfen sollte. Eine ganz ähnliche Pflicht gab es schon vor über 10 Jahren und damals mit noch stärkeren Pflichten für den Vermieter. Wäre diese Pflicht nicht verfassungskonform gewesen, dann wäre damals mit Sicherheit jemand erfolgreich dagegen vorgegangen - ist aber nicht. Heute, wo die Verpflichtung weniger weit reicht, dürften die Chancen eher schlechter als besser sein. Diese bis zu 1.000 Euro können je nach Wohnungsgröße eine ganze Reihe an Monatsmieten sein - aus reiner Boshaftigkeit auf die zu verzichten, so dumm dürften die meisten Vermieter nicht sein. --88.130.95.138 20:46, 25. Sep. 2015 (CEST)[Beantworten]

Mglw. Nebenfrage: Wie ist das denn, wenn (so mal als Bleistift) der Schwiegervater von der bisherigen Mietwohnung zur verheirateten Tochter und deren Mann in deren Wohnung zieht, ohne dort Miete zu zahlen? Wer soll dann einen "Einzug" bestätigen? Si! SWamP hier: Inaktive Admins 23:41, 24. Sep. 2015 (CEST)[Beantworten]

Das wäre der perfekte Beleg fürs Finanzamt. Zuwendung! Voteil! Vorteilsnahme! Ausgewulft! Her mit der Kohle, bis es dem Staat gehört und dann gibt es die «Single-Einheitswohnung» mit 44 m² wie im Sozialgesetzbuch definiert. --Hans Haase (有问题吗) 23:49, 24. Sep. 2015 (CEST)[Beantworten]
Entschuldige bitte, ich meine die Frage ernsthaft. Der im Beispiel genannte Mann ist alt und wohnt nun wieder bei der Familie, ohne Miete. Wenn ich aber die initiale Frage richtig verstehe, muss die *Abmeldung* bestätigt werden, nicht die Anmeldung. Si! SWamP hier: Inaktive Admins 23:54, 24. Sep. 2015 (CEST)[Beantworten]
Nein, beides. -- Liliana 23:55, 24. Sep. 2015 (CEST)[Beantworten]
es gibt im geschilderten Fall aber doch gar keinen Vermieter? Wer soll denn dann bestätigen? Si! SWamP hier: Inaktive Admins 23:58, 24. Sep. 2015 (CEST)[Beantworten]
der Eigentümer. -- Liliana 23:58, 24. Sep. 2015 (CEST)[Beantworten]
Also es wird im Prinzip eine Regelung wieder eingefügt, wie es sie bereits bis 2002 gab. Der wesentliche Unterschied besteht darin, daß der Vermieter keinen Kontrollzwang hat, das sich seine Bewohner wirklich anmelden, dies ist bloß noch eine KANN-Regel. Das Ganze betrifft natürlich nur Personen, welche sowohl meldepflichtig als auch wohnberechtigt sind. Der Schwiegervater fällt unter die Regelung des § 553 BGB, das wägt zwischen dem berechtigten Interesse der Mieter und dem Interesse des Vermieters hinsichtlich einer Überbelegung ab. Wenn heute teilweise 300 Personen in einer Wohnung gemeldet sind, ist der Mißbrauch offensichtlich. Genauso soll es vorkommen, das unerlaubter Familiennachzug in Wohnungen zur Überbelegung führen. Erstmal Privatsache, wenn der Schwiegervater auf dem Sofa im Wohnzimmer schläft, wenn die Wohnung aber nur 2 Räume und 31 Quadratmeter hat, kann es strittig werden. Miethöhe ist davon nicht betroffen, nur die Pflicht solche Bescheinigung zu erteilen. Problem sind dabei nicht unbedingt die Eltern der Eheleute, sondern eher Geschwister und deren Nachwuchs. Die Grenze liegt da bei 10qm je Erwachsener und 6 je Kind. Oliver S.Y. (Diskussion) 23:56, 24. Sep. 2015 (CEST)[Beantworten]

Folgende Situation. Ich habe eine Wohnung in der Stadt X in der ich auch studiere. Dort bin ich 4-5 Tage die Woche, nicht jedoch in den Semesterferien. Die verbringe ich bei meinen Eltern, deren Abstellkammer gemütlicher ist als diese Absteige. Der Mitevertrag für meine Wohnung läuft über meine Mutter und die bezahlt das ganze auch.

Soll ich nun irgendetwas tun. Ich will mich nicht umelden.--93.230.91.45 17:03, 25. Sep. 2015 (CEST)[Beantworten]

Theoretisch müsstest du die Wohnung als Zweitwohnung anmelden und dafür dann Zweitwohnsitzsteuer zahlen. Ansonsten begehst du womöglich Betrug. -- Liliana 20:24, 25. Sep. 2015 (CEST)[Beantworten]
Die Meldepflicht beruht auf dem "Beziehen" einer Wohnung. Das Verbringen einer Zeit in einer fremden Wohnung während der Ferien ist kein Beziehen. Somit besteht kein Konflikt hinsichtlich der Zweitwohnsitzsteuer. Dieses Problem hat Deine Mutter.Oliver S.Y. (Diskussion) 20:30, 25. Sep. 2015 (CEST)[Beantworten]
Und du schädigst Deine Uni-Stadt, da Du dort nicht gemeldet bist und daher der Stadt Zuschüsse für Dich nicht gewährt werden,obwohl Du durch Deinen Aufenthalt dort Unkosten verursachst. --Hachinger62 (Diskussion) 23:29, 25. Sep. 2015 (CEST)[Beantworten]

Man konnte sich schon 1992 in (z. B. Spanien) oder 1995 (in der Tschechei) "polizeilich melden". In Spanien hieß das Residencia, ich hatte dort ein Jahr meinen Hauptwohnsitz und in Deutschland keinen. Heute ist das noch viel einfacher. Welche Adresse auf dem Ausweis hinten drauf steht, interessiert das andere Land nicht. Die ändern daran auch nichts, das dürfen sie nicht. Wie auch die deutsche Polizei (oder wer auch immer) meinen spanischen Führerschein zwar anschauen darf, ihn strenggenommen aber nicht fordern und erst Recht nicht einbehalten darf, der gehört nämlich dem spanischen König. Von Gewaltverbrechen sehen wir mal ab... Als Deutscher kann man sich überall in Europa anmelden, wenn man die dortigen Gesetze einhält. Solange man nicht polizeilich gesucht wird, geht das dann den deutschen Staat auch nichts an. --Pölkkyposkisolisti 00:35, 26. Sep. 2015 (CEST)[Beantworten]

Solange du dort auch tatsächlich deinen Lebensmittelpunkt hast ist das ok. Ansonsten ist das Fahren ohne Führerschein. Und gerade weil er nicht in deinem Eigentum ist, kann ihn dir die deutsche Polizei abnehmen, das hat nämlich der spanische König erlaubt. --Eingangskontrolle (Diskussion) 21:16, 26. Sep. 2015 (CEST)[Beantworten]
Anderswo musste man sich selbst als Tourist polizeilich melden. Ich erinnere mich, 1984 für einen Zwei-Tage-Aufenthalt bei meiner Stiefoma in Ungarn auf der Meldebehörde rumgesessen und Meldezettel in einer mir fremden Sprache ausgefüllt zu haben. --Rôtkæppchen₆₈ 01:15, 26. Sep. 2015 (CEST)[Beantworten]
Anderswo muss man sich selbst als Tourist polizeilich melden. Der Normaltourist merkt davon aber meistens eher wenig, weil das i. d. R. das Hotel macht. Das fällt einem nur dann näher auf wenn man privat übernachtet. In den USA z. B. (woogegen sich US-Bürger nirgendwo melden müssen).--Antemister (Diskussion) 19:46, 28. Sep. 2015 (CEST)[Beantworten]

25. September 2015

Mobbing

Warum sind übergewichtige Schüler meist Außenseiter? --89.144.198.94 17:49, 25. Sep. 2015 (CEST)[Beantworten]

Vom Durchschnitt oder vom gerade aktuellen Ideal abweichende Menschen werden schnell zum Außenseiter (gemacht). Das ist ein wohl ziemlich archaisches Verhalten, das bei Kindern besonders deutlich wird. Zum Glück kann man sich das weitgehend abgewöhnen. In eine ungewollte Außenseiterposition gelangt man nicht nur als Dicker, das kann auch bei Abweichungen ganz anderer Art passieren. Hautfarbe, Religion, sexuelle Orientierung, Hakennase, lahmes Bein, rote Haare usw. Abhilfe schaffen erlernte Dinge wie Toleranz und Akzeptanz auf der einen Seite, emanzipatorisches Selbstbewusstsein auf der anderen. Ist leicht gesagt, aber natürlich schwer. Rainer Z ... 19:31, 25. Sep. 2015 (CEST)[Beantworten]
Übergewicht entspricht nicht dem Schönheitsideal. Daher ist es für dicke Kinder schwieriger beliebt zu sein (also nicht gemobbt zu werden) als für schlanke. Aber unmöglich ist es trotzdem nicht. --MB-one (Diskussion) 19:28, 25. Sep. 2015 (CEST)[Beantworten]
Muss auch dazusagen dass Übergewicht ja stark mit Disziplinlosigkeit ("Armut" in Industriegesellschaften) konnotiert, genug Leute wollen mit solchen nicht allzuviel Umgang haben.--Antemister (Diskussion) 20:07, 25. Sep. 2015 (CEST)[Beantworten]
Mit dem Schönheitsideal hat das eher wenig zu tun. Es reicht wenn man anders ist, als die meisten anderen. Wenn es einen gibt der dem Schönheitsideal entspricht und der Rest der Klasse übergewichtig ist, dann werden die Übergewichtigen eher nicht gemobbt, sondern der schöne. Aber stark übergewichtige Kinder sind hierzulande (zum Glück) noch eher die Ausnahme. -- Jonathan 20:11, 25. Sep. 2015 (CEST)[Beantworten]
Das hat auch damit zu tun, dass die sich halt oft nicht wehren können. Wenn der Dicke nicht so ist, wie man ihn sich vielleicht vorstellt, (isst, anstatt sich zu bewegen, kein Sport, bei dem er abnehmen würde, wenig Kraft) sondern (trotzdem) stark, dann wird er oft nicht gemobbt. --MannMaus 20:16, 25. Sep. 2015 (CEST)[Beantworten]

Als früher übergewichtiges Kind muß ich dem Klischee wiedersprechen. Es gibt auch unter den "Dicken Kindern" unterschiedliche Typen. Sie können genauso Täter und Mobber sein, insbesondere wenn nicht nur schwerer, sondern auch größer bzw. körpüerlich weit entwickelter sind (Typ Nelson), dann gibt es die klassichen "Opfer", welche häufig nicht nur dick, sondern auch etwas geistig minderbemittelt sind (Typ Ralph). Und dann natürlich noch die Nerds und Clowns. Das Problem sind nicht die Mobber, welche vermeintlich dem Ideal entsprechen, sondern diejenigen Abweichler, welche versuchen, in der Hackordnung höhere Plätze zu erreichen, indem sie vermeintlich Abweichende herabstufen wollen. Also ganz einfach, rechtzeitig solche Typen auf die Fresse hauen oder mit Sand füttern, und der Rest der Schulzeit ist kein Problem, außer dann dann hier die Threads kommen, warum Brillenträger, Ausländer, Kleine, Dünne, Rothaarige und Schamhafte gemobbt werden. Jedes Mobbingopfer ist auch irgendwann mal Mittäter, auch wenn mans nicht gern zugeben will.Oliver S.Y. (Diskussion) 20:21, 25. Sep. 2015 (CEST)[Beantworten]

Es soll Leute geben, die es schon als Kind mit dem Fresse hauen absolut nicht haben. Was machste dann? Rainer Z ... 20:47, 25. Sep. 2015 (CEST)[Beantworten]
Frühzeitig ein schützendes soziales Netz aufbauen, sowohl was Freunde als auch "ideale" Kinder angeht. "Immer an der Wand lang" ist auch eine Überlebensstrategie.Oliver S.Y. (Diskussion) 21:06, 25. Sep. 2015 (CEST)[Beantworten]
Aha. »Immer an der Wand lang« mag eine Überlebensstratege sein, ist aber ziemlich entwürdigend. Ich habe die Schule gewechselt, danach bin ich aufgeblüht. Rainer Z ... 22:21, 25. Sep. 2015 (CEST)[Beantworten]
Ja, da wurde das wesentliche gesagt. Die überzogenen Ideale entspringen einer Art von Werbung, die nicht das Produkt in den Vordergrund stellt, sondern das Produkte der Branche als Lösung geschaffener Probleme suggeriert und darstellt. Es funktioniert mit Kleidung, Nahrungsergänzung, Drogerieartikeln, Musik und motorisierter Mobilität. Die Kollateralschäden davon zeigen sich bei den Empfängern der Werbenachricht als Hirnschiss, der sie zu Akteuren macht, die diesen suggerierten Idealen selbst nicht entsprechen und sich beschriebener Situation des Fragestellers negativ auswirkt. Auf das Gewicht hat die Geringwertschätzung meist einen nicht unbedingt messbaren, aber vorhanden negativen Einfluss. Das mag vergleichbar sein mit dem Verhalten von Borderlinern, deren Stimmungs-High sich in Mitkopplung befindet, also sich aufschaukelt und gegenseitig und mit sich selbst verstärkt und sich durch das Publikum ebenfalls Echo bekommt. Diese Mitkopplung abreisen zu lassen kann darin erfolgen, sie öffentlich zu demütigen. Beispiele, von denen ich abrate, wären eine schallende Ohrfeige vor unentschlossenem Publikum, oder der Wurf eines großen nassen Schwammes, der im Intimbereich des Mobbers unbeobachtet von dritten aufprallt und sein Wasser dort abgibt, kann recht hilfreich sein und eignet sich, wenn das Publikum parteiisch ist. Das Schulpflicht besteht, passiert in der Schule nicht all zu viel, in Lehre und Beruf ist das mindestens Abmahnung oder Kündigung, bis hin zur strafrechtlichen Verfolgung! Solchen Zeitgenossen folgt man mal auf die Toilette der Schule, am besten mit einem guten Verbündeten, der die Klappe halten kann und hinterher glaubwürdig aussagt, er war auf der Toilette und habe nichts mitbekommen, außer dass es immer Streit zwischen den beiden gab. Der Zweite ist nur dafür da, das Schamgefühl bei Wasserlassen des Mobbers zu erhöhen. Auf die Idee sich umzudrehen und sich mit Urin zu verteidigen kommt er damit um so weniger. Ferner darf er die Zeit dafür nicht bekommen. Für den Transport des Wassers tut es hier ein Becher. Übrigens wirkt das auch auf und bei der Redegewandtheit. Dies ist strafrechtlich weniger Relevant. Die Beispiele zeigen, auf welchen primitiven Eigenschaften dies beruht.
Ein typisches Beispiel für die Vorgänge im Gehirn des Opfers ist die folgende Situation, die zeigt, wie Opfer die Opferrolle annehmen: Bei einem Besuch eines Dritten lernen sich Mobber und Opfer kennen. Mobber hat eine Führerschein, Opfer erzählt es sei gerade dabei den Führerschein zu machen. Mobber suggeriert, der Führerschein sei nicht nur die Berechtigung, sondern die bescheinigte Fähigkeit, ein Fahrzeug zu führen, was auch eine psychologische Frage zur Eigen- und Außenwahrnehmung von Verkehrsteilnehmern ist, denn es befinden sich weit über 50 % der Fahrer besser als der Durchschnitt der Fahrer. Im Verlauf des Gesprächs fragt der Mobber sein Opfer, warum es den Führerschein nicht auf der Rolltreppe mache, das sei doch billiger als in der Fahrschule, worauf er beim Opfer die Fähigkeit und der damit relativ fest verknüpften Selbstwertschätzung mit dem Schlagwort Rolltreppe verknüpfte. Der Gastgeber amüsierte sich unterdessen die Fähigkeit ein Fahrzeug auf diese vorhandene Weise zu steuern, auszulagern. Um sich Zustimmung zu holen, äußerte sich der Mobber über die ein oder andere Richtlinie kritisch über die ein oder andere Regelung im Straßenverkehr, die in der Fahrschule thematisiert wird. Unterbewusst findet dabei der Vergleich statt, der Abstand, Geschwindigkeit, Richtung und Vorfahrt weitgehend selbst regelt, was der Verwunderung der verblüffenden Vereinfachung zunächst eine Menge pragmatisches ergibt und den Humor gelingen lässt. Der Mobber wollte nun einen Effekt auf sein Opfer projizieren, aber kein Täter sein. Dazu erzählte er einem Vierten, der das Opfer kannte, er solle ihm mit „Hallo Rolltreppe“ begrüßen. Würde der vierte nun «die wirksame Grußformel» an weitere übermitteln, ohne selbst den Inhalt der Konversation des Besuches zu kennen, aber von der Reaktion des Opfers allein motiviert zu sein, dies weiter zu verbreiten. Nach diesem Phänomen, welches keines ist, wird politischer Humor in Sozialen Netzwerken verbreitet. Auch hier erahnen viele den realen Hintergrund der Nachricht nicht. Die virale Werbung, die das ist, entfaltet ihren Effekt. --Hans Haase (有问题吗) 11:25, 26. Sep. 2015 (CEST)[Beantworten]
"Übergewichtige Schüler sind meist Außenseiter." <= Bitte belegen.
"Außenseiter" ist immer die Person, die nicht den Normen und Vorstellungen einer (betrachtenden) Gruppe entspricht. Dabei gibt es Minuspunkte für Sprache, Verhaltensweisen und Aussehen. Wer am meisten Minuspunkte sammelt, wird ausgegrenzt und weggebissen (Gleich und gleich gesellt sich gern). Bei den Dicken ist der Dünne Außenseiter. In Dortmund der Schalke-Fan, usw. usf. Solches Denken hilft Kindern, die nach Gruppenanerkennung lechzen ("Lechz, lechz!", bei der Abgrenzung von "Wir" und "Die". Kinder, die das differenzierter sehen können, weil die Eltern (soziales Umfeld etc.) ihren Job gemacht haben und ihnen das eingebläut beigebracht haben, brauchen das weniger. 213.169.163.106 13:18, 26. Sep. 2015 (CEST)[Beantworten]
Weil die Sonne meistens grün ist. --217.84.83.134 03:32, 27. Sep. 2015 (CEST)[Beantworten]
Übergewichtige Kinder weisen häufiger als schlanke Verhaltensauffäligkeiten auf (bspw. ADHS), weil das dopaminerge System eine Schieflage aufweist, die sowohl Verhaltensauffälligkeiten als auch der Entstehung von Übergewicht Vorschub leistet. Viele werden auch erst aus Frust übergewichtig, weil sie aus verschiedenen Gründen bereits als schlanke Kinder ausgegrenzt wurden. Dann werden sie wegen dem Übergewicht noch mehr ausgegrenzt und futtern noch mehr et cetera. --94.219.10.213 19:57, 28. Sep. 2015 (CEST)[Beantworten]

Wo ist das?

Foto --2A02:810D:27C0:5CC:D58B:24D5:C5C1:ACD8 17:51, 25. Sep. 2015 (CEST)[Beantworten]

Error (410) Something went wrong. Don't worry, your files are still safe and the Dropboxers have been notified. Sicher, dass der Link richtig ist? --mfb (Diskussion) 19:43, 25. Sep. 2015 (CEST)[Beantworten]
Hmmm, geht [4]? --2A02:810D:27C0:5CC:D58B:24D5:C5C1:ACD8 19:57, 25. Sep. 2015 (CEST)[Beantworten]
Also bei mir geht beides. Wo das ist, hab ich aber auch keine Ahnung. Kannst du das wenigstens irgendwie nach Kontinent eingrenzen oder so? -- Jonathan 22:23, 25. Sep. 2015 (CEST)[Beantworten]
Angebenes Aufnahmedatum ist der 24.09.2015 10:46. Die angegebenen GPS-Koordinaten sind: NS=50/47/48.5, EW=7/11/43.56, Höhe über Normal-Null 62. Die Koordinaten zeigen wohl nach Siegburg. --Pp.paul.4 (Diskussion) 01:35, 26. Sep. 2015 (CEST)[Beantworten]
Cool, jetzt wissen wir wo das Foto ist. Wenn mich nicht alles täuscht, dann ist da Wetterhorn und Schreckhorn zu sehen, die Kamera stand also in den Berner Alpen, vermutl. in der Nähe von Grindelwald. Just $0.02 von -- Iwesb (Diskussion) 02:57, 26. Sep. 2015 (CEST)[Beantworten]
Sorry, aber das ist witzlos, weil ja nicht der Ort des abfotografierten Zeitungsausschnitts gefragt ist, sondern der auf dem Zeitungsausschnitt abgebildete Ort. --Rôtkæppchen₆₈ 03:09, 26. Sep. 2015 (CEST)[Beantworten]
Da: http://www.rigikulm.ch/ --тнояsтеn 16:24, 29. Sep. 2015 (CEST)[Beantworten]
Kräftig weit weg von Grindelwald, dafür richtig (Rigi). --83.79.9.134 17:42, 29. Sep. 2015 (CEST)[Beantworten]
Und das „Wetterhorn“ ist demzufolge der Uri Rotstock: Panorama mit Bergnamen via [5]. --Sitacuisses (Diskussion) 04:08, 30. Sep. 2015 (CEST)[Beantworten]

VW-Abgasskandal

Ich habe eine Frage zu dem VW-Abgasskandal in Bezug auf österreichische bzw. deutsche Autos von VW. Müssten die betreffenden Fahrzeuge nicht augenblicklich die Zulassung verlieren? Wenn ich das richtig verstanden habe, verstoßen alle diese Fahrzeuge gegen die aktuell geltenden Abgasrichtlinien und müssten sofort stillgelegt werden? Und was Deutschland betrifft: Es ist doch eigentlich so, dass man die Umweltplakette nur bekommt wenn man die entsprechenden Abgasnormen erfüllt?

--Generator (Diskussion) 20:37, 25. Sep. 2015 (CEST)[Beantworten]

Die Abgasnormen müssen auf dem Prüfstand gemessen werden und genau dort und nur dort erfüllen die Fahrzeuge die Norm. In der Norm steht nichts davon, dass die Emissionsgrenzwerte außerhalb des Prüfstands auch erfüllt sein müssen. Deswegen sehe ich da auch gar kein Problem. --Rôtkæppchen₆₈ 20:46, 25. Sep. 2015 (CEST)[Beantworten]
darüber hinaus ganz interessant und erhellend: VW-Skandal: Rücktrittsrecht vom Autokauf wegen Manipulation von Abgaswerten? --JD {æ} 21:00, 25. Sep. 2015 (CEST)[Beantworten]
Das ist letztlich auch eine politische Entscheidung: Die Autokäufer sind ja genau so getäuscht worden wie die Untersucher am Prüfstand. Will man jetzt die Käufer bestrafen für eine Täuschung, der sie selbst schuldlos zum Opfer gefallen sind? Oder sollte man nicht eher denjenigen bestrafen, der aktiv getäuscht hat? Wenn dieser Skandal das Unternehmen VW nachher eine Stange Geld kostet, dann ist das doch vom Prinzip her nichts anderes als der Handel mit CO²-Zertifikaten... --88.130.95.138 21:02, 25. Sep. 2015 (CEST)[Beantworten]
Die Abgasnormen müssen auf dem Prüfstand gemessen werden und genau dort und nur dort erfüllen die Fahrzeuge die Norm. In der Norm steht nichts davon, dass die Emissionsgrenzwerte außerhalb des Prpfstands auch erfüllt sein müssen...
Deshalb baue ich die Katalysatoren auch immer nur für die Abgasuntersuchung ein, damit bestehen meine Fahrzeuge die HU... anschließend kommen die Kats raus und werden im trockenen Keller für die nächste HU gelagert, damit sie nicht kaputt gehen. <scnr> --Btr 21:15, 25. Sep. 2015 (CEST)[Beantworten]
Mag sarkastisch gemeint worden sein, aber ich kenne im Raum Frankfurt am Main alleine zwei Autowerkstätten, wo das genau so gemacht wird. --91.3.9.24 09:42, 26. Sep. 2015 (CEST)[Beantworten]
...bei meiner BMW mache ich das auch so... die besteht die Abgasuntersuchung allerdings auch mit Y-Rohr und ohne Kat, wenn sie richtig heiß gefahren wurde und ein fähiger Prüfer -also nicht die Dilettanten von TÜV- am Werk ist. --Btr 12:45, 26. Sep. 2015 (CEST)[Beantworten]
BK Du schneidest ein heißes Eisen an. Auf dem Prüffeld haben die Motoren den Test bestanden und haben die Zulassung erhalten. Wenn sich, was bisher noch niemand genau weiß, herausstellen sollte, dass das Steuergerät den Abgasfilter bei Straßenfahrt abschalten sollte, müsste die Allgemeine Betriebserlaubnis entzogen oder eine andere Schadstoffklasse vergeben werden. Die Zahlen, die Dobrindt so täglich herauslässt, dokumentieren, dass im Verkehrsministerium bereits die Köpfe rauchen. Der Neue bei VW wird u.a. als vordringliche Aufgabe haben, in Deutschland eine Überreaktion zu verhindern. Die Deutschen Behörden können sich aber auch keinen Schnellschuss erlauben, dazu ist das Land zu stark vom Auto(Export) abhängig und im juristischen Sinne sind die Vorwürfe (noch) nicht rechtsgültig/ gesichert. --2003:68:ED12:500:A0CF:7019:7DE9:678 21:25, 25. Sep. 2015 (CEST)[Beantworten]
Wie willst Du das reproduzierbar festlegen? Die Emissionsmessungen werden aus gutem Grund auf dem Prüfstand und nicht auf der Straße gemacht. Das ergibt reproduzierbare Ergebnisse. VW hat da meiner Meinung nach nur eine bestehende Regelungslücke genutzt. Ein Betrug liegt meiner nicht maßgeblichen Meinung nach nicht vor. Auch ein Erlöschen bestender Zulassungen oder Schadstoffklassen kann ich mir nicht vorstellen, da die Emissionswerte den bestehenden Regelungen nach ermittelt wurden. Was allerdings getan werden muss, ist die Regelungen für Emissionsmessungen und Motorsoftware dahingehend zu ändern, dass Messwerte oder Sensorik für Navigation, ABS, ASR, ESP oder Einparkhilfe von der Motorregelung nicht ausgewertet werden dürfen. Außerdem halte ich es für geboten, dass Motorregelungssoftware quelloffen sein oder von unabhängigen Stellen zertifiziert werden muss. --Rôtkæppchen₆₈ 23:19, 25. Sep. 2015 (CEST)[Beantworten]
Quelloffen heißt aber nicht unbedingt nachvollziehbar, das zeigt nicht umsonst jedes Jahr der International Obfuscated C Code Contest. -- Liliana 23:22, 25. Sep. 2015 (CEST)[Beantworten]
Diesmal war ein interessantes aktives Publikum [6] dabei, das es ziemlich gut auf den Punkt brachte. --Hans Haase (有问题吗) 00:01, 26. Sep. 2015 (CEST)[Beantworten]
Rot..., vergesse die Emissionen. Die müssen irgendwie gemessen werden und da hat man sich wie du schreibst auf den Prüfstand geeinigt. Der Kasus ist der sich geheim abschaltende Abgasfilter, der die Betriebserlaubnis (und auch die Emissionen) verändert. Nur da kann das KBA eingreifen.--2003:68:ED3A:5700:64D6:E84E:DA92:6CB9 10:03, 26. Sep. 2015 (CEST)[Beantworten]
Im Lebensmittelsektor ist diese Art der Verbrauchertäuschung gang und gäbe. Niemand regt sich ernsthaft darüber auf, dass Pudding mit Vanillearoma niemals eine Vanilleschote auch nur von weitem gesehen hat, und ob die Scheibe gekochter Schinken wirklich nur von einem Tier stammt, interessiert auch kaum jemand. Das, was der Verbraucher glaubt zu bekommen und das, was er wirklich erhält, sind hier zwei verschidene Dinge.- VW hat die geforderten Prüfstandmessungen bestanden. Jeder Autofahrer weiß, dass diese Werte unmöglich in allen Situationen eingehalten werden können, z.B. beim Kaltstart und im Stop-and-Go-Verkehr. Im Normalbetrieb hat dann die Bordelektronik auf "Fahrspaß" geschaltet. Wenn in den zuständigen Verordnungen keine eindeutigen Vorgaben gemacht wurden, ist das wahrscheinlich nicht mal gerichtsverwertbar.--Optimum (Diskussion) 12:18, 26. Sep. 2015 (CEST)[Beantworten]
PS.: Vielleicht sind Autofahrer, die mal die ASU bei einen VW-Käfer gesehen haben, von diesem "Skandal" weniger überrascht. Da wurden die entsprechenden Schrauben und Rädchen so eingestellt, das der Motor stotterte wie eine asthmakranke Harley. Das war aber anscheinend die Einstellung mit den niedrichsten Grenzwerten, denn so wurde gemessen. Weil aber der Kunde mit dieser Einstellung nicht vom Hof kam, ohne den Motor dreimal abzuwürgen, wurde nach der Messung alles wieder auf optimalen Rundlauf zurückgedreht. --Optimum (Diskussion) 12:29, 26. Sep. 2015 (CEST)[Beantworten]
Um den Vergleich mit dem Vanillearoma aufzugreifen: Wenn ein Puddinghersteller behauptet, es wäre echte Vanille drin, das ist aber nicht der Fall, wäre es durchaus Betrug und die Menschen würden sich darüber aufregen. Oder wenn sie zur Lebensmittelkontrolle einen modifizierten Pudding geben, aber die im Handel eine andere Zusammensetzung hätten. VW hat klar gesagt, dass die und die Werte vorhanden sind. Sie haben nicht als Fußnote geschrieben "Gilt nur bei der Prüfung". ;) Es ist ja nicht so, dass die Werte durchaus real sind und abweichen können. Nach allem, was wir wissen, können die Werte überhaupt nicht in der Realität eingehalten werden (und das ist ein Unterschied, insbesondere, wenn VW gerade damit geworben hat). So ähnlich wie, wenn ein Naturjoghurthersteller schreibt, dass die Zusammensetzung der enthaltenen Kirschzubereitung aufgrund der Eigenart als Naturprodukt abweichen kann, sich dann aber herausstellt, dass der Hersteller bewusst gar nie Kirschen zugesetzt hat und die angegeben Prozentwerte daher grundsätzlich falsch sind. Davon abgesehen ist "andere machen das ja auch" keine gültige Ausrede. Irgendwo muss man halt anfangen aufzuräumen... --StYxXx 14:56, 27. Sep. 2015 (CEST)[Beantworten]
Der Verschiebebahnhof, in dem der Schaden rangiert wird wurde nur ansatzweise angesprochen. Dieser wird nur so thematisiert, dass er als erwähnt wahrgenommen wird, seine gesamte Funktionalität nicht bewusst werden soll. Die Vorschrift besagt, den Prüfstand benutzen zu müssen. Die Prüfung muss nach wie vor durchgeführt werden. Die Amerikanischen Grenzwerte differieren zu den Europäischen. Das etwas in Europa faul ist, mag die Nennung der Zahl von 2.8 Mio Fahrzeugen[7] bereits aussagen. Ob nun alles als Versehen und zum Schaden des Verbrauchers deklariert wird, indem die Feinstaubplakette zurückgezogen wird und der Verbraucher, der den Kaufbetrag erstattet hat, der Dumme ist, zwischen Wertminderung durch Benutzung und anteilig bis nicht entschädigt zu werden, oder ob nur eine Nachbesserungskampagne gestartet wird, die wegen der Terminentwicklung auf die Zuverlässigkeit des Fahrzeugs schlägt und die Neuanschaffung auf diese Weise im Nachhinein ernötigt, oder die Vorschriften gelockert werden, was dem Vertriebsinteresse der Lobby entgegenwirken würde oder beim Mitbewerb auf Klage hinauslaufen könnte, stellt die Neutralität des Rechtsstaat, in seinem Interessenskonflikt, genannt VW-Gesetz, erneut auf die Probe. --Hans Haase (有问题吗) 12:14, 26. Sep. 2015 (CEST)[Beantworten]
Das ist zu viel Polemik auf Nebenschauplätzen. Sachstand ist, dass im Fahrbetrieb der Abgasfilter abgeschaltet wird, was weder bei der Zulassungsprüfung durch das KBA (als Auftraggeber) bekannt war und aus Sicht des Umweltschutzes so auch nicht akzeptiert worden wäre. Das ist eine Täuschung der Zulassungsbehörde, der Umweltbehörde, wie auch des Kunden. Arglist liegt vermutlich nur bei denen vor, die die Software entwickelt und eingesetzt haben. Als erstes ist zu verlangen, dass diese Software ausgetauscht wird, also auch im Fahrbetrieb der Filter zugeschaltet bleibt. Was da nun an die dem Kunden versprochene Leistung des Fahrzeuges nicht eingehalten wird, muss geklärt werden. Da ist aus den Angaben in den Prospekten möglicherweise sogar kein Strick zu drehen. Und die subjektive Empfindung bei der Probefahrt ein spritziges Auto gehabt zu haben und nun eine lahme Ente fahren zu müssen, reicht vermutlich für Schadenersatz nicht aus. Anders sieht es mit dem Tatbestand einer (zusätzlichen) Umweltverschmutzung aus, das ist sicher noch lange diskussionsfähig. --2003:68:ED3A:5700:64D6:E84E:DA92:6CB9 14:23, 26. Sep. 2015 (CEST)[Beantworten]
Ist Umweltverschmutzung ein Strafrechtstatbestand?--85.4.233.141 21:39, 26. Sep. 2015 (CEST)[Beantworten]
Siehe Artikel Umweltstrafrecht und dort verlinkte Gesetzestexte. --Rôtkæppchen₆₈ 02:24, 27. Sep. 2015 (CEST)[Beantworten]
Juristisch gesehen hat nicht VW die Umweltverschmutzung verursacht, sondern der einzelne Fahrer des Autos. Jeder der z.B. seinen Kat abschraubt und mit dem Auto rumfährt begeht den Tatbestand der Umweltverschmutzung, inklusive Steuerhinterziehung. Strafverhindernd im VW-Fall ist, dass der Halter/Fahrer nicht gewusst hat, dass der Abgasfilter während der Fahrt abgeschaltet ist und Strafmilderung würde sich bei denen auswirken, wenn das Auto nur ein Mal im Monat gefahren ist und die verursachte Umweltverschmutzung nur gering war. Eigentlich müsste jetzt jeder betroffene VW-Fahrer sein Auto stehen lassen und dürfte es nicht mehr fahren. Dass das bisher von den Behörden nicht eingefordert werden kann, liegt an der Rechtsunsicherheit, besonders weil man nicht weiß welche Fahrzeuge betroffen sind. Die bisher pauschalen Aussagen von VW reichen nicht aus um tätig zu werden und Gefahr ist nicht im Verzuge. Auch rückwirkend gesehen haben die Feinstaubmessungen in den Städten keine "abnormalen" Werte gegeben. --2003:68:ED69:6100:D82:37D2:C876:4539 09:06, 27. Sep. 2015 (CEST)[Beantworten]
Es ist lange bekannt, dass PKW-Abgase keinen Einfluss auf die Feinstaubimmission haben. Das ist unabhängig davon, ob die Abgasreinigung aktiv ist oder nicht. Der immittierte Feinstaub stammt von LKW-Abgasen, Hausbrand (Brennstoff), Industrie, Bremsen-, Reifen- und Kupplungsabrieb oder Verbrennungstriebfahzeugen der Eisenbahn. Die als „Feinstaubplakette“ bezeichneten Umweltplaketten sind wirkungsloses Schlangenöl. --Rôtkæppchen₆₈ 09:13, 27. Sep. 2015 (CEST)[Beantworten]
Falls du das nicht ironisch gemeint haben solltest: Na dann fahr mal mit deiner roten Feinstaubplakette nach Schtuagart nei, wenn nur noch die grüne erlaubt ist und erzähl das dem dich stoppenden Polizeibeamten. Der wird sich kugeln und möglicherweise zum Alkotest schreiten, auch wenn du möglicherweise recht hast. Die Gegner der Feinstaubplakette lassen sich vieles einfallen und schreiben vieles in den Medien. Sicher ist das halt nicht und der ungefilterte PKW-Diesel erzeugt auch seinen Feinstaub, wie sein großer Kollege im Lastwagen auch.--2003:68:ED69:6100:D82:37D2:C876:4539 09:27, 27. Sep. 2015 (CEST)[Beantworten]
Stuttgart-Neckartor ist doch der beste Beweis: Alle bisherigen Maßnahmen zur vermeintlichen Feinstaubminderung waren wirkungslos, auch das Verbot von PKWs mit gelber oder roter Plakette oder das LKW-Durchfahrtverbot für Stuttgart. --Rôtkæppchen₆₈ 09:36, 27. Sep. 2015 (CEST)[Beantworten]
Hier wird immer wieder vom Abschalten des Abgasfilters berichtet. Wie soll denn der Katalysator (ich nehme an dass damit der "Abgasfilter" gemeint ist) abgeschaltet werden? Die Abgase gehen nach wie vor durch den Kat, da ist keine Vorrichtung zum Abschalten dran. Die Abgase werden gereinigt, nur sind diese unsauberer als im Prüfstandtest. -- Frila (Diskussion) 09:32, 27. Sep. 2015 (CEST)[Beantworten]
Vor Erfindung der Abgasreinigung nannte man das „unterstöchiometrischen Betrieb“: Es wurde ein Überschuss an Kraftstoff eingespritzt, sodass die Verbrennung unvollständig war. Dadurch stieg die Leistung, aber der Ausstoß an Kohlenstoffmonoxid und Ruß stieg. --Rôtkæppchen₆₈ 09:45, 27. Sep. 2015 (CEST)[Beantworten]
Das mit dem Kat (am Benziner) war nur ein Beispiel und Analogie und es wurde ausdrücklich "abschrauben" geschrieben. Im VW-Fall handelt es sich um Diesel-Motoren, die Reinigung erfolgt durch einspritzen von Harnstoff (AdBlue, SCR) oder Reinigung und Verbrennen der ausgefilterten Rückstände (LNT-System). Beides wurde wohl während der Fahrt abgeschaltet um besonders Sprit zu sparen.--2003:68:ED69:6100:D82:37D2:C876:4539 09:51, 27. Sep. 2015 (CEST)[Beantworten]

Die Eingangsfrage finde ich lustig. Oder interessant, je nachdem wie man es sehen will. Kaum tönt ein Politiker etwas in der tagesschau (hier ein Grüner, von wegendie Autos verlören ihre Zulassung) schon wird das ungeprüft als Tatsache übernommen. Lustig, wie man in Deutschland manchmal drauf ist. --Heletz (Diskussion) 10:06, 27. Sep. 2015 (CEST)[Beantworten]

@Heletz:: Du weißt aber schon was ein Fragezeichen bzw. die Phrase: "Wenn ich das richtig verstanden habe" bedeutet? Was meinst du, warum ich hier auf der Auskunft nachgefragt (schon wieder dieses wort das mit -ge aufhört und mit fra- beginnt) habe?
  • Weil ich etwas "als Tatsache übernommen" habe.
oder
  • meine Frage(n) beantwortet haben will.
btw habe ich keine Ahnung von welchen Politiker du sprichst. Meine Frage entstand durch den Wikipediartikel, in dem steht, dass die Autos im Normalbetrieb das 5bis35fache des in der USA geltenden gesetzlichen Grenzwertes abstoßen (Ich geh mal davon aus, dass das bedeutet, dass es auch die europäischen Regelungen nicht erfüllt). Generator (Diskussion) 17:50, 28. Sep. 2015 (CEST)[Beantworten]
Das, was der Grüne sagt, dürfte nicht falsch sein. Die Fahrzeuge verlieren tatsächlich ihre Typzulassung, sofern die Software nicht ausgetauscht wird und der Abgasfilter auch bei Fahrt funktioniert. VW arbeitet ja an der neuen Software, will sie ja austauschen und das beruhigt vorerst auch ein Mal die Behörden in USA und auch hier. Übrigens sind in USA für diese Schummelei keine direkte Strafen für den Halter im Gesetz vorgesehen, wie nur der Entzug der Typzulassung möglich ist. (Ein Auto nicht fahren können und laufen müssen ist für einen Amerikaner auch Strafe genug. SCNR) Mehr Gelassenheit täte auch uns Deutschen gut. Wunder dauern immer etwas länger. Die SW muss geschrieben und ausgetestet, sowie zugelassen werden. Dann erfolgt die Rückrufaktion durch das KBA, denn nur die haben die Adressen der Halter. Vor einem halben Jahr tut sich da nichts, weil es nicht schneller geht.--2003:68:ED69:6100:81C2:5D8F:DC76:9910 11:25, 27. Sep. 2015 (CEST)PS Ich lese gerade. dass das KBA den VW bis 7.10 Frist gesetzt hat einen Durchführplan auf den Tisch zu legen. Darin wird dann stehen, wann die neue SW zur Verfügung steht und eine Rückrufaktion gestartet werden kann. Die Mogelsoftware soll von Bosch stammen, die aber dem Besteller darauf hingewiesen haben, dass die rechtlich bedenklich und nur für interne Testzwecke einsetzbar ist. Diese Information, wie auch ein Hinweis eines VW-Mitarbeiters sind angeblich in der VW-Hierarchie versandet.--2003:68:ED69:6100:81C2:5D8F:DC76:9910 14:08, 27. Sep. 2015 (CEST)[Beantworten]

Da geht mit Manchem die Phantasie durch. Hier erklärt ein Anwalt, welche Rechte Kunden haben. --Heletz (Diskussion) 18:14, 27. Sep. 2015 (CEST)[Beantworten]

Danke für die vielen Antworten! Generator (Diskussion) 15:05, 29. Sep. 2015 (CEST)[Beantworten]

Dieser Abschnitt kann archiviert werden. Generator (Diskussion) 15:05, 29. Sep. 2015 (CEST)

Blütenpflanzen, die in höheren Lagen wachsen

Alpenenzian

Warum sind deren Blüten meist weiß, gelb oder rot, und nur selten blau oder violett? WIr lagen vor Madagaskar (Diskussion) 22:53, 25. Sep. 2015 (CEST)[Beantworten]

Weil du nicht hoch genug nach oben gewandert bist. Vielleicht kommst du aus Madagaskar oder Deutschland, wo es mehr Hügel als Berge gibt. --85.6.169.237 23:02, 25. Sep. 2015 (CEST)[Beantworten]
Enzian wächst in mittleren Lagen. Ich fragte nach höheren Lagen. WIr lagen vor Madagaskar (Diskussion) 23:06, 25. Sep. 2015 (CEST)[Beantworten]
Was sind denn für dich höhere Lagen? --Schlesinger schreib! 23:09, 25. Sep. 2015 (CEST)[Beantworten]
so 2000 Meter und mehr. Da wächst noch gelber Enzian, aber blauen gibt es nicht mehr viel. Warum nicht? WIr lagen vor Madagaskar (Diskussion) 23:27, 25. Sep. 2015 (CEST)[Beantworten]
Ein Faktor ist auf jeden Fall evolutionär: Was für Bestäuber existieren und welche Farben können diese sehen? Bekannt sein könnte die Tatsache, dass auf den Galapagos-Inseln heimische Pflanzen ausschließlich gelbliche Blütenfarben besitzen wegen der einzig heimischen Galapagos-Holzbiene (vgl. Suchergebnisse: "galapagos gelbe blüten holzbiene" bei DuckDuckGo). --Speravir (Disk.) 01:09, 26. Sep. 2015 (CEST)[Beantworten]
Stimmt Deine These überhaupt? Ist die Verteilung denn anders als in sonstigen Lagen? Sind - ohne menschliches Zutun - denn blaue oder violette Blütenpflanzen sonst in der Relation tatsächlich zahlreicher? Blaue oder violette Blütenpflanzen sind glaube ich auch sonst eher die wenigeren. Bedenke auch, eine (sehr) niedrige Gesamtanzahl (hier von Arten) könnte statistische Daten oft falsch erscheinen lassen. -- WikiMax - 13:14, 26. Sep. 2015 (CEST)[Beantworten]
Ich hab' mal bei Aichele/Schwegler: Blumen der Alpen reingeschaut, wo praktischerweise nach Blütenfarben sortiert wird. Ohne jetzt die verschiedenen Blütenfarben nach Höhe sortiert zu haben, kann ich die Annahme des Fragestellers nicht bestätigen. Oberhalb 2000 m treten nach einem kurzen Durchfliegen fast alle Farben auf, allerdings werden die Pflanzen und meist damit auch die Blüten kleiner, daher fallen evtl. einige Farben auch weniger auf. --Diorit (Diskussion) 13:08, 27. Sep. 2015 (CEST)[Beantworten]
Danke an Benutzer:Speravir, das wäre eine Erklärung, auch an die anderen, die sich Gedanken gemacht und geantwortet haben, für ihren guten Willen. WIr lagen vor Madagaskar (Diskussion) 19:47, 29. Sep. 2015 (CEST)[Beantworten]

Geblitzt worden - Fahrer unklar

Angenommen, ein Privat-Pkw wird von mehreren Personen gefahren. Der Halter erhält eine Anhörungsbogen wegen einer Geschwindigkeitsüberschreitng. Als Beweismittel wird ein Frontfoto erwähnt, das allerdings nicht mitgeschickt wurde.

Muss der Halter das Schreiben beantworten und seine Personalien angeben, auch wenn diese im Briefkopf ja schon alle angegeben sind?

Für den Fall, dass man weiß, wer der Fahrer war, ist es dann vorteilhaft (kostengünstiger, zeitsparender o.ä.), diesen direkt anzugeben?

Wenn man nicht weiß, wer der Fahrer war, und man keine Angaben zur Sache macht, wie wird dann weiter vorgegangen, um ihn zu identifizieren? Und kann man ohne Anwalt das Blitzerfoto einsehen, um selbst die Identifikation vorzunehmen? (nicht signierter Beitrag von 94.218.120.104 (Diskussion) 23:43, 25. Sep. 2015 (CEST))[Beantworten]

Wenn der Halter keine Angaben macht, und der Fahrer nicht ermittelt werden kann, wird das Verfahren in der Regel eingestellt. Je nachdem, was für eine Sanktion damit nicht verhängt werden kann ist es möglich das dann das Führen eines Fahrtenbuches angeordnet wird. [8] --Oliver S.Y. (Diskussion) 23:49, 25. Sep. 2015 (CEST)[Beantworten]
Danke, ja. Wie wird denn versucht, ihn zu identifizieren? Bekommt man das Foto zugeschickt? Kommen Polizisten und befragen die Nachbarn? Und ist man vor einem Fahrtenbuch sicher, wenn der Anhörungsbogen erst mehr als zwei Wochen nach dem Blitzen erstellt und verschickt wurde?--94.218.120.104 23:59, 25. Sep. 2015 (CEST)[Beantworten]
Abgleich mit dem Foto auf dem Personalausweis zum Beispiel, in neuerer Zeit auch Abgleich mit Facebook-Profilfotos. Nachbarn werden in der Regel dann befragt, wenn man ausdrücklich das Zeugnisverweigerungsrecht als Grund für die Nichtaussage angibt, da dann der Fahrer in der näheren Familie vermutet wird.
Vor einer Fahrtenbuchauflage ist man, wenn ich mich recht entsinne, erst nach sechs Wochen sicher. Bis zu sechs Wochen soll man sich zurückerinnern können. -- Liliana 00:07, 26. Sep. 2015 (CEST)[Beantworten]
Ah, danke für die Infos :-) Ist eine Fahrtenbuchauflage auch dann möglich, wenn man mithilft, indem man zumindest eine vollständige Auswahl der möglichen Fahrer angibt? Es kann ja vorkommen, dass mehrere davon sich ähnlich sehen, aber keiner mehr genau weiß, ob oder wann er mit dem Tat-Auto gefahren ist. Dann schränkt man zumindest den Täterkreis ein. Und als weitere Frage: Wie geht die Polizei vor, wenn der Täter dann anhand des Foto nicht exakt ermittelbar ist und ggf. beide mögliche Fahrer (als Verwandte) vom Zeugnisverweigerungsrecht Gebrauch machen? Riskiert man dann ein teures Gerichtsverfahren mit Gutachtern? --94.218.120.104 00:26, 26. Sep. 2015 (CEST)[Beantworten]
Solange der genaue Fahrer nicht ermittelt wird, ja. Und ein Gutachten wird man wegen einer einfachen Geschwindigkeitsübertretung sicher nicht anfertigen, anders sähe es aus, wenn es um Straftaten ginge wie Fahrerflucht oder fahrlässige Körperverletzung/Tötung. -- Liliana 00:29, 26. Sep. 2015 (CEST)[Beantworten]
Worauf ist denn das erste Ja bezogen? Ist eine eine Fahrtenbuchauflage trotz Personenkreis-Angabe möglich? Und/Oder ist droht ein Gerichtsverfahren mit zusätzlichen Kosten, wenn der Fahrer nicht angegeben wird (angenommen, es geht dabei lediglich um ca. 100 Euro Busgeld, einen Punkt und kein Fahrverbot)? PS: Ich habe bisher nur von einer Zwei-Wochen-Frist bzgl. Erinnerung bzgl. Fahrer gelesen. --94.218.120.104 00:45, 26. Sep. 2015 (CEST)[Beantworten]
Ersteres. Ein Gerichtsverfahren gibt es im Ordnungswidrigkeitenverfahren nicht bzw. erst dann, wenn erfolglos Widerspruch gegen einen Bußgeldbescheid eingelegt wird. -- Liliana 00:51, 26. Sep. 2015 (CEST)[Beantworten]
Verstanden. Das heißt, eine Fahrtenbuch-Auflage wäre möglich. Einer Schätzung meinem derzeitigen Wissensstand nach wäre sie allerdings eher unwahrscheinlich (bei mehr als zwei Wochen zwischen Tat und Anhörungsbescheid und zumindest Mithilfe-Versuch über Personenkreis-Angabe).
Ist es aussichtsreich, nach dem Beweisfoto zu fragen, um den Täter ermitteln zu können? Sollte die Foto-Einsicht bzw. die Identifikation damit nicht möglich sein, wie ginge es dann weiter? Angenommen, mehrere Personen sehen der Person auf dem Beweisfoto ähnlich, weder die Personen selbst, der Halter oder Nachbarn können den Fahrer identifizieren (und niemand kann sich mehr an die Fahrt erinnern)? Ein Bußgeldbescheid kann so ja nicht ausgestellt werden. Wird das Verfahren dann ohne Folgen (abgesehen von evtl. Fahrtenbuch) eingestellt? Und ohne Bußgeldbescheid oder Anhörungsbogen an die mögichen Fahrer verjährt der Vorfall ohnehin nach drei Monaten. --94.218.120.104 01:07, 26. Sep. 2015 (CEST)[Beantworten]
Man kann es versuchen, es kostet nichts. Wenn die Anfrage abgelehnt wird, spricht das auch gegen eine Fahrtenbuchauflage. Wenn der Fahrer nicht festgestellt werden kann, wird das Verfahren eingestellt. -- Liliana 01:49, 26. Sep. 2015 (CEST)[Beantworten]

(BK)Siehe auch Fahrtenbuch#Auflage nach § 31a StVZO. Falls der Fahrzeughalter den Fahrer kennt, ist es natürlich „kostengünstiger, zeitsparender o.ä, diesen direkt anzugeben“. Akteneinsicht („Blitzerfoto“) ist unter Aufsicht auch ohne Anwalt möglich, OWiG/49. --Vsop (Diskussion) 00:17, 26. Sep. 2015 (CEST)[Beantworten]

Dankeschön! Kommen denn höhere Kosten auf den Betroffenen zu, wenn er einen Kreis möglicher Fahrer angibt, aber keinen Tat-Fahrer benennt (benennen kann)?--94.218.120.104 00:26, 26. Sep. 2015 (CEST)[Beantworten]
dummes Gesülze, wenn Du oder einer Deiner Bekannten zu schnell gefahren ist, dann zahl gefälligst! Fahr halt im Rahmen des Erlaubten und heul hier nicht rum!! --Joschi71 (Diskussion) 02:44, 26. Sep. 2015 (CEST)[Beantworten]
  • In der Schweiz wäre der Fall klar, genau deswegen zahlt seit der Gesezesänderung der Halter. Keine Angabe zum Fahrer = Busse bleibt an Halter hängen. War er es nicht, und gibt er den Fahrer an, ist er fein raus. Es wird glaube Zeit das auch in Deutschland einzuführen. --Bobo11 (Diskussion) 03:58, 26. Sep. 2015 (CEST)[Beantworten]
Klar, bei Mord verknackt man einfach den letzten, der die Person lebend gesehen hat. Noch mehr so idiotische Vorschläge? --Tommes  23:40, 26. Sep. 2015 (CEST)[Beantworten]
Wichtig ist zu wissen, daß in Deutschland der Halter niemand belasten muß, also auch nicht die möglichen Personen anzugeben braucht. --Heletz (Diskussion) 07:34, 26. Sep. 2015 (CEST)[Beantworten]
In Deutschland hat der Halter genau zwei Möglichkeiten: Petze oder Märtyrer. Entweder er verrät seinen Mitbenutzer und hat kein Problem mit der Bußgeldstelle, aber mit seinem verpetzten Mitfahrer, oder er zahlt das Bußgeld märtyrerhaft selbst, um es sich mit seinem Mitbenutzer nicht zu verscherzen. --Rôtkæppchen₆₈ 12:51, 26. Sep. 2015 (CEST)[Beantworten]
Naja, eigentlich doch. Wenn man gar nichts sagt, kann man für bis zu sechs Wochen in Beugehaft gesteckt werden, § 46 Abs. 5 OWiG. -- Liliana 12:59, 26. Sep. 2015 (CEST)[Beantworten]

(BK)@Benutzer:Heletz: Dass der Halter „niemand belasten muß“, gilt nur, solange er ein (möglicherweise) Beschuldigter ist, StPO/136. Ist er das nicht mehr, hat er als Zeuge „die Pflicht auszusagen, wenn keine im Gesetz zugelassene Ausnahme vorliegt“, StPO/48. --Vsop (Diskussion) 13:11, 26.

Das gilt aber nur vor Gericht! Vor der Polzei muß er gar nichts sagen! --Heletz (Diskussion) 13:15, 26. Sep. 2015 (CEST)Sep. 2015 (CEST)[Beantworten]
Ist die StPO im OWi-Verfahren überhaupt relevant? --Rôtkæppchen₆₈ 13:21, 26. Sep. 2015 (CEST)[Beantworten]
Ja, die Regeln der StPO gelten nach § 46 OWiG auch im OWi-Verfahren, mit einigen Ausnahmen (in eine psychiatrische Klinik etwa kann man wegen einer OWi nicht eingewiesen werden). -- Liliana 13:24, 26. Sep. 2015 (CEST)[Beantworten]
Aber auch als Zeuge müsste man sich doch nicht selbst belasten, oder? Und wenn man es für möglich hält, dass man später z.B. eines Mordes (den man natürlich nicht begangen hat) beschuldigt wird, könnte die Belastung eines anderen das Alibi des zukünftig möglicherweise mordbeschuldigten Zeugen beeinträchtigen… Ändert natürlich nichts daran, dass ich als Fahrzeughalter den schuldigen Fahrer nicht mehr fahren lassen würde. --nenntmichruhigip (Diskussion) 16:06, 26. Sep. 2015 (CEST)[Beantworten]
Und das sagt der Anwalt zu der beliebten Polizei-Masche "Als Zeuge müssen Sie jetzt aussagen!"--Heletz (Diskussion) 16:10, 26. Sep. 2015 (CEST)[Beantworten]

Können auch Minderjährige als Halter eingetragen sein? --Tommes  23:40, 26. Sep. 2015 (CEST)[Beantworten]

Anscheinend nein: "Ein Fahrzeug darf auf Minderjährige ohne eine Behinderung nur dann zugelassen werden, wenn die minderjährige Person für dieses Fahrzeug eine entsprechende Fahrerlaubnis besitzt." -- Liliana 00:03, 27. Sep. 2015 (CEST)[Beantworten]
Q? --nenntmichruhigip (Diskussion) 00:59, 27. Sep. 2015 (CEST)[Beantworten]
Das ist inzwischen allgemein übliche Praxis; begründet wird es in der Regel damit, dass aus der beschränkten oder gar fehlenden Geschäftsfähigkeit in Verbindung mit der fehlenden Fahrerlaubnis das Fehlen der Verfügungsgewalt über das Fahrzeug abgeleitet werden könne. Damit aber fehlt es an der Haltereigenschaft, die ja gerade eingetragen werden soll. Diese Konstruktion der Zulassungsämter wurde nötig, nachdem immer mehr Eltern dazu übergegangen waren, ihre Fahrzeuge auf ihre Kinder zuzulassen - um genau die eingangs beschriebene günstige Situation im Falle von Verkehrsverstößen herbeizuführen. --Snevern 23:36, 27. Sep. 2015 (CEST)[Beantworten]

26. September 2015

Latein(isch)

Ich komm nicht drauf. Was steht auf den Steinen?

Danke. --Tommes  05:05, 26. Sep. 2015 (CEST)[Beantworten]

Auf dem ersten: "Futura Facies" (das künftige Gesicht, Aussehen), auf dem zweiten "Vnima (?) Similitudo" (einheitliche Ähnlichkeit). Scheinen Hinweise auf den Tod zu sein. --Heletz (Diskussion) 07:24, 26. Sep. 2015 (CEST)[Beantworten]
Ultima Similitudo? --King Rk (Diskussion) 07:53, 26. Sep. 2015 (CEST)[Beantworten]
+1. Ja, unbedingt. Grüße Dumbox (Diskussion) 07:57, 26. Sep. 2015 (CEST)[Beantworten]
Ok, es könnte auch "Vltima" heißen, die Buchstaben sind schon sehr schadhaft. Also "die letzte, äußerste Ähnlichkeit". --Heletz (Diskussion) 08:06, 26. Sep. 2015 (CEST)[Beantworten]
Also Das zukünftige Gesicht und die äußerste Ähnlichkeit. Danke. Ich las immer "Vitima.." Hat das auf den Säulen des Tores eines Kirchhofes irgendeine besondere Bedeutung, außer der, die man sich dazu laienhaft zusammenreimt? --Tommes  22:58, 26. Sep. 2015 (CEST)[Beantworten]
Ich denke nicht. Hier (pdf) lese ich, dass da auch noch fui quod es - eris quod sum steht/stand: also das geläufige Memento-mori-Motiv. Grüße Dumbox (Diskussion) 23:10, 26. Sep. 2015 (CEST)[Beantworten]
Im verlinkten PDF, S. 98, steht allerdings, die Pforte in Hainewalde sei mit steinernen Totenköpfen geschmückt, womit die Inschrift stimmig würde. --Pp.paul.4 (Diskussion) 15:59, 28. Sep. 2015 (CEST)[Beantworten]

Kann man Primasprit 96% (natürlich unvergällt) unverdünnt in kleinsten Mengen trinken? --85.180.133.176 05:51, 26. Sep. 2015 (CEST)[Beantworten]

Bei der Bw hieß es bei solchen Fragen dann: "Ich weiß nicht, ob Sie können!" --Heletz (Diskussion) 07:18, 26. Sep. 2015 (CEST)[Beantworten]
Chuck Norris trinkt sowas als Muttermilchersatz. scnr -- Iwesb (Diskussion) 07:35, 26. Sep. 2015 (CEST) [Beantworten]
im Artikel steht, dass man ihn zu Wodka und sowas weiterverarbeiten kann... also richtig schlimm giftig scheint das Zeug nich zu sein... obwohl unser Physiklehrer immer meinte, dass auch Ethylalkohol Gift ist, weil er desinfiziert... vllt zerstört das Zeug erstmal die Schleimhäute bis es genug verdünnt ist... --Heimschützenzentrum (?) 08:28, 26. Sep. 2015 (CEST)[Beantworten]
Vergällte Alkohole werden mit einem Azeotrop erzeugt. Die Azeotropverschleppung per Destillation ist im kleinen Maßstab aufwändiger und teurer als die Steuerzu entrichten. Grüße, --Ghilt (Diskussion) 08:39, 26. Sep. 2015 (CEST)[Beantworten]
Ja natürlich kannst du den trinken. Man kann alles trinken, was flüssig ist. Ob dir das schadet, ist eine andere Frage, die davon abhängt, wie klein für dich die "kleinste" Menge ist. Geoz (Diskussion) 11:59, 26. Sep. 2015 (CEST)[Beantworten]
War eigentlich auch das Erste, was mir durch den Kopf ging: man kann alles trinken, was flüssig ist. Aber: nur Chuck Norris gurgelt mit flüssigem Stahl. Ein bisschen ist es auch abhängig von der Temperatur. meint -- Iwesb (Diskussion) 12:05, 26. Sep. 2015 (CEST) [Beantworten]
Er hier hat mit flüssigem Gold gegurgelt. Ist ihm wohl nicht gut bekommen, aber nach Bekömmlichkeit wurde ja nicht gefragt ;-) Geoz (Diskussion) 15:07, 26. Sep. 2015 (CEST)[Beantworten]
Aus Erfahrung: ja ;) Gab es durchaus schon auf Partys und auch in örtlichen Studentenkneipen als Shots. Ob es angenehm und lecker ist muss aber wohl jeder selbst entscheiden. Es ist auch "nur" etwa das doppelte bis dreifache eines Wodkas und da stirbt man auch nicht nach zwei Kurzen. Diverse Rumsorten, die man in Bars (als Shot) bekommt, erreichen auch mal 70 Prozent (Strohrum zum Beispiel hat zwar mehr, wird aber seltener derart Verkauft). Absinth durchaus mehr (77%igen sah ich häufiger - da ist die Desinfektionswirkung interessanterweise sogar größer als bei 96%igem). Da ist der Unterschied schon nicht mehr so groß. --StYxXx 15:07, 27. Sep. 2015 (CEST)[Beantworten]

Silbermünzen als Zahlungsmittel - Theorie und Praxis

In Österreich ausgegebene 5 € und 10 € Silber- und Kupfermünzen sind in Österreich auch offizielles Zahlungsmittel, und theoretisch ist ihre Annahme verpflichtend - aber die meisten VerkäuferInnen kennen diese Münzen und ihren Status gar nicht und nehmen sie daher nicht. (Vor ein paar Tagen habe ich die Rauchfangkehrer gefragt, ob sie für die Bezahlung des "Pickerls" auch Silbermünzen nehmen und zur Antwort erhalten "Nein, nur Gold - kiloweise!") Wie sieht das juristisch aus? Machen sich Händler, die die Annahme verweigern, irgendwie strafbar? --KnightMove (Diskussion) 11:21, 26. Sep. 2015 (CEST)[Beantworten]

Es besteht zwar eine Pflicht zur Annahme als Zahlungsmittel, aber keine Pflicht, überhaupt ein Kaufgeschäft abzuwickeln. Geht also lediglich um die Formulierung, welche die Verkäuferin als Grund nennt, und da es sich um VerkäuferInnen und nicht RechtsanwältInnen handelt, ist es angesichts des Streitwerts schlicht unmöglich, das Recht durchzusetzen.Oliver S.Y. (Diskussion) 11:33, 26. Sep. 2015 (CEST) PS - Probleme mit solchen Sondermünzen haben eher Automatenaufsteller bzw. Betreiber, die eine A-Lösung finden müssen.Oliver S.Y. (Diskussion) 11:34, 26. Sep. 2015 (CEST)[Beantworten]
Der Münzbesitzer ist doch selbst schuld, wenn er eine Sondermünze zum Nominalwert ausgeben will. Üblicherweise liegt der Sammlerwert einer solchen Münze über dem Nominalwert, sodass sich die Frage nach Automatenumstellung gar nicht stellt, da auf den Automaten immer draufsteht, welche Münzen er akzeptiert. Die alten Fahrkartenautomaten der Stuttgarter Straßenbahnen hatten z.B. keinen Schacht für 20-Cent-Münzen, weswegen man dort nicht mit 20-Cent-Münzen zahlen konnte. Die SSB war stillschweigend davon ausgegangen, dass sich die 20-Cent-Münzen nicht durchsetzen, weil es in Westdeutschland auch nie eine 20-Pfennig-Münze gab. Später hat die SSB ihre Automaten erneuert und in den neuen Automaten auch einen 20-Cent-Schacht vorgesehen. --Rôtkæppchen₆₈ 12:46, 26. Sep. 2015 (CEST)[Beantworten]
Der Streit über die Annahme eines bestimmten gesetzlichen Zahlungsmittels (z.B. großer Geldschein oder Sondermünze) kann ja durchaus nach Abschluss eines wirksamen Vertrages erst entstehen: Ich muss ja beim Abschluss des Kaufvertrages nicht nachweisen, dass ich den Kaufpreis auch zahlen kann bzw. nur mit einem dem Verkäufer genehmen Geldstück/-schein. Und wenn sich nach Abschluss des Kaufvertrages herausstellt, dass ich es nicht kann, wird der Vertrag dadurch nicht unwirksam (auch wenn in der Praxis natürlich der Verkäufer die Waren einfach wieder ins Regal legt): Ich habe weiterhin Anspruch auf den Kaufgegenstand; der Vertrag löst sich nicht von allein in Rauch auf, nur weil dem Verkäufer meine Münze nicht gefällt.
Ich weiß ehrlich gesagt nicht, wie's in Österreich ist (und bin im Moment zu faul, es nachzuprüfen), aber in Deutschland könnte ein Rechtsstreit auch um 5 Euro geführt werden - eine Mindestklagesumme oder eine Geringfügigkeitsgrenze als Klagevoraussetzung gibt es nicht. --Snevern 23:17, 27. Sep. 2015 (CEST)[Beantworten]
Nur so ein Gedanke, das öst. Nationalbankgesetz regelt nur die verpflichtende Annahme von Banknoten. Oliver S.Y. (Diskussion) 23:36, 27. Sep. 2015 (CEST)[Beantworten]
Der Artikel beruft sich ja auch nicht auf das Nationalbankgesetz, sondern das Scheidemünzengesetz, § 8. --KnightMove (Diskussion) 13:53, 29. Sep. 2015 (CEST)[Beantworten]

Grammatikfrage: spielen mit...?

Hallo zusammen. Zufällig bin ich über diesen Edit gestolpert. Der Edit selbst ist mir relativ schnurz, aber mein eigentlich ganz gutes Sprachgefühl sagt, dass die vorherige Version richtig sei. Überlegen führt auch zu "Sie spielt mit weM?" --> "Mit eineM jungeN Fuchs", allerdings auch zu "Mit jungeM Fuchs". Was ist denn nun richtig, wenn man sowohl "ein" als auch "Fuchs" weglässt und "junge" substantiviert? Kann mich jemand aufklären? Danke :-) Gruß, --Benutzer:Apierta 17:41, 26. Sep. 2015 (CEST)[Beantworten]

(a) "...spielt mit Jungem" und (b) "...spielt mit Jungen": Sprachlich korrekt ist beides, je nachdem, ob Einzahl oder Mehrzahl. Auf dem Bild sieht man nur das Alttier mit einem Jungen, also trifft hier (a) zu. -- Geaster (Diskussion) 17:49, 26. Sep. 2015 (CEST)[Beantworten]
Wenn im Dativ zwei Adjektive aufeinander folgen ("einem" + "kleinem"), dann wird beim zweiten aus dem "m" ein "n", daher "einem kleinen Fuchs". 90.184.23.200 18:13, 26. Sep. 2015 (CEST)[Beantworten]
"Jungem" als Dativ des Substantivs "Junges" ist richtig. Ebenfalls richtig ist "spielt mit dem Jungen".
Etwas anderes ist der Dativ Singular von "Junge"; der lautet "einem Jungen". Die oben angedachte Substantivierung des Adjektivs "jung" führt mMn zu dem Substantiv "der Junge"; das ist nicht das, was im Artikel gemeint ist. Im Fall "das Junge" liegt dagegen eine Konversion vor.
Wieder etwas anderes ist das Adjektiv "jung". Da heißt der Dativ Singular mit bestimmtem Artikel "einem jungen Mann", ohne Artikel dagegen "jungem Mann". Beides ist hier nicht gefragt. --88.130.81.45 19:04, 26. Sep. 2015 (CEST)[Beantworten]
Natürlich ist es eine Substantivierung, nur eben im Neutrum: ein junges Tier -> ein Junges. Substantivierte Adjektive werden in der Regel wie attributive Adjektive gebeugt (Duden Grammatik (1966), 2240f), und das heißt für den Dativ: mit jungem Kind, mit Jungem (ohne Artikel, stark gebeugt), aber mit einem/dem jungen Kind, mit einem/dem Jungen (schwach). Grüße Dumbox (Diskussion) 20:02, 26. Sep. 2015 (CEST)[Beantworten]

Uff. Ich sag mal danke, auch wenn ich mir noch nicht wirklich sicher bin, ob ich wirklich mehr als "ja, die Änderung im Artikel ist richtig" verstanden habe. Gruß, --Benutzer:Apierta 15:27, 27. Sep. 2015 (CEST)[Beantworten]

Kyrillischer Schriftzug

Kann jemand des Russischen Mächtiges lesen, was da geschrieben steht? Gruß -- ♦ The Great Zaganza ♦ 18:02, 26. Sep. 2015 (CEST)[Beantworten]

Mal ein erster Anfang. Es ist Ukrainisch. Ich kann leider kein Ukrainisch, nur ein bisschen Russisch: ~TORGOVYJ DOM / KNJAS JURIJ TRUWEPKOJ I KO? ?? KIEW. Es war aber nicht leicht zu lesen für mich. Irgendein Haus (= dom), vielleicht Torgauer Haus? / Dann irgendetwas das einem Juri Truwepkoj (könnte auch Trubepkoj sein, kann man nicht gut erkennen) gehört. Was die Zeichen am ende heissen weiss ich nicht. 90.184.23.200 18:11, 26. Sep. 2015 (CEST)[Beantworten]
Sowas wie Handelsgesellschaft Prinz Jurij Trubezkoi & Co. Siehe Trubezkoi (Adelsgeschlecht), aber von Adelsgeschichte und so'm Zeug hab ich null Ahnung, sorry. -ZT (Diskussion) 20:32, 26. Sep. 2015 (CEST)[Beantworten]
Natürlich ist das Russisch und kein Ukrainisch. Allerdings handelt es sich um das Russisch vor der Russischen Rechtschreibreform von 1918, deshalb das ungewohnte Schriftbild. Übersetzt lautet die Inschrift "Handelshaus Fürst Juri Trubezkoi & Co Kiew". --91.177.187.138 00:36, 27. Sep. 2015 (CEST)[Beantworten]
"торговый домъ / князь Юрій Трубецкой и ко Кіевъ" ist aber kein reines oder heutiges ukrainisches Alphabet. Das Zeichen ъ gibt es im Ukrainischen (heute) nicht. Und ъ gehört nicht an дом. Und Кіев heißt auf ukrainisch zudem Київ. Seltsam, findet --Tommes  00:40, 27. Sep. 2015 (CEST)[Beantworten]
Wirst Du hier («Klick») nicht fündig? -Tommes  00:42, 27. Sep. 2015 (CEST)[Beantworten]
wie schon oben geschrieben, handelt es sich um reines Russisch. Auf Ukrainisch müsste die Inschrift "Торговий дім князь Юрій Трубецькой і ко." heissen. Allerdings hat zur damaligen Zeit kein einigermassen gebildeter Mensch Ukrainisch geschrieben oder gar im Geschäftsverkehr verwendet. Das Zeichen ъ, früher der Buchstabe "jer", heute als hartes Zeichen bezeichnet, stand ursprünglich hinter hart gesprochenen Konsonanten. Die Verwendung wurde mit der Rechtschreibreform jedoch stark eingeschränkt.--91.177.187.138 00:50, 27. Sep. 2015 (CEST)[Beantworten]
Hast Recht, Deine Antwort kam mir irgendwie zuvor. Ist das Wort vor "Кіевx" ein "въ", also "im" mit diesem jer/harten Zeichen? Kannst Du das letzte Zeichen hinter "Кіев" benennen? Muß es nicht ein "e" sein (6. Fall - "in Kiew"/"в Кіеве"), statt eben dieses "twjorduj snak" (So kenne ich es)? --Tommes  01:14, 27. Sep. 2015 (CEST)[Beantworten]
Bei dem letzten Buchstaben im Wort Kiew handelt es sich um ein ѣ. Dieser Buchstabe wurde durch das е ersetzt. Ursprünglich wurde das ѣ ganz anders ausgesprochen, war es aber zuletzt in der Aussprache nicht mehr vom е zu unterscheiden. --91.177.187.138 01:26, 27. Sep. 2015 (CEST)[Beantworten]
Auf Ukrainisch wäre es eher торгівельний дім, oder gar торгівельний будинок. Was das Jat angeht - die ursprüngliche Aussprache war wohl dem englischen offenen a ähnlich. Der Laut verschwand aber schon irgendwo im frühen Mittelalter, der Buchstabe blieb wundersam bis 1918 erhalten.--Alexmagnus Fragen? 16:35, 27. Sep. 2015 (CEST)[Beantworten]

Danke allen Beteiligten – Ich hätte vielleicht gleich sagen sollen, dass es sich wohl um einen Schriftzug aus der Zeit um 1900 handelt. Jedenfalls bleibt die auch von dem Auktionshaus kolportierte Anekdote, die Pistole sei in größerer Stückzahl von den russischen Sozialdemokraten für die Revolution von 1905 erworben worden, eher nicht halt- bzw. belegbar. Gruß ♦ The Great Zaganza ♦ 11:46, 28. Sep. 2015 (CEST)[Beantworten]

27. September 2015

Ponyhaltung Schweiz - Tierschutzverordnung?

Ich habe in einer Gemeinde 2 Ponys gesehen, die auf einem Schotterplatz gehalten werden, den ich subjektiv für zu klein halte. Unter dem Schotterplatz befindet sich ein Gitter (sichtbar), die Fläche ist schwer messbar, aber wohl ca. um die 20m2 inkl. Unterstandsboxen. Brauchen Ponys keinen Rasen? Reicht die Fläche? https://www.admin.ch/opc/de/classified-compilation/20080796/index.html

http://www.tierschutz.com/publikationen/pferde/infothek/mb_pferdesport02.pdf (Verbindlichkeit fraglich.) --85.4.233.141 00:17, 27. Sep. 2015 (CEST)[Beantworten]

Niemand von uns kennt die Umgebung dort geschweige oder wie die Tiere sonst gehalten werden. Ein Schotterplatz dürfte wohl ok sein, solange ausreichend nahrung zur verfügung steht und der Schotter genügend verfestigt ist, sodass keine Verletzungsgefahr für Mensch ung Tier besteht. Um was für Gitter handelt es sich denn? Eventuell solch Gitterförmige Steinplatten?. 20qm klingt klein, sicher dass sie dort dauerhaft und ausschließlich gehalten werden und es nicht vielleicht nur der Hof ist und die Tiere mehr ausgeritten werden? Ich mein es gibt Pferde die mit Ausnahme vom Ausritt, ihr Leben in Boxen verbringen, die weit unter 10qm haben und viele Nutzpferde (Kutschen) kriegen auch eher selten Rasen unter die Hufe, eventuell haben die beiden Ponys sogar in Anführungsstrichen ein Luxusleben. --178.2.78.6 01:33, 27. Sep. 2015 (CEST)[Beantworten]
Das Gitter besteht aus Maschendraht, darüber wurde Schotter (bzw. eher grösseres Kies) verstreut, man sieht die Hexagonstruktur vom Maschendraht an mehreren Stellen. Ich nehme an, dass die Tiere einmal im Tag gefüttert werden. Nahrung, Heu oder Gras ist im Gehege jedoch nicht vorhanden. Es handelt sich um ein Altersheim, die Ponys werden dort dauerhaft gehalten. Allerdings ist das Altersheim momentan im Umbau und den Tieren fehlt menschlicher Kontakt. Daneben ist ein Geisen/Hasengehege angelegt (dieses scheint genügend gross). Vielleicht kann nächste Woche einige Fotos nachreichen.--85.4.233.141 11:08, 27. Sep. 2015 (CEST)[Beantworten]
"Den Tieren fehlt menschlicher Kontakt" - haben sie dir das gesagt? Menschlicher Kontakt ist widernatürlich, Tiere wollen Kontakt mit ihresgleichen. --83.79.9.134 11:24, 27. Sep. 2015 (CEST)[Beantworten]
Der Mensch ist auch Teil der Natur. Die Paarhaltung ist erfüllt, die Beschäftigungsmöglichkeit in einer Schotterwüste hingegen wohl stark eingeschränkt.--85.4.233.141 13:22, 27. Sep. 2015 (CEST)[Beantworten]

Wenn du klüger bist als der Tierhalter, dann bring seine Ponys zum Metzger und sie werden erlöst. --83.79.9.134 10:53, 27. Sep. 2015 (CEST)[Beantworten]

Ist nicht Thema. Es gibt gesetzliche Grundlagen für die Tierhaltung.--85.4.233.141 11:08, 27. Sep. 2015 (CEST)[Beantworten]
Kannst dich auch beim Veterinäramt des betreffenden Kantons melden, oder auf dem nächsten Polizeiposten. Das hat mal eine Nachbarin gemacht weil sie das Gefühl hatte, unsere Katze hätte zuviel Auslauf. Man bekommt dort auch gesagt, dass man total ernst genommen wird und die Behörden bedanken sich freundlich für den Hinweis. --83.79.9.134 11:16, 27. Sep. 2015 (CEST)[Beantworten]
In D gibts Vorschriften bezüglich der Größe von Pferdeboxen. Die sind aber bei weitem nicht so groß wie die oben genannte Fläche. Vom Gesetz her hat ein Pferd im Gegensatz zu einem menschlichen Häftling keinen Anspruch auf täglichen Hofgang oder sowas, genauso wie es Rindvieh gibt, das zeitlebens angekettet ist. Pferde brauchen übrigens nicht zwingend einen Unterstand, die können auch im Regen und im Schnee draußen bleiben, sie kriegen dann ein dichteres Winterfell, was aber nicht wächst, wenn sie meistens im warmen Stall bleiben.--Giftzwerg 88 (Diskussion) 00:47, 28. Sep. 2015 (CEST)[Beantworten]

Wollte Marx das Geld abschaffen?

In einer Diskussion über "Geld abschaffen" wurde der Name von Karl Marx eingeworfen, als ob diesem selbiges vorgeschwebt wäre. Da ich weder Das Kapital gelesen habe noch auf die Schnelle aus Wikipedia schlau werde... frage ich einfach mal hier nach fundierten Antworten. --Quetsch mich aus, ... itu (Disk) 10:19, 27. Sep. 2015 (CEST)[Beantworten]

Zumindest wurde uns das so in der Schule erzählt: Im Kommunismus gibt es kein Geld mehr. --Pölkkyposkisolisti 10:37, 27. Sep. 2015 (CEST)[Beantworten]
Selbst das klingt nach einer wenig zutreffenden Verallgemeinerung, obwohl die schnell nachlesbare Tatsache dass Marx _auch_ Kommunist gewesen sein soll und nicht nur Sozialist, der Anknüpfungspunkt für die Beantwortung der Frage sein sollte. --Quetsch mich aus, ... itu (Disk) 11:26, 27. Sep. 2015 (CEST)[Beantworten]
Also Pölky, Deine Schulzeit ist ja doch etwas länger vorbei, und diese Frage war da sicher nicht in dieser Form gestellt worden. Unter [9] wird es ganz gut dargestellt.

"Und solange der „Tauschwert die gesellschaftliche Form der Produkte bleibt“, (MEW 42: 80) kann das Geld nicht aufgehoben werden. Marx warnt hier ausdrücklich: „Es ist nötig, dies klar einzusehn, um sich keine unmöglichen Aufgaben zu stellen und die Grenzen zu kennen, innerhalb deren Geldreformen und Zirkulationsumwandlungen die Produktionsverhältnisse und die auf ihnen ruhenden gesellschaftlichen Verhältnisse neugestalten können.“ (MEW 42: 80)" - Marx hat im Gegensatz zu anderen linken Philosophen den Weg zum Kommunismus als viel umfangreichere Entwicklung verstanden und definiert, und halt auch vor solchen Auswirkungen gewarnt. Im Übrigen gibt es einen Unterschied zwischen dem Wegfall von Zahlungsmitteln durch fehlende Bedeutung, und der zwangsweisen Abschaffung ohne Äquivalent, wie es manche Experimente versucht haben. Sie kann nur Teil einer gesellschaftlichen und persönlichen Entwicklung sein, wo die Gesellschaft die Bedürfnisse des Einzelnen umfassend befriedigt, aber gleichzeitig der Einzelne seine Bedürfnisse entsprechend den Möglichkeiten anpasst. "Die Familie ist die natürliche Kernzelle der Gesellschaft und hat Anspruch auf Schutz durch Gesellschaft und Staat." stammt übrigens nicht von Marx, sondern aus dem UNO-Pakt II. Marx schrieb da wohl ebenso was von kleinster Zelle der Gesellschaft. Wenn solche soziale Basis fehlt, kann man nicht andere Aspekte des Kommunismus als Einzelpaket durchsetezen. Im Übrigen ist die Abschaffung von "Kapital", also Geldreserven etwas anderes als die Abschaffung von Geld als Tauschwertersatz.Oliver S.Y. (Diskussion) 11:30, 27. Sep. 2015 (CEST)[Beantworten]

Er wollte nicht das Geld abschaffen, sondern eine kommunistische Gesellschaft einführen. Dass Geld danach, vielleicht, einmal unwichtig werden könnte und nicht mehr gedruckt wird, wäre eine Folge gewesen. --83.79.9.134 11:36, 27. Sep. 2015 (CEST)[Beantworten]

Zunächst ist unklar, was genau in der Frage mit "dem Geld" und mit "abschaffen" gemeint ist.

Die diversen Funktionen des Geldes als Schatz, Zahlungsmittel, Weltgeld werden im Kapital, Kapitel 3.3 diskutiert [10].

Wie und aus welchem Grund im Kapitalismus das Geld auf einer abstrakteren und grundlegenderen Ebene als besondere Äquivalentform von allen anderen Waren ausgeschlossen wird ist Gegenstand des ersten Kapitels des Kapital.

In der zitierten Passge aus den Grundrisse der Kritik der politischen Ökonomie spricht Marx über "Geldreformen und Zirkulationsumwandlungen", womit m.E. keine "Abschaffung" von Geld gemeint ist.

Was für die Situation einer Übergangsphase für notwendig hielt schildert er in der Kritik des Gothaer Programms. Demnach sollte jeder Arbeiter anstelle von "Geld" für die geleisteten Arbeitsstunden einen Schein erhalten, mit dem er Konsumtionsmittel erwerben kann, die mit dem gleichen Arbeitsquantum hergestellt worden sind:

"Womit wir es hier zu tun haben, ist eine kommunistische Gesellschaft, nicht wie sie sich auf ihrer eignen Grundlage entwickelt hat, sondern umgekehrt, wie sie eben aus der kapitalistischen Gesellschaft hervorgeht, also in jeder Beziehung, ökonomisch, sittlich, geistig, noch behaftet ist mit den Muttermalen der alten Gesellschaft, aus deren Schoß sie herkommt. Demgemäß erhält der einzelne Produzent - nach den Abzügen - exakt zurück, was er ihr gibt. Was er ihr gegeben hat, ist sein individuelles Arbeitsquantum. Z.B. der gesellschaftliche Arbeitstag besteht aus der Summe der individuellen Arbeitsstunden. Die individuelle Arbeitszeit des einzelnen Produzenten ist der von ihm gelieferte Teil des gesellschaftlichen Arbeitstags, sein Anteil daran. Er erhält von der Gesellschaft einen Schein, daß er soundso viel Arbeit geliefert (nach Abzug seiner Arbeit für die gemeinschaftlichen Fonds), und zieht mit diesem Schein aus dem gesellschaftlichen Vorrat von Konsumtionsmitteln soviel heraus, als gleich viel Arbeit kostet. Dasselbe Quantum Arbeit, das er der Gesellschaft in einer Form gegeben hat, erhält er in der andern zurück."[11]

Rosenkohl (Diskussion) 12:10, 27. Sep. 2015 (CEST)[Beantworten]

Das klingt nach einer scheinbaren Geld-Abschaffung ... mit Betonung auf Schein .... --Quetsch mich aus, ... itu (Disk) 13:48, 27. Sep. 2015 (CEST)[Beantworten]
Laut Marx muss das Geld im Laufe der Geschichte entfallen, dafür hat er ja die Geschichte exakt vorausgesehen. Zeitlebens war er auf Gaben Dritter, etwas des reichen Engels angewiesen - sprich ganz ernst gemeint war das füs erste nicht. Serten DiskSkeptisch : Kritik 12:15, 27. Sep. 2015 (CEST)[Beantworten]
Seufz. --91.44.83.12 15:11, 27. Sep. 2015 (CEST)[Beantworten]

Ich bin kein Marx Experte. Soweit ich es verstanden habe plädiert Marx zunächst dafür die "Dikatur des Proletariarts" einzuführen. In dieser Gesellschaft existiert noch Geld und auch ein alles im allen konventionelles Staatswesen, jedoch sollen laut Plan die Ungerechtigkeiten die es laut Marx im Kapitalismus gibt , insbesondere die ökonomische Ungleichheit Schritt für Schritt abgeschafft werden.

Diese Fortschritte sollen dann irgendwann als Endziel in den Kommunismus münden. Eine hochgradig egalitäre Gesellschaftsordnung in der es keinerlei Herrschaft mehr von Menschen über Menschen gibt. Beachtenswert ist das Marx nicht allzu viel über diese Kommunistische Gesellschaft und ihre Praxis sagt. Er geht hauptsächlich auf den Zwischenschritt also die Dikattur des Proletariarts ein und versucht zu begründen warum er ihn für notwendig hält.

In der Auseinandersetzung zwischen Marx und Proudhon merkt Engels sinngemäß an , das er (Proudhon) das was wir als eine Art Endziel sehen sofort will. (hier geht es ihm jedoch weniger um die Ökonomie sondern eher um die Freiheit des Individuums). Das deutet daraufhin das der Kommunismus also auch im Sinne Marx in letzter konsequenz eine Art anarchistische Idealwelt ist. Ich weiß nicht ob Marx je etwas nennenswertes dazu gesagt hat ob es in dieser laut ihm Idealwelt noch Geld gibt. Im heutigen Sinne sicherlich eher nicht.

Jedoch können sich auch einige Arnarchisten eine Art Messeinheit vorstellen die den Aufwand zur Herstellung, Förderung , Bearbeitung usw ausdrückt. Das heutige Geld was ja letztendlich auch Spekulationsmasse, Investitionsmasse ist und ungleich verteilt ist wird aber wohl eher abgelehnt. --93.217.141.139 13:17, 27. Sep. 2015 (CEST)[Beantworten]

Vielleicht ergänzend: In den Theorien der Freiwirtschaft bestehen auch Vorstellungen von einem Umlaufgesicherten Geld (Freigeld), das einem stetigen Wertverfall unterworfen ist, wenn es nicht innert eines bestimmten Zeitraums wieder in Umlauf gebracht wird. --91.44.83.12 15:09, 27. Sep. 2015 (CEST)[Beantworten]

Hat mit dieser Frage überhaupt nichts zu tun.--93.217.141.139 17:08, 27. Sep. 2015 (CEST)[Beantworten]

Wird Geld als Tauschwert durch Tauschwertzettel ersetzt, ändert sich genau was? Eher nichts außer dem Namen. --Tommes  19:30, 27. Sep. 2015 (CEST)[Beantworten]

Marx wollte in einer Produktionsgenossenschaft Geld nicht "als Tauschwert" durch sogenannte "Tauschwertzettel" ersetzen. Tausch im Kapitalismus bedeutet Tausch von Waren zu ihrem Wert; die Steuerung der Warenproduktion erfolgt einerseits direkt durch die privaten Kapitalinvestoren, andererseits indirekt durch den Erfolg auf dem Markt. Der Staat nimmt im Kapitalismus Steuern ein und reguliert die Sozialversicherungen, um auf diese Weise ein Minimum an gesellschaftlicher Reproduktion zu gewährleisten.
In einer Produktionsgenossenschaft, um die es in der Kritik des Gothaer Programms geht, wo also Bescheinigungen über geleistete Arbeitsstunden für Konsummittel eingelöst werden können, erfolgt die Planung der Produktion durch die Mitglieder der Genossenschaft; also nicht etwa analog zum Kapitalismus durch Reinvestition von Bescheinigungen, die besonders fleißige und sparsame Arbeiter einmal gesammelt aber nicht eingelöst haben. Für die Bescheinigungen können überdies nur soviel Konsummittel eingelöst werden, wie "nach Abzug der Arbeit für die gemeinschaftlichen Fonds" übrigbleibt. Diese gemeinschaftlichen Fonds tragen einerseits Kosten für die Produktion selbst: zum "Ersatz für verbrauchte Produktionmittel", zur "Ausdehnung der Produktionmittel", "Reserve und Assekuranzfonds" zur Absicherung. Andererseits Kosten für die gesellschaftliche Reproduktion: "allgemeine Verwaltungskosten", gemeinschaftliche Bedürfnisse wie "Schulen und Gesundheitsvorrichtungen etc.", "Fonds für Arbeitsunfähige".
Rosenkohl (Diskussion) 21:21, 27. Sep. 2015 (CEST)[Beantworten]

Tjo, das hat Rosenkohl solide erklärt. Erwähnensewert sind vielleicht noch die praktischen Probleme an dieser Überlegung die Marx so nicht richtig eingeschätzt hat. Dazu zähle ich z.B. eine unglaubliche fixierung auf die Ökonomie die andere Kapitalformen vernachlässigt hat. In diesem Bereich hat dann der Franzose Boudieu sehr viel geleistet. Ferner gelang es in der Praxis eben nicht die selbe wirtschaftliche Dynamik die konkurierende Kapitalisten ausgelöst haben in einer zentralgesteuerten Wirtschaft auszulösen. --93.217.141.139 21:44, 27. Sep. 2015 (CEST)[Beantworten]

Eine wichtige Kritik an der Zentralwirtschaft - Friedrich_August_von_Hayek#Die_Anma.C3.9Fung_von_Wissen_.281974.29 - war die Inspiration für Wikipedia. Geld und Warenpreise am Markt sind wichtige Informationsträger, die ein Plan nicht ersetzen kann. Serten DiskSkeptisch : Kritik 12:07, 28. Sep. 2015 (CEST)[Beantworten]
  • Zur Diskussion des Unterschiedes von Bourdieus und Marx' Kapitalbegriffen und Auffassungen der gesellschaftlichen Realität siehe z.B. Sven Ellmers: [12] (27 Seiten pdf). Marx war nicht auf die Ökonomie "fixiert", sondern er erachtete die Formen der ökonomischen Produktion als Basis, von der im Kapitalismus alle anderen Formen des gesellschaftlichen Überbaus abgeleitet sind, erachtete somit die Gesellschaft selbst als auf die Produktion "fixiert".
  • Der Realsozialismus etwa ab 1917 war selbst Teil der Weltwirtschaft. Die sozialistischen Staaten waren nach Innen autoritäre Regime, und nahmen zugleich nach außen am Welthandel teil. Auch Warengeschäfte zwischen sozialisitischen Staaten wurden nach dem Tauschwert abgewickelt.
  • Gleichwohl trifft es zu, daß der Realsozialismus etwa ab 1917 eine graduell geringere Dynamik entwickelte im Sinne geringerer technischer, wissenschaftlicher und kultureller Innovationsfähigkeit, bei zugleich geringerer Spaltung zwischen Arm und Reich; und es ist tatsächlich naheliegend, dies auf eine geringere individuelle Handlungsfreiheit zurückzuführen.
  • Wert und Geldpreis sind in der Tat im Kapitalismus Informationsträger zur Organisation des Produktionsprozesse; zugleich enthalten Wert und Geldpreis keine volle, sondern stets nur eine Teilinformation, und bewirken aus diese Weise, den Produktionsprozess für die Marktteilnehmer zu verschleiern d.h. zu fetischisieren.
  • Jedoch trifft nicht zu, daß Planung der Produktion im Kapitalismus ausschließlich "durch konkurierende Kapitalisten" erfolgt; vielmehr erfolgt diese Planung gerade durch ein Zusammenwirken von dezentralen Investitionsentscheidungen der Kapitalisten und einem zentralen Markt; auf diese Weise bildet sich eine komplexe fraktale Struktur.

Rosenkohl (Diskussion) 15:12, 28. Sep. 2015 (CEST)[Beantworten]

Bücherkritiksammlung

Gibt es zu Rotten Tomatoes oder Metacritic ein Äquivalent für Bücher, also eine Website, welche die Kritiken von Büchern sammelt?--JTCEPB (Diskussion) 15:10, 27. Sep. 2015 (CEST)[Beantworten]

Perlentaucher ist doch sowas? --King Rk (Diskussion) 15:38, 27. Sep. 2015 (CEST)[Beantworten]
Stimmt, hat aber zum von mir gesuchte Buch keine Rezensionen. Kennt wer noch weitere?-- JTCEPB (Diskussion) 15:45, 27. Sep. 2015 (CEST)[Beantworten]
Ich. --83.79.9.134 17:08, 27. Sep. 2015 (CEST)[Beantworten]
Bringt mich ehrlichgesagt nicht weiter.--JTCEPB (Diskussion) 17:17, 27. Sep. 2015 (CEST)[Beantworten]
Metacritic hatte mal eine Buchsparte, hamse vor drei oder vier Jahren abgeschafft, weil das Buch ist tot. --Edith Wahr (Diskussion) 17:21, 27. Sep. 2015 (CEST)[Beantworten]
Das Buch ist tot!? Igitt, ich bin ein Leichenfledderer! --Snevern 22:49, 27. Sep. 2015 (CEST)[Beantworten]
Und ich wohne in einem Massenbegräbnis? -- Zerolevel (Diskussion) 08:27, 28. Sep. 2015 (CEST)[Beantworten]
Du wohnst in einer Bibliothek? 129.13.72.198 14:15, 28. Sep. 2015 (CEST)[Beantworten]
aktueller Stand 8076 Bände - ohne die Fach-Literatur meiner Frau. -- Zerolevel (Diskussion) 17:50, 29. Sep. 2015 (CEST)[Beantworten]
Also 8078. Der grosse Diät-Ratgeber+Wie erziehe ich meinen Mann NEIN! Nicht hauen, nicht hauen, bitte nicht hauen... -- Ich möchte hier lieber Unerkannt bleiben (Diskussion) 10:07, 30. Sep. 2015 (CEST) [Beantworten]
Umsätze Buchhandel in Deutschland:
  • 1980 3,9 Mrd.
  • 1990 6,5 Mrd.
  • 2000 9,3 Mrd.
  • 2005 9,6 Mrd.
  • 2009 9,7 Mrd. [13]
  • 2012 9,52 Mrd. [14]
  • 2013 9,54 Mrd. [15]
  • 2014 9,32 Mrd. [16]
Prgonose
"Berichte über (m)ein Ableben waren verfrüht!"
--Eike (Diskussion) 14:39, 28. Sep. 2015 (CEST)[Beantworten]
Sind das nur Umsätze mit kodexförmigen Büchern oder sind da auch andere Medien drin, also Ebooks, Ebookreader, Hörbücher und so? --Rôtkæppchen₆₈ 16:46, 28. Sep. 2015 (CEST)[Beantworten]
Da sind auch E-Books drin - deren Anteil aber eh nur bei 4,3% (2014) liegt. Ob Hörbücher auch dazugezählt werden, weiß ich nicht, würde es aber vermuten. Aber es geht in dieser Frage ja auch um die Bewertung von Buch-Inhalten (nicht um tote Bäume), und die sind in allen Darreichungsformen dieselben. --Eike (Diskussion) 18:46, 28. Sep. 2015 (CEST)[Beantworten]

Ammedyysli

Warum heissen die baseldeutschen Ammedyysli Ammedyysli? --83.79.9.134 15:18, 27. Sep. 2015 (CEST)[Beantworten]

[17]. Wenige Sekunden. 77.176.64.44 15:33, 27. Sep. 2015 (CEST)[Beantworten]
Und was ist dort bei der genannten Webseite mit "Botz heitere Faane" gemeint? --83.79.9.134 15:42, 27. Sep. 2015 (CEST)[Beantworten]
Diese Randgruppendialekte! Unaussprechlich und unverständlich, ein Hindernis für den Binnenmarkt. Brüssel soll die deutsche Sprache vereinheitlichen und wenn die Schweizer nicht mitmachen, fliegen sie aus der EU raus. --83.79.9.134 20:11, 27. Sep. 2015 (CEST)[Beantworten]
Sie sind drin??? Dumbox (Diskussion) 20:13, 27. Sep. 2015 (CEST)[Beantworten]
Nein.--JTCEPB (Diskussion) 20:18, 27. Sep. 2015 (CEST)[Beantworten]
Sie wollen auch nicht rein. So blöd sind sie nicht. --83.79.9.134 21:04, 27. Sep. 2015 (CEST)[Beantworten]
„Botz heitere Faane“ war mir bisher unbekannt, jedoch wenige Sekunden später bekannt. --Pp.paul.4 (Diskussion) 11:08, 28. Sep. 2015 (CEST)[Beantworten]
Zum «Botz» oder «Potz» auch hier. Überhaupt gibt’s hier Auskunft zu vielen Dialektfragen. Gruss, --Freigut (Diskussion) 11:19, 28. Sep. 2015 (CEST)[Beantworten]

Dieselkraftstoff

Dieselkraftstoff: Komposition von Dieselkraftstoff - Anteile von Kohlenstoff etc. --91.45.253.147 16:48, 27. Sep. 2015 (CEST)[Beantworten]

Wikipedia: "Die Hauptbestandteile des Dieselkraftstoffes sind vorwiegend Alkane, Cycloalkane und aromatische Kohlenwasserstoffe..." --83.79.9.134 16:52, 27. Sep. 2015 (CEST)[Beantworten]
Faustregel: Dieselkraftstoff besteht zu etwa einem Siebtel aus Wasserstoff, der Rest Kohlenstoff. --Rôtkæppchen₆₈ 17:17, 27. Sep. 2015 (CEST)[Beantworten]
Faustregel: Behauptungen ohne Belege sind genaudas. Ob Volumen- oder Massenanteile gemeint sind, wird auch nicht geschrieben. --83.79.9.134 17:53, 27. Sep. 2015 (CEST)[Beantworten]
Geh von der von Dir zitierten Zusammensetzung aus, vernachlässige die Ränder der Alkane und Du hast … sechs siebtel Kohlenstoff und ein siebtel Wasserstoff. Das ergibt sich aus der allgemeinen Summenformel CnH2n+2 für Alkane und CnH2n für Cycloalkane und den molaren Massen von 1 g/mol für Wasserstoff und 12 g/mol für Kohlenstoff. Alles das sind triviale Fakten und deshalb nicht belegpflichtig. Wenn Du ein Chemiebuch hast, wo abweichendes drinsteht, nur her damit. --Rôtkæppchen₆₈ 18:04, 27. Sep. 2015 (CEST) Volumenanteile sind nur bei Gasen oder Trinkalkohol üblich, anderswo sind Massen- oder Stoffmengenanteile üblich, deswegen bin ich stillschwiegend von Massenantielen ausgegangen. Der hier sinnlose Stoffmengenanteil der einzelnen Elemente ergibt sich direkt aus den allgemeinen Summenformeln: ⅔ H und ⅓ C. --Rôtkæppchen₆₈ 18:10, 27. Sep. 2015 (CEST)[Beantworten]
Du meinst Massenanteile. --83.79.9.134 18:13, 27. Sep. 2015 (CEST)[Beantworten]
+ 1/1... *rotfl* *scnr* --Heimschützenzentrum (?) 22:21, 27. Sep. 2015 (CEST)[Beantworten]

Affective computing in Deutsch

Die Portale Informatik und Software erlauben Fragen, geben aber keine Antworten. Deshalb hier gefragt: Gibt es enzyklopädisch akzeptierte deutsche Begriffe für affective computing und cognitive computing? Danke für verwendbare Antworten! Bikkit ! (Diskussion) 17:46, 27. Sep. 2015 (CEST)[Beantworten]

Nein. Antwort ohne Beleg weil negativa non sunt probanda. --83.79.9.134 17:55, 27. Sep. 2015 (CEST)[Beantworten]
Die Übersetzung "affektive EDV" scheint gebräuchlich. 90.184.23.200 18:54, 27. Sep. 2015 (CEST)[Beantworten]
Nein. --83.79.9.134 18:59, 27. Sep. 2015 (CEST)[Beantworten]
Ich finde 4 x "affektive EDV" und 3 x ""affektive Datenverarbeitung". Das scheint mir zu wenig zu sein, um als etablierter Begriff verwendet zu werden. Dann muß eben Umschreibung und das Original her. Hier erledigt! Bikkit ! (Diskussion) 17:17, 28. Sep. 2015 (CEST)[Beantworten]

Die Bank sprengen

Ich habe vor einiger Zeit den Terminus "Die Bank sprengen" gelesen und hätte gern gewusst, was das bedeutet bzw. wann das gebraucht wird. M. E. taucht das im Kontekt con Casinos auf.

(Ich meine, dass ich die Anfrage schon mal gestellt habe, wollte gerade nachlesen und habe mich durchs Archiv der letzten 3 Wochen gewühlt, konnte aber nichts finden. Vielleicht bilde ich mir das aber auch nur ein.) --91.53.19.130 18:49, 27. Sep. 2015 (CEST)[Beantworten]

siehe weiter oben unter #Was_bedeutet_es_im_Casino_.22die_Bank_sprengen.22.3F
gruß, --JD {æ} 18:50, 27. Sep. 2015 (CEST)[Beantworten]

Danke! --91.53.19.130 19:41, 27. Sep. 2015 (CEST)[Beantworten]

Foto vs. Thumbnail

Hallo,
mich bewegt Folgendes: Aus einem Foto wurde eine Person wegretuschiert. Als ich heute wegen der Geokoordinaten die EXIF-Daten online mittels Jeffrey's Exif Viewer auslesen ließ, wurde mir ein Thumbnail angezeigt, auf dem die Person noch enthalten ist. Wie funktioniert das? Die normale Thumbnail-Ansicht im Explorer zeigt die aktuelle Version des Fotos (nur halt kleiner) an. So möchte ich das Foto im entsprechenden Artikel nicht verwenden. Ich will aber auch nicht pauschal _alle_ EXIF-Daten entfernen, wie das manche Tools anbieten. So wundert sich --Tommes  19:48, 27. Sep. 2015 (CEST)[Beantworten]

Es kann per Exif ein extra Vorschaubild in der Bilddatei stecken, um die sich manche Grafikprogramme nicht kümmern. Das sollte sich mit einem passenden Grafikprogramm oder z. B. mit ExifTool beheben lassen. --Eike (Diskussion) 19:51, 27. Sep. 2015 (CEST)[Beantworten]
Ich suche zur Entfernung desselben ein Onlinetool. --Tommes  20:51, 27. Sep. 2015 (CEST)[Beantworten]
Online-Tools interessieren mich nicht so, aber ich vermute, da ist was dabei: https://www.google.de/search?q=edit+exif+online --Eike (Diskussion) 21:35, 27. Sep. 2015 (CEST)[Beantworten]
Danke, aber Google kenne ich. Ich werde warten müssen, bis aich an einen Rechner mit entsprechender Software komme. --Tommes  01:25, 28. Sep. 2015 (CEST)[Beantworten]
Google kennt jeder, aber manchmal fehlt es einem ja an den passenden Begriffen. Die gefundenen Websites können das alle nicht?
Mit Exiftool geht es so: exiftool -ThumbnailImage= input.jpeg
--Eike (Diskussion) 07:56, 28. Sep. 2015 (CEST)[Beantworten]

Was ist ein § meistens

Ein Paragraph in einer Rechtsnorm ist genau ... was? Rechtsnorm und Rechtssatz erklären mir das nicht so richtig. Vermutlich hat Rechtsnorm zwei Bedeutungen: Zum einen das Gestzeswerk, zum andern die verschiedenen Regelungen innerhalb eines Gesetzeswerkes. Grund für mein Nachfragen ist diese Änderung (Stelle > Rechtsnorm). --93.133.128.188 20:20, 27. Sep. 2015 (CEST)[Beantworten]

Ein Paragraph ist eigentlich einfach ein Absatz im Sinne eines Gliederungsabschnitts. Manche Gesetze (insbesondere die sogenannten "Artikelgesetze", die Regelungen in anderen Gesetzen ändern) enthalten nur ganz wenige (und dafür manchmal enorm umfangreiche) Paragraphen. In zahlreichen Paragraphen deutscher Gesetze ist mehr als ein Sachverhalt geregelt; andererseits ergeben sich manche inhaltlich zusammenhängenden Regelungen nur aus dem Zusammenspiel mehrerer Paragraphen.
Im Zusammenhang mit der genannten Änderung erscheint mir "Rechtsnorm" schöner als schlicht "Stelle" - und inhaltlich passend ist es auch. "Norm" ist ja auch ein schillernder Begriff, mit dem man sowohl die umfassende Regelung eines Sachverhalts als auch schlicht einen Paragraphen (oder sogar einen Teil von ihm) bezeichnen kann. Ich würde die Änderung daher befürworten. --Snevern 22:44, 27. Sep. 2015 (CEST)[Beantworten]
Schon die Wortherkunft läßt darauf schließen, dass Paragraph nicht mehr als ein inhaltlich neutrales (bisweilen mit Buchstaben unterteiltes) Ordnungselement ist, hinter dem sich eine weitere Struktur aus Absatz, Nummer, Buchstabe auffächert. Ich habe keine Ahnung, aber ich hätte eher Rechtsquelle verwendet, weil ich denke, dass sich aus einer Rechtsnorm ein Rechtssatz ergibt, der in einer Rechtsquelle wirkt. Und diese wird ja offenbar in der Spalte aufgeführt. Aber vielleicht irre ich mich ja auch. --91.44.83.12 23:19, 27. Sep. 2015 (CEST)[Beantworten]

Python Shell und sudo

Guten Abend, viel gegoogelt, keine Lösung gefunden im Terminal kann ich ein selbstgeschriebenes Programm mit Hilfe von "sudo" öffnen. ich möchte es aber nicht nur im Terminal öffnen, aber es kommt immer "Errno 13 - Permission Denied" wie schaffe ich es, das Programm automatisch als sudo mit IDLE zu öffnen? vielen dank --2A02:8070:4187:1800:98A4:AA97:D55E:9C60 20:23, 27. Sep. 2015 (CEST)[Beantworten]

hi! 1. es gibt das Sticky Bit... aber dann können alle das Ding als superuser ausführen... 2. was spricht gegen ein Shell Script, das dann #sudo selbstgeschriebenes# ausführt? --Heimschützenzentrum (?) 22:16, 27. Sep. 2015 (CEST)[Beantworten]
Du verwechselst Setuid und Sticky... -- FriedhelmW (Diskussion) 22:41, 27. Sep. 2015 (CEST)[Beantworten]
oops... stimmt... --Heimschützenzentrum (?) 23:19, 27. Sep. 2015 (CEST)[Beantworten]


Startest du denn dein Python-Skript überhaupt aus einem Terminalemulator? Den braucht sudo nämlich, weil es ja nach dem Kennwort fragen muss, das macht es nur auf einem TTY-Device. Alternativ option --askpass verwenden und das Kennwort darüber übergeben. -- Janka (Diskussion) 00:26, 28. Sep. 2015 (CEST)[Beantworten]
Oh mist, ich versteh nur Bahnhof. Danke trotzdem--91.89.90.46 14:01, 28. Sep. 2015 (CEST)[Beantworten]
Skript-Kiddie? -- Janka (Diskussion) 20:54, 28. Sep. 2015 (CEST)[Beantworten]

Starte idle mittels sudo: sudo idle meinprogramm.py --Tbhgeo (Diskussion) 13:23, 29. Sep. 2015 (CEST)[Beantworten]

Wappen der Republik Mazedonien

--87.177.91.103 21:43, 27. Sep. 2015 (CEST) Meine Frage:Das Original-Wappen von Mazedonien ist nicht das dargestellte Wappen. Durch die konservative Regierung wurde das Original-Wappen der Republik Mazedonien per Parlamentsbeschluß verändert, d.h. der rote Stern oben in der Mitte verschwand.(Das sollte man wenigstens erwähnen). Ich stelle die Frage deshalb, weil ich bei meinen zahlreichen Besuchen festgestellt habe, daß viele speziell ältere Menschen dies als eine Beleidigung Mazedoniens empfinden. Der rote Stern stammt aus den Gründerzeiten Mazedoniens und jeder Partisan trug ihn. Ohne die Partisanen gäbe es den Staat Mazedonien überhaupt nicht. Sie sollten auch erwähnen, daß die Nationalflagge Mazedoniens mehrfach verändert wurde, aber das nur nebenbei. Fazit: Warum zeigen Sie nicht das alte Original-Wappen und das aktuelle? --87.177.91.103 21:43, 27. Sep. 2015 (CEST)[Beantworten]

Vorschlag: Sei Mutig und füge es selbst in die entsprechenden Artikel ein.--JTCEPB (Diskussion) 22:20, 27. Sep. 2015 (CEST)[Beantworten]
Im Artikel Wappen Mazedoniens ist der Sachverhalt aber korrekt dargestellt. --Rôtkæppchen₆₈ 22:56, 27. Sep. 2015 (CEST)[Beantworten]

Gleichbehandlung Selbstständiger mit Forschern und Hochqualifizierten

Hallo! Aktuell kommt ja in der Flüchtlingsdebatte vieles auf die Tagesordnung, was bislang schon galt, aber nicht groß bemerkt wurde. Dazu gehört der Familiennachzug von Personen mit Aufenthaltstiteln. Unter [18] ist nachzulesen, daß es bei den nötigen Deutschkenntnissen Ausnahmen für bestehende Ehen von Forschern und Hochuqualifizierten gibt, was mir durchaus verstänndlich ist. Nur warum werden Selbstständige dieser Gruppe ohne weitere Qualifikation gleichgestellt. Ich meine damit Restaurantbesitzer, Taxiunternehmer oder Tankstellenpächter, die nun wie Ärzte und Atomphysiker behandelt werden, und nicht wie Kellner, Taxifahrer oder Tankwarte. Kennt jemand die rechtliche Begründung dafür? Bei anderen Regelungen S. 30/40 ist das ungefähr so, dort wird die Gruppe als "Studierende, Arbeitnehmer oder Selbstständiger" definiert. Schreibfehler, oder wirklich unterschiedliche Regeln für den selben Personenkreis?Oliver S.Y. (Diskussion) 23:13, 27. Sep. 2015 (CEST)[Beantworten]

Diese ganze "Deutschpflicht für Ausländer" ist ein Haufen Unsinn, über den man sich keine allzu großen Gedanken machen sollte. Es sei denn, man will die Erkenntnis gewinnen, dass unsere "Eliten" ein Haufen Vollidioten sind. -- Janka (Diskussion) 00:34, 28. Sep. 2015 (CEST)[Beantworten]
Also ich sehe durchaus einen Unterschied zwischen Restaurantbesitzern und Kellnern. --mfb (Diskussion) 00:47, 28. Sep. 2015 (CEST)[Beantworten]
Ja, aber diese werden gleich behandelt^^, nur nicht deren Ehepartner. Was ich ja bei "Wissenschaftlern" auch verstehen, wenn diese häufig die Anstellungen wechseln, und die Ehepartner nicht immer eine neue Sprache lernen können/wollen. Aber Selbstständige sind ja eher auf Dauer ansässig. Warum dann nicht wie Arbeitnehmer? @Janka, sry, ich erlebe es fast täglich, das selbst Migranten der ersten Generation nach 50 Jahren in Deutschland nicht in der Lage sind, ein einfaches Gespräch zu führen, oder am öffentlichen Leben ihrer Umgebung durch Teilnahme an deutschsprachigen Veranstaltungen mitzuwirken. Integration sieht anders aus, und Parallelwelten schaden Migranten wie Natives.Oliver S.Y. (Diskussion) 00:54, 28. Sep. 2015 (CEST)[Beantworten]
Ich bezweifle nicht den Nutzen, Deutsch zu lernen. Ich bezweifle den Nutzen, irgendwelche waghalsigen gesetzlichen Pflichten einzuführen, die nichts erzeugen außer einem Haufen Bürokratie bei Null Nutzen wegen der vielen Ausnahmen. -- Janka (Diskussion) 03:48, 28. Sep. 2015 (CEST)[Beantworten]
Hm. Also ich stehe eigentlich auf dem Standpunkt dass alle Flüchtlinge die ankommen Englisch lernen sollten und nur wer das gut kann, sollte mit Deutsch anfangen - wenn das unbedingt sein soll. --Quetsch mich aus, ... itu (Disk) 02:25, 29. Sep. 2015 (CEST)[Beantworten]
»ich erlebe es fast täglich, das selbst Migranten der ersten Generation nach 50 Jahren in Deutschland nicht in der Lage sind, ein einfaches Gespräch zu führen« Das ist exakt die Generation der »Gastarbeiter«, bei denen an Intergrationsbemühungen überhaupt nicht gedacht wurde. Sie sollten (und wollten) ja nur ein paar Jahre hier malochen. Obendrein waren das in der großen Mehrheit sehr einfache, junge Leute vom Dorf ohne nennenswerte Schulbildung. Aber wie das so ist – sie haben natürlich Familien gegründet, aus ein paar Jahren wurden Jahrzehnte. Man kann nur hoffen, dass aus den damaligen Fehlern gelernt wurde und wird.
Ich erinnere mich noch an die Atmosphäre in den 60er, 70er Jahren. Deutschland war damals weit hermetischer als heute, Misstrauen und Fremdenfeindlichkeit waren allgemein sehr ausgeprägt, bis in die Politik hinein. Die türkischen Einwanderer haben darauf verständlicherweise mit Einigelung in eine »Parallelgesellschaft« reagiert. Wozu übrigens auch die stark angestiegene Frömmigkeit gehört. Mit einer anderen Einwanderungspolitik wäre das nicht passiert. Rainer Z ... 14:59, 28. Sep. 2015 (CEST)[Beantworten]
Diese Einigelungsthese klingt sehr interessant nett, aber wenn ich mir das rein zeitlich betrachte haut das für mich nicht hin. Vielmehr scheint mir die zunehmende Religiösität deutscher Migranten synchron zum globalen Prozess zunehmender „Re-Islamisierung“ oder „Fundamentalisierung“ zu laufen und entsprechend von aussen induziert zu sein. --Quetsch mich aus, ... itu (Disk) 02:25, 29. Sep. 2015 (CEST)[Beantworten]
Die Re-Islamisierung im Nahen Osten hat sicher auch eine Rolle gespielt, doch muss die ja auch auf fruchtbaren Boden fallen. Un der wurde sicher auch durch die politische, wirtschaftliche und gesellschaftliche Ausgrenzung der Einwanderer über Jahrzehnte bereitet. Rainer Z ... 11:25, 29. Sep. 2015 (CEST)[Beantworten]
Du willst die These wohl retten, aber das haut mMn nicht hin. Nur zu einem kleinen Teil dürfte diese Kausalität zutreffen. Der Grossteil der Gastarbeiter hatte garantiert nicht vor sich zu integrieren. Ich denke die Vorstellung gab es gar nie (man wollte ja eigentlich zurückkehren), sondern es lief einfach darauf hinaus sich auf die familiäre Existenz zu beschränken plus Kontakte am Arbeitsplatz, mehr nicht; mehr brauchte es nicht und mehr wollte der Gastarbeiter auch nicht wirklich. (Bei den Nachgeborenen haben wir natürlich eine ganz andere Situation, allein durch den Schulbesuch,etc.).
Es lief also schon früher auf Koexistenz hinaus. In der Reinform ist das das was wir heute massenhaft haben: Ausländerghettos. Aber auch ausserhalb der Ghettos ist ganz überwiegend die Beschränkung auf die Familie typisch(wie gesagt für die jüngere Generationen sieht es natürlich nochmal etwas anders aus).
Koexistenz ist genau das Gegenteil von Integration. Integration bedeutet aus meiner Sicht Anpassung, bedeutet Assimilation.
Und die hängt immer ganz entscheidend vom Migrant ab. Und das sehr individuell. Gutes Anschauungs und Vergleichsbeispiel ist eine Ausländergruppe die überhaupt nie unter den Begriff Migrant gefasst wird: das sind unsere „amerikanischen Freunde“. Die stehen uns zweifellos mental und kulturell näher als viele nichtwestliche Migranten. Bei hier lebenden US-Amerikanern herrscht die hochgradige „Einigelung“ - allerdings völlig problemlos, was nicht nur darauf beruht dass es überhaupt keine Berührungspunkte nach aussen gibt, sondern auch an einer betont zurückhaltenen Art. Nur ganz wenige Amerikaner gehen hierzulande den anderen Weg und passen sich (dann auch problemlos) an, oder versuchen zumindest ihr Bestes.(Disclaimer: bin kein Amerikafreund) --Quetsch mich aus, ... itu (Disk) 12:42, 29. Sep. 2015 (CEST)[Beantworten]
Nicht unwichtig: Die Wertvorstellungen und das Verhalten von Vermietern und Mietern haben die Ghettobildung und damit die kleinen Inseln von (tendenziellen) Parallelgesellschaft ja befördert. Aus Sicht der Emigranten ist es verständlich, wenn sie dort wohnen wollen, wo "ihre Leute" bereits sind. Sehr oft hatten sie aber auch gar keine andere Wahl und die Folge war ein geringeres Hineinwachsen in die Aufnahmegesellschaft. --91.44.83.12 19:49, 28. Sep. 2015 (CEST)[Beantworten]
Die Islamisierung seit den 1970ern fiel aber überall in der islamsichen Welt auf fruchtbaren Boden und beschränkt sich nicht auf die unintegrierten Türken/Moslems in Deutschland/Europa. An sich ist eine Abschottung von Minderheiten/Einwanderergruppen und deren friedliche Koexistenz mit anderen ethnischen Gruppen und der Mehrheitsgesellschaft ja nichts per se negatives - du kannst niemanden dazu zwingen, sich mit bestimmten Personen in der Freizeit zu treffen, Geschäfte zu machen oder sie zu heiraten. Chinatowns stellen doch keine Problemviertel (mehr) dar, sind heute sogar eine Touristenattraktion. Die 1-2 Mio. Polen in D verhalten sich so unauffällig, dass man sie gar nicht wahrnimmt (andere Osteuropäer auch). Das "Intergrationsproblem" betsteht doch vielmehr darin das Deutsche/Christen/"Weiße" und Moslems schlicht verfeindet und letztere auf Expansionskurs sind (man erkläre mir mal, wie Homoehe und Aufhängen Schwuler an Baukränen zusammengehen soll).--Antemister (Diskussion) 21:36, 29. Sep. 2015 (CEST)[Beantworten]

Filmt eigentlich hier jemand die morgige Mondfinsternis? Wäre schön ein gutes Zeitraffervideo auf der obigen Seite zu haben. Selbst Commons hat nichts was an das Spektakel morgen heranreicht. Danke :) --Impériale (Diskussion) 23:39, 27. Sep. 2015 (CEST)[Beantworten]

28. September 2015

Bürgerkrieg in Deutschland - wer behauptet sowas?

Ein Bürgerkrieg (=Situation wie in Syrien oder Somalia) in Deutschland ist absolut abwegig und unvorstellbar. Solche Behauptungen sind als absoluter Schwachsinn einzustufen, das ist mir klar. Trotzdem habe ich eine solche Behauptung vor etwa einer Stunde gehört... unter Berufung auf einen "Pasching" oder "Fasching" oder so ähnlich. Man hat nämlich behauptet, man wisse nicht genau, wie die "Situation im Januar in Deutschland" sei, es könne/würde evtl. "Bürgerkrieg" ausbrechen.

Daher habe ich nun doch einige Fragen: Welche Gruppe (von Verschwörungstheoretikern) oder welche Person(en) behaupten, es würde bald Bürgerkrieg in Deutschland geben? Weswegen behaupten diese Leute sowas, bzw. aus welchem Anlass sollte es Bürgerkrieg geben? Etwa wegen der Flüchtlinge/-spolitik? Oder weil Nazis oder Kommunisten einen Aufstand anzetteln wollten wie 1932? Oder will die Reichsbürgerbewegung bzw. eine Kommissarische Reichsregierung das Deutsche Reich zurückerobern? --ObersterGenosse (Diskussion) 00:28, 28. Sep. 2015 (CEST)[Beantworten]

Sowas behaupten vor allem solche, die einen solchen Krieg anzetteln wollen. Ich bin mir aber ziemlich sicher, dass rechte Putschisten oder paramiltärische Gruppen in Deutschland nicht mal annähernd eine Chance haben sich mit Gewalt durchzusetzen. Dazu saufen die zuviel Bier.--Giftzwerg 88 (Diskussion) 00:36, 28. Sep. 2015 (CEST)[Beantworten]
Michael V. Hayden soll das angeblich mal gesagt haben.--JTCEPB (Diskussion) 01:04, 28. Sep. 2015 (CEST)[Beantworten]
Und zwar vor über 7 Jahren. 100 mal abgeschrieben und immer noch falsch zitiert, er sprach im Original nur von boosting the potential for unrest and extremism und weder von Germany noch von Bürgerkrieg. --83.79.9.134 07:55, 28. Sep. 2015 (CEST)[Beantworten]
Spiegel-TV hat letztens einen "Facebook-Hetzer" ("KZ") besucht. Das war - vorsichtig ausgedrückt - ein ganz einfaches Gemüt, der sein Leben noch nie selbständig (ohne staatliche Unterstützung) auf die Reihe bekommen hat. In die gleiche Kategorie würde ich das hier einordnen. (Selbstverständlich gibt es auch in D - trotz Schulpflicht - sehr viele gleichartig limitierte Gemüter....)--Wikiseidank (Diskussion) 08:11, 28. Sep. 2015 (CEST)[Beantworten]
Wartet es nur ab. Hotte ist/wird z.Zt. stark gedemütigt. Angie hört nicht auf ihn, die Medien vergackeiern ihn nur, Zähneknirschend muss er Flüchtlinge rein lassen, die Maut habens ihm versaut, das Betreuungsgeld auch, seine Minister sind nur noch in Witzblättern zu finden. Papst sind sie auch nicht mehr und die vorgesehenen Elogen auf das Nationalsymbol Strauß sind im Rest Deutschland wüst verzerrt worden. Zur Zeit sind sie mit dem Oktoberfest(Bier) beschäftigt, die Zeit bis zum Weihnachtsbock ist noch lang und unter dem Bürgerkriegsaspekt bekommt auch die Allianz mit einem gewissen Ungar einen ganz anderen Sinn. Da braut sich was zusammen. Holzauge sei wachsam.--2003:68:ED07:D000:80C9:F14D:5630:1377 09:48, 28. Sep. 2015 (CEST)[Beantworten]
Da braucht's keinen Bürgerkrieg, da können die Bayern doch einfach ein Referendum zur Unabhängigkeit starten. Dass deswegen gleich Bürgerkrieg ausbricht, glaub ich ja nicht...--ObersterGenosse (Diskussion) 10:00, 28. Sep. 2015 (CEST)[Beantworten]
Magst recht haben dazu kommt noch, dass Hotte aus den Ruhrgebiet Bayerns (Ingolstadt) stammt, mit deren Gmütliachkeit habens nicht viel am Hut. Und da die Bayern hauptsächlich in der Marine zu finden sind und die Norddeitschn bei den Gebirgsjägern, hat auch die Bundeswehr vorgesorgt. --2003:68:ED07:D000:80C9:F14D:5630:1377 10:16, 28. Sep. 2015 (CEST)[Beantworten]
Da muss ich mich mal Wikiseidank anschließen. Vielleicht steckt ja auch die Hoffnung dahinter, dass danach alles besser wird für ihn.
@ObersterGenosse: Auf Facebook kann man solcherlei täglich finden.
--Eike (Diskussion) 10:06, 28. Sep. 2015 (CEST)[Beantworten]
Das hier ist nicht das Cafe, trotzdem möchte ich darauf hinweisen, daß eine Denkweise mit Vokabeln wie "unvorstellbar", "abwegig" "absolut schwachsinnig" selbst Schwachsinn ist, weil sie jeglichen Sicherheitsanspruch einer Gesellschaft mißachtet. Wer den Deutschen Herbst 1977 erlebt hat, weiß vieleicht noch, mit wie wenig Aufwand die Gesellschaft ins Chaos zu stürzen ist. Da genügen wenige Aktionen, und die Politiker setzen die Verfassung außer Kraft, um z.B. das Militär innerhalb Deutschlands gegen Menschen einzusetzen. Nicht wirklich alle Militärs werden das mitmachen, und schon hat man Waffen, Ausrüstung und Motivation auf 2 Seiten. Das alles noch befeuert von einer Bevölkerungsgruppe, die dem Staat seit 45 Jahren in inniger Ablehnung gegenübersteht (die studierten Alt-68), und einem zornigen Prekariat, welches sich um Chancen wie Zuwendung betrogen fühlt. Dazu noch solche Gruppen wie die Organisierte Kriminalität, welche die Chance wittert, ihre Macht zu vergrößern. Diese Gefahr war schon immer vorhanden, und die Linksautonomen Gewaltexzesse sind genauso ein jährliches Ritual wie rechtsextreme Hooligangruppe und nationalextreme Bürgerwehren. Das Ganze trifft dann mit den Flüchtlingen auf eine Gruppe, welche nicht homogen ist, außer wenn es um Massenphänomene geht. Wirklich unvorstellbar, daß man sich bei 20 Grad Nachtfrost nicht mehr mit Zeltunterkünften abfindet, und schon durch den Lagerkoller mit 1500 anderen fremden Menschen Gewaltausbrüche wie in Calden häufiger werden? Die Polizei ist schon jetzt am Rande der Arbeitsfähigkeit, in Berlin benötigt man für einen Notruf schon jetzt 30 Minuten (wegen Asbest in der Leitzentrale). Ist es wirklich absurd, da anzunehmen, daß die Polizeilehrer recht haben/hatten, welche das Eintreffen innerhalb von Minuten als Grundvoraussetzung eines erfolgreichen Einsatzes predigen, da sonst der Mob die Straße übernimmt? Ihr könnt ja prima auf die Gruppe der Leistungsempfänger herabsehen, aber wenn man Syrien/IS oder Somalia betrachtet, die Strukturen stammen aus der Mittelschicht und wir sind so angreifbar geworden, das ein Stromausfall in Frankfurt sowohl E-Cash, das Internet als auch einen Großteil des Bahnverkehrs lahmlegt. Genug Angreifpunkte, um die Grundlage für einen Bürgerkrieg zu legen. Denn Waffen sind dafür nun wirklich genügend vorhanden.Oliver S.Y. (Diskussion) 10:46, 28. Sep. 2015 (CEST)[Beantworten]
Ich sehe nicht auf "Leistungsempfänger" herab (seltsames Wort, übrigens), sondern auf dumme Menschen, die die Verantwortung für ihre Probleme bei den Schwächsten suchen und Gewalt für ein angemessenes Mittel halten. --Eike (Diskussion) 10:53, 28. Sep. 2015 (CEST) [Beantworten]
Ging an Wikiseidank, die Schema ähneln sich, nur die Bezeichnungen ändern sich. Ob nun "Arbeitslose", "Sozialhilfeempfänger" oder "H4ler", es ist immer die Einkommensgruppe ganz am Ende gemeint. --Oliver S.Y. (Diskussion) 10:58, 28. Sep. 2015 (CEST)[Beantworten]
Noch seltsamer ist übrigens der Nichtleistungsempfänger. Der empfaengt Nichtleistung. Quatsch, gibts nicht? Leider doch: [19] --Nurmalschnell (Diskussion)

Stehender Spruch eines damals kurz von der Pensionierung befindlichen Lehrers: "Da traf ich einen Mann und sagte zu ihm: 'Der Dritte Weltkrieg hat bereits begonnen"' Und der Mann sagte zu mir: 'Der Zweite Weltkrieg hat noch gar nicht aufgehört!'" Immer wieder lustig, solche Sachen! *sncr*--Heletz (Diskussion) 12:21, 28. Sep. 2015 (CEST)[Beantworten]

@Oliver: ist nicht Cafe, aber es geht um die Feststellung, dass es viele Menschen gibt, die im "echten" Leben nicht zurechtkommen (würden/gekommen sind) und jetzt nur durch staatliche Hilfe zurecht kommen, wie Lehrer, Hochschullehrer, Verwaltungsmitarbeiter, Subventionsempfänger usw. Nur weil D seit 66 Jahren eine Demokratie ist, besteht D nicht aus 100% Demokraten. Einen Nachweis (Demokraten-Schein) benötigt man dafür nicht und kann trotzdem die Möglichkeiten einer Demokratie (bspw. Meinungsfreiheit) nutzen.--Wikiseidank (Diskussion) 13:30, 28. Sep. 2015 (CEST)[Beantworten]

Menschenaffen und Zeichensprache

Hallo, mir sind die Namen von zwei Gorillas und einem Schimpansen entfallen, die auf Zeichensprache sehr interessante Dinge übermittelt haben. Einer der Gorillas beschrieb die Tötung seiner Mutter durch Wilderer, der andere hatte eine Antwort auf die Frage "Was ist der Tod?", und der Schimpanse log seine Pflegerin an. Kann mir einer sagen, wie diese individuellen Affen hießen? Vielen Dank und Gruß--Martin-rnr (Diskussion) 01:42, 28. Sep. 2015 (CEST)[Beantworten]

Der mit der Antwort auf die Frage nach dem Tod hieß jedenfalls Koko. Habe ich mal auf einer Seite im Umfeld von PETA (aber eher spirituell orientiert) gefunden. --ObersterGenosse (Diskussion) 02:26, 28. Sep. 2015 (CEST)[Beantworten]
Der Schimpanse könnte Washoe (Schimpansin) sein.--JTCEPB (Diskussion) 03:10, 28. Sep. 2015 (CEST)[Beantworten]
Es gäbe ebenfalls noch den Schimpansen Nim Chimpsky (https://en.wikipedia.org/wiki/Nim_Chimpsky) und einen Orang Utan namens Chantek, die ebenfalls Zeichensprache erlernten.--95.223.244.70 07:03, 28. Sep. 2015 (CEST)[Beantworten]

"Qualifizierte" Kriminalität

Was meint die Polizei mit "qualifizierter" Kriminalität eines bestimmten Genres? Was genau ist etwa "qualifizierte Bandenkriminalität"? Wodurch unterscheidet sie sich von "unqualifizierter Bandenkriminalität"? --KnightMove (Diskussion) 07:35, 28. Sep. 2015 (CEST)[Beantworten]

"Qualifiziert" bedeutet im juristischen ein Konkretisierung (sonst überwiegen ja oft die "unbestimmten Rechtsbegriffe" - Treu und Glauben usw.). Bei Kriminalität vielleicht(!) nicht nur den Anschein der Kriminalität, sondern deren Nachweis/Bestätigung durch Gerichtsurteil?--Wikiseidank (Diskussion) 08:05, 28. Sep. 2015 (CEST)[Beantworten]
Wieso gibst du hier Antworten, wenn du sie mit einem Fragezeichen versehen muss? Was "qualifiziert" meint, steht in der Wikipedia: Qualifikation (Strafrecht). --83.79.9.134 08:19, 28. Sep. 2015 (CEST)[Beantworten]
Danke für die Antwort. Ich hätte ja vermutet, es geht darum, ob man für die Aufnahme in die Bande eine Prüfung ablegen muss oder nicht ;) -- HilberTraum (d, m) 08:44, 28. Sep. 2015 (CEST)[Beantworten]
Qualifizierte Kriminalität ist doch, wenn sie schlauer als die Polizei ist. SCNR--2003:68:ED07:D000:80C9:F14D:5630:1377 09:26, 28. Sep. 2015 (CEST)[Beantworten]
Weil der Artikel die Frage des Fragestellers nicht abschließend beantwortet und deswegen vielleicht überarbeitungsbedüftig ist? Er enthält lediglich: den Aspekt der Verwirklichung (von mir so beantwortet) und den Aspekt der Straftatbestandsverschärfung, der sich aus dem im Artikel genannten Beispiel auf die Frage nicht ableiten lässt. Das Beispiel im Artikel spricht nicht von einer Qualifikation ("qualifizierte Körperverletzung"). Ohne weitere Kenntnissse der Fragestellung gebe ich einen Impuls (deswegen "?") zur Beantwortung, der da lautet: die Polizei geht nicht nur von einer vermutlichen Bande aus, sondern hat einen "Nachweis/Bestätigung" ("durch Gerichtsurteil?"), dass es eine Bande ist.--Wikiseidank (Diskussion) 12:46, 28. Sep. 2015 (CEST)[Beantworten]
Das ist doch Geschwurbel. Qualifikation und gemeinschaftlich bzw. als Bande begangene Straftaten sind zwei unterschiedliche Aspekte und haben erstmal garnix miteinander zu tun. Als Bande begangene Straftaten werden im StGB auch als solche bezeichnet. Ich kann nicht erkennen, dass der Artikel Qualifikation (Strafrecht) überarbeitungsbedürftig ist. Das ist doch nachvollziehbar: Eine Form der qualifizierten Straftat ist z.B. die besondere Schwere der Tat. Bisweilen wird das sogar durch die Aufteilung eines Paragraphen dezidiert ausgeführt. Beispiel: § 244 StGB (Diebstahl mit Waffen; Bandendiebstahl; Wohnungseinbruchdiebstahl) und § 244a StGB (Schwerer Bandendiebstahl): Hier ergibt sich die Qualifikation, also der schwere Bandendiebstahl u.a. aus der Schwere des Delikts Diebstahl. Es wird ja direkt auf den § 243  StGB (Besonders schwerer Fall des Diebstahls) Bezug genommen. Die Qualifikation besteht hier u.a., wenn das gestohlene Gut aus einem verschlossenen Behältnis gestohlen wird oder zum Zweck des Diebstahls in Räume eingebrochen wird und entsprechendes Werkzeug zum Einsatz kommt. Wenn man diese Tat mit einem Diebstahl vergleicht, bei dem drei Täter Äppel aus der Straßenauslage eines Obsthändlers klauen kommt man, wenn man jetzt noch ein kleines bißchen die eigenen grauen Zellen in Bewegung setzt, dem Verständnis von Qualifikation im strafrechtlichen Sinne auf die Spur. Und wenn man die Bilder von Überwachungskameras im Kopf hat, wenn innert kürzester Zeit eine Bande mit Hilfe eines Autos und eines Stahlseils die Sicherungen eines Geldautomaten überwinden und Abräumen und Verschwinden kann, bevor man noch Stopp Polipopp gesagt hat (Beispiel1, Beispiel 2), dann kann man auch ahnen, was die Polizei nach Angaben des Fragestelles (unter anderem) meinen könnte. (Für alle, die sich schwertun: Die Qualifikation bei dem Beispielen ist nicht, dass die so effizient zusammenarbeiten und offenbar wissen, was sie tun müssen sondern der Gebrauch von Werkzeug, das Eindringen in verschlossene Räume und das Aufbrechen von Behältnissen. Ob es sich um Banden handelt muß allerdings noch nachgewiesen werden, das ergibt sich nicht aus den Videoclips.) --91.44.83.12 14:26, 28. Sep. 2015 (CEST)[Beantworten]
In Berlin kümmert sich das LKA 41 um qualifizierte Bandenkriminalität. Und zur Ausgangsfrage: qualifizierte Bandenkriminalität ist nicht das Gegenteil von unqualifizierter Bandenkriminalität, sondern hat seine Ursache in der juristischen Bedeutung von "qualifiziert".--Wikiseidank (Diskussion) 15:49, 28. Sep. 2015 (CEST)[Beantworten]

Magnetfeld der Sonne

Ich versuche gerade, das komplizierte Magnetfeld der Sonne und die heliosphärische Stromschicht zu verstehen. Mit eurer Hilfe kann es vielleicht OMA-tauglicher werden. Meine Fragen hierzu:

  1. Man kann die Sonne, anders als die Erde, ja anscheinend nicht als ungefähren Stabmagneten (Dipol) betrachten. Wie sieht also das Magnetfeld der Sonne aus? Wie verlaufen diese durch die differenzielle Rotation "aufgewickelten" Feldlinien im Raum?
  2. Wie muss man sich das vorstellen, dass das nördliche und das südliche Magnetfeld getrennt voneinander sind?
  3. Verbinden beim Sonnenmagnetfeld die Feldlinien nicht die beiden Hemisphären? (Wo gehen sie dann hin?)
  4. Spielt die Neigung der Polachse der Sonne gegen die Ekliptik (7,25°) eine Rolle für die Parker-Spirale?
  5. Wenn ja, warum? Die Ekliptik ist doch nur die Ebene der Planeten und kann doch dem Sonnenwind egal sein, er breitet sich doch radial in alle Richtungen aus?
  6. Geschieht die Umpolung des Sonnenmagnetfelds zum Höhepunkt der Aktivität oder zum Minimum der Aktivität im Sonnenfleckenzyklus?

--Neitram  11:55, 28. Sep. 2015 (CEST)[Beantworten]

6. ist beantwortet, die (vor)letzte Umpolung war 2001, also zum Aktivitätsmaximum. Und anscheinend ist das mit den Umpolungen auch ziemlich irregulär, mit zeitweise zwei Nordpolen und wandernden Polen. [20] --Neitram  14:00, 28. Sep. 2015 (CEST)[Beantworten]

Wasser auf dem Mars - schon wieder

Wegen der Ankündigung der Nasa heute abend spekulieren Journalisten ob man eventuell Wasser auf dem Mars gefunden hätte. Meine Frage dazu: Wie oft denn noch? Hat man nicht inzwischen vier oder fünf mal in Folge Wasser auf dem Mars gefunden. Ich meine alleine die Bilder von den mit Eis bedeckten Polkappen sind doch uralt und es hat ja wohl bisher noch keiner bezweifelt dass es sich dabei um Wasser handelt. --87.140.192.0 12:20, 28. Sep. 2015 (CEST)[Beantworten]

Zum Letzten: lange Zeit nahm man an, dass die Polkappen des Mars nur aus Trockeneis bestünden, nicht Wassereis. --Neitram  13:37, 28. Sep. 2015 (CEST)[Beantworten]
Ja aber inzwischen nicht mehr. Wasser auf dem Mars finden ist wie Sand in der Sahara finden. Es ist absolut nichts neues oder überraschendes. --87.140.192.0 13:59, 28. Sep. 2015 (CEST)[Beantworten]
Hast du mal einen Link? Ich habe nur Spekulationen über flüssiges Wasser gefunden. --Eike (Diskussion) 14:02, 28. Sep. 2015 (CEST)[Beantworten]
z.B. [21]. Gibt aber mehrere. --87.140.192.3 14:46, 28. Sep. 2015 (CEST)[Beantworten]
Recht spät im Text, zugegeben, aber auch da ist von "flüssigem Salzwasser" die Rede. --Eike (Diskussion) 14:48, 28. Sep. 2015 (CEST)[Beantworten]
"...dass in diesen Rinnen entlang steiniger Hänge Salzwasser fließt..." -- es geht um oberflächlich in Rinnen fließendes flüssiges Wasser. --Neitram  15:20, 28. Sep. 2015 (CEST)[Beantworten]
Das wäre doch immer noch nichts neues. Abflussrinnen die nur durch kürzlich fließendes Wasser entstanden sein können sind doch auch schon seit Jahren in Dutzenden Variationen bekannt. Wenn der Rover nicht gerade in einen See gefahren und dort versunken ist gäbe es jetzt nichts zum Thema Wasser das noch überraschen könnte. --87.140.193.0 16:44, 28. Sep. 2015 (CEST)[Beantworten]
Dieser Artikel aus dem Guardian liefert schon genaueres. Es geht um Wasser, das jetzt im Sommer fliesst. Bisher gab es meines Wissens nur Strukturen, die in der Vergangenheit durch Wasser entstanden sind. Bei den Polkappen ging man, glaube ich, immer davon aus, dass das Wasser direkt von der Eis- in die Dampfphase und umgekehrt uebergeht. --Wrongfilter ... 17:27, 28. Sep. 2015 (CEST)[Beantworten]
Nein es gab vor mindestens zwei Jahren bereits Bilder die in unterschiedlichen Jahrezeiten aufgenommen wurden wo man auf einen einen glatten Hügel und auf dem nächsten einige Monate später aufgenommen deutlich Spuren von geflossenem Wasser sieht. - Die Nachricht ist inzwischen veröffentlicht. Und wie erwartet ist es eine Variation von "Weiterhin Wasser auf dem Mars vorhanden". --87.140.193.2 18:17, 28. Sep. 2015 (CEST)[Beantworten]

Mal nebenbei gefragt: Ist das mal wieder eine Fehlleistung der Journalisten oder sind es bereits die Nasa-Leute selber, die "Beweise" nicht von "Vermutungen" unterscheiden können? Ich hab verschiedene Medienberichte gelesen und überall geht das kreuz und quer durcheinander: Mal heißt es "Beweise", dann wird im Konjunktiv weitergeschrieben: "es könnte sich um ... handeln" oder "es gibt Hinweise auf..." Oder ist Planetenforschung keine richtige Wissenschaft, wo man wenigstens einigermaßen zwischen Beweis und Vermutung unterscheidet? --Fogelfau (Diskussion) 20:10, 28. Sep. 2015 (CEST)[Beantworten]

Hier nur mal ein Beispiel für das Durcheinander, das ich eins drüber angesprochen hab. Von einem seriösen Medium, nämlich tagesschau.de (Hervorhebungen von mir): "Die US-Raumfahrtbehörde NASA hat nach eigenen Angaben Beweise, dass es auf dem Mars flüssiges Wasser gibt. Darauf deuten Messdaten der Raumsonde "Mars Reconnaissance Orbiter" hin. Salziges Schmelzwasser könnte demnach im Marssommer Bäche bilden." Wie kann etwas auf "Beweise", die man hat, "hindeuten"? Was ist das für eine Aussage? Mit dem darauffolgenden "könnte" wird dann klar, dass es sich um Spekulation handelt. Und so ungefähr sehen sämtliche Medienberichte zum Thema aus. --Fogelfau (Diskussion) 20:57, 28. Sep. 2015 (CEST)[Beantworten]

Hier ist die Originalmeldung. Da wird über flüssiges Wasser auch nur spekuliert. Das können genausogut auch hydratisierte Salze (Kristallwasser) sein. --Rôtkæppchen₆₈ 21:44, 28. Sep. 2015 (CEST)[Beantworten]
"Beweise" sind warscheinlich ein Uebersetzungsfehler. Siehe z.B. hier: "provide the strongest evidence yet" wuerde ich uebersetzen mit "stellen die bis jetzt staerksten Hinweise dar". --Nurmalschnell (Diskussion) 11:55, 29. Sep. 2015 (CEST)[Beantworten]
Ich würde das mit Indiz übersetzen, siehe auch en:wikt:evidence. --Rôtkæppchen₆₈ 15:05, 29. Sep. 2015 (CEST)[Beantworten]
in der en im Artikel Water on Mars steht: Water on Mars exists today almost exclusively as ice, though it also exists in small quantities as vapour in the atmosphere and occasionally as low volume liquid brines in shallow Martian soils und weiter Some liquid water may occur transiently on the Martian surface today but only under certain conditions. da ist der Artikel in Nature. --just aLuser (Diskussion) 09:43, 30. Sep. 2015 (CEST)[Beantworten]

Unterschied Hochnebel und Bewölkung

Ich hatte Pech: Der Vollmond war praktisch die ganze Nacht über zu sehen (naja, während ich schlief, habe ich es nicht so genau beobachtet), aber als ich dann gegen halb drei in die Puschen kam, sah ich (Norddeutschland, Weser) gerade noch den Eklipsenanfang, und ab da zog es sich zu und hatte nur ganz selten winzige Lücken, bis der Mond wieder untergegangen war. Das, was sich da so hartnäckig ins Sichtfeld schob, hätte ich ganz naiv als "Bedeckung" oder "Bewölkung" bezeichnet, aber die Medien belehren mich, daß die Wetterdienste es "Hochnebel" nennen - was ist denn eigentlich der Unterschied bzw. richtig? (Ich würde "Nebel" für unstrukturiert halten, aber das Zeugs da oben hatte relativ klare Konturen, deshalb hielte ich "Wolken" eigentlich für passender. Durch die Lücken und Spalten kamen recht klar Sterne hindurch, Orion war zu sehen, längere Zeit auch die Venus, nur im Westen war halt durchgehend "Waschküche". Höhe (Wolkenuntergrenze) kann ich nicht gut schätzen - es mögen 5000 m gewesen sein.) --92.224.158.60 12:16, 28. Sep. 2015 (CEST)[Beantworten]

Ich habe in der Schule gelernt, dass die Definition für Wolken auch auf Nebel zutrifft. Und jetzt such doch bitte einmal im Artikel Nebel nach dem Wort Wolken! Demnach hast du Recht, bei Wolken von Hochnebel zu reden ist also etwa so falsch und richtig wie Nebel als Bodenwolken zu bezeichnen. Hochnebel hört sich so an, als würde man in den Bergen in Wolken/im Nebel herumwandern. --MannMaus 13:32, 28. Sep. 2015 (CEST)[Beantworten]
Oder hat der Lehrer von Bewölkung geredet? Ach, wir haben hier auch einen Artikel über Hochnebel. Dann waren es aber keine 5000 m. --MannMaus 13:40, 28. Sep. 2015 (CEST)[Beantworten]
Eine praktische Lösung könnte wohl darin bestehen, herauszufinden, welcher Bedeckungsgrad und welche Wolkenuntergrenze für den Flughafen Bremen und dessen Umgebung in der Zeit von 3-6 Uhr registriert wurde. Leider weiß ich nicht, wo man das nachsehen kann. (nicht signierter Beitrag von 92.224.74.251 (Diskussion) 17:38, 28. Sep. 2015 (CEST))[Beantworten]
Ich entsinne mich, dass der Wetterfrosch im theoretischen Unterricht in der Flugschule, das Wort Hochnebel nicht hören wollte, weil das richtigerweise Stratuswolke sei. Nebel und Wolken seien gleich, ersterer zeichnet sich durch Bodenkontakt aus, Wolken sind oben und geben unten Sicht frei. Die im Fernsehen sprechen trotzdem von Hochnebel, ist vermutlich eine andere Schule.--2003:68:ED61:E400:2458:2CB9:F0EA:CCC3 19:43, 28. Sep. 2015 (CEST)[Beantworten]

Auseinandersetzung durch Schock?

Gibt es in der Psychologie oder Argumentationstheorie oder Rhetorik irgendeinen Namen für den Effekt, dass uns etwas, dass uns schockiert, viel mehr aufrüttelt als eine sachlich vorgetragene Kritik? Gibt es Untersuchungen darüber, ob das unsere Meinung nicht sogar stärker verändert? --92.217.165.168 13:45, 28. Sep. 2015 (CEST)[Beantworten]

Wird schwierig sein, dafür einen Effekt (mit einem Namen) zu finden. Ich biete aber mal Desillusionierung an. Gibt keinen Artikel darüber. Über Konfrontationstherapie gibt es einen Artikel, aber die arbeitet nicht mit Schock, sondern mit langsamen Herantasten. Vielleicht findest du auch im Artikel Aufmerksamkeit Mechanismen, die einen solchen Effekt begründen. -- Amtiss, SNAFU ? 14:19, 28. Sep. 2015 (CEST)[Beantworten]
Ich versuche deine Frage zu verstehen -- mir erscheint sie tautologisch. Etwas, das uns mehr aufrüttelt, rüttelt uns mehr auf. --Neitram  15:24, 28. Sep. 2015 (CEST)[Beantworten]
Da fällt mir psychologische Kriegsführung ein.--2003:68:ED61:E400:2458:2CB9:F0EA:CCC3 19:45, 28. Sep. 2015 (CEST)[Beantworten]
+1 "versuche deine Frage zu verstehen" Bitte Reformulierung der Frage - denn: Auch eine sachlich vorgetragene Kritik (der Sender ist sachlich) kann schockiert aufgenommen werden (der Empfänger ist durch die Wucht der rationalen Argumente emotional schockiert).
V1: Könnte es sein, dass gemeint ist, dass eine emotional vorgetragene Kritik/Aussage mehr Wirkung (positiv oder negativ) haben könnte, als eine rein rationale vorgetragene Aussage? [Das hängt ganz von der Situation ab...]
V2: Meinst du blöder Hansl, dass ein Schocker-Anrumpeln einem mehr auf den Sack gehn tut, als ein eierköpfiges Dahergelabere?
(V1 und V2 haben dieselbe Aussage, werden nur mit anderen Gewürzen vorgetragen...) ;-) 213.169.163.106 19:51, 28. Sep. 2015 (CEST)[Beantworten]
Ich glaube kaum, dass die Aufdrucke auf Zigarettenschachteln dazu geführt haben, das nennenswert weniger geraucht wird. Zukünftig sollen Bilder von Raucherbeinen und Raucherlungen auf den Packungen abgedruckt und so die Raucher schockiert werden. Ob das stärker wirkt, als die Info, dass die Anzahl Deiner Spermatozyten zurückgeht?--Optimum (Diskussion) 20:43, 28. Sep. 2015 (CEST)[Beantworten]
Die Frage ist zu allgemein gestellt für eine konkrete Antwort. Wenn Du das präzisieren könntest, könnte man gezielter suchen, was Soziopsychologen dazu schreiben. --94.219.10.213 22:50, 28. Sep. 2015 (CEST)[Beantworten]
In dem Stück Davor von Günter Graß, uraufgeführt Anfang 1969 im Berliner Schiller-Theater will angesichts des Grauens im Vietnamkrieg der Oberschüler Scherbaum seinen Dackel Max auf dem Berliner Kurfürstendamm verbrennen, damit - wie Der Spiegel referiert - „die "Topfhüte" auf der Kempinski-Terrasse begreifen, wie Napalm wirkt, und ihnen "der Kuchen aus dem Gesicht fällt".“ (Mit den „Topfhüten“ sind die mehr oder weniger wohlhabenden Damen gemeint, die das elegante Café damals frequentierten.) Die Geschichte findet auch in den erst später im gleichen Jahr bei Luchterhand erschienenen Roman örtlich betäubt Eingang. ([22]) Tatsächlich hat Graß, der unter anderem auch als Gutachter auftrat in einem Prozeß gegen Mitglieder der Kommune I (sogenannte „Moabiter Seifenoper“) wegen eines aus Anlaß einer Brandkatastrophe in Brüssel provokanten Flugblatts mit dem satirischen Aufruf, Kaufhäuser anzuzünden („Unsere belgischen Freunde haben endlich den Dreh heraus, die Bevölkerung am lustigen Treiben in Vietnam wirklich zu beteiligen: sie zünden ein Kaufhaus an, dreihundert saturierte Bürger beenden ihr aufregendes Leben und Brüssel wird Hanoi. (...) Burn ware-house burn! Kommune I, 24. Mai 1967“) den Plot seiner Dackel-verbrennen-Geschichte aus der Wirklichkeit geschöpft.
Ich lese: „Eine aufsehenerregende Ankündigung gab es wohl am 30. Juli 1968 tatsächlich: „Um zu beweisen, daß der Vietnamkrieg - würde er nur gegen Hunde, anstatt gegen Menschen geführt - schon nach kurzer Zeit durch den Protest der internationalen Tierschutzvereine und tierliebender Richter gestoppt würde, kündigt die Internationale der Kriegsdienstgegner die öffentliche Verbrennung eines Hundes an. Daraufhin erhebt sich eine Protestwelle; unter anderem macht eine Frau das Angebot, lieber sich als den Hund verbrennen zu lassen.“ Kraushaar, Wolfgang: „Notizen zu einer Chronologie der Studentenbewegung“, in Mosler, Peter (Hg.), Was wir wollten, was wir wurden. Studentenrevolte - zehn Jahre danach. Reinbek bei Hamburg: Rowohlt 1982, S. 249-295, hier S. 282.“ (zit.n. Jan Henschen: Die RAF-Erzählung: Eine mediale Historiographie des Terrorismus. Kultur- und Medientheorie. transcript Verlag, Bielefeld 2014, ISBN 9783839423905, S. 83, Anm. 179).
Wohlgemerkt: Graß surfte da auf einer Welle, die nicht die Seine ist. Das Time Magazine konstatierte: „Grass ist ein fanatischer Anhänger der Mäßigung.“ und der Verlag wirbt noch immer gerne mit diesem Statement. Bereits 1969 attestierte ihm Peter Schneider: „Um nicht ohnmächtig zu sein, hat er sich mit der etablierten Macht verbündet.“ (zit.n. Der Spiegel Nr. 8/1969, 17. Februar 1969, S. 145. Es gehört zu den kulturhistorischen Treppenwitzen, dass Peter Scheider nicht viel später Graß auf noch elendere Weise nachfolgte.) Beispielhaft geht es also um die Methode der Provokation. Der äußerst dünne und eher beliebige Artikel gibt dazu jedoch nicht viel her. --91.44.83.12 23:58, 28. Sep. 2015 (CEST)[Beantworten]
Ergänzend zum provokativen Spektakel: Es gibt das Damaskuserlebnis, mit dem sich die Selbstsicht verändert. Und allgemein das Aha-Erlebnis, das das Überraschende der Erkenntnis hervorhebt und das Schlüsselerlebnis [23], das eine lebensverändernde Veränderung oder Erkenntnis auslöst und zu psychischer Belastung oder Entlastung führen kann. Für all diese Formen einer Entwicklung ist jedoch keine schockierende oder traumatische Erfahrung mit der fraglos größeren Nachhaltigkeit zwingende Voraussetzung. Ich halte den Eindruck, den eine Auseinandersetzung oder Erfahrung hinterlassen kann für ein quasi stufenloses Resultat. Dabei kann ein „cooler“, sachlicher Umgang möglicherweise sogar mehr beeindrucken als eine dramatische Inszenierung des Selbstmitleides oder ein ausgeflipptes brüllendes Affenmännchen, das mit dem Messer auf mich losgeht. --91.44.83.12 12:43, 29. Sep. 2015 (CEST)[Beantworten]

Polizei (Deutschland): Wieviele Polizisten gibt es?

Aus dem Artikel:

Angehörige der Polizei sind zumeist Polizeibeamte und Polizeiangestellte. Im Jahr 1992 gab es 219.887 Länderpolizisten und 26.424 Bundespolizisten und somit insgesamt 246.311 Polizisten in der Bundesrepublik. Auf einen Polizisten kommen derzeit etwa 330 Bundesbürger.

Im Vollzugsdienst sind in der Regel Polizeivollzugsbeamte zuständig. Diese bilden mit 265.000 Personen bundesweit den hauptsächlichen Personalkörper. Die Beamten haben mitunter eine spezielle berufliche Funktion bei der Polizei.

Die Zahl der Polizisten in Deutschland lag in den Jahren 1997 bis 2012 zwischen 229.000 und 252.230.

Gibt es jetzt 246.311 Landes- und Bundespolizisten und zusätzlich 265.000 Polizeivollzugsbeamte?

Wenn die Zahl der Polizisten in den letzten Jahren bei zwischen 229.000 und 252.230 lag, sind 265.000 Polizeivollzugsbeamten dann keine Polizisten? Ich sehe da gerade den Wald vor lauter Bäumen nicht. --2003:76:E6F:E435:4D52:5FC0:F791:29FF 15:28, 28. Sep. 2015 (CEST)[Beantworten]

also die 246.000 sind von 1992. die zahl ist über 20 jahre alt und damit eher museal. herkunft unbelegt. woher die zweite kommt, weiss ich nicht und aus welchem jahr sie darstellt, auch nicht. sie erscheint mir aber im verhältnis zu den anderen sehr hoch und die daten bei statista geben das auch nirgends her. allerdings scheint die zahl dank wiikipedia schon fröhlich durch presse und internet gewandert zu sein. die dritte wiederum ist vm statistischen bundesamt und als solche sicher die validistete und für heute aussagekräftigste. -- southpark 19:24, 28. Sep. 2015 (CEST) (abgesehen davon wäre ich dafür die 265.000 zu löschen, sofern nicht jemand eine brauchbaren beleg dafür findet)[Beantworten]
Ich lese hier bei statista: 2012: 243.982, 2011: 243.201, 2010: 243.625, (...) 2005: 248.188 (...) 2000: 250.178 (...) 1997: 229.424. Der Spitzenwert liegt 1999 bei 252.230 --91.44.83.12 20:55, 28. Sep. 2015 (CEST) Wert von 1999 korrigiert (nicht 52.230 sondern 252.230) --2003:45:4651:C22:C35:DB92:B15D:E7BE 12:04, 29. Sep. 2015 (CEST) (hier auch als 91.44.83.12 unterwegs)[Beantworten]
Die Frage ist doch, etwa 250000 oder etwa 500000. Wohl eher das Erstere. --195.200.70.46 09:00, 29. Sep. 2015 (CEST)[Beantworten]

Richtlinie Licht-Signalanlagen

In Frankfurt am Main an der Ecke Adalbertstrasse/Grosse Seestrasse beträgt die Grünphase für Fußgänger nur 5 Sekunden. Gibt es ein subjektives Grundrecht für Alte, Gehbehinderte und Rollstuhlfahrer die Adalbertstraße zu überqueren ?

--77.176.38.166 15:32, 28. Sep. 2015 (CEST)[Beantworten]

Übliches Missverständiss. Die Grünphase muss nicht so lange sein wie man zum Überqueren der Strasse benötigt, sondern zeigt nur an wie lange man loslaufen darf. Die Ampel muss für "feidliche" Fahrstrassen so lange auf Rot stehen, bis der Fussgänger die Strasse geräumt hat (Der Fussgänger hat ddan aber auch schon rot). Kurzum 5 Sekunden Loslaufzeit sind genügend lange. In anderen Ländern würde die Ampel nach 5 Sekunden anfangen zu blinken oder auf gelb wechseln. --Bobo11 (Diskussion) 15:38, 28. Sep. 2015 (CEST)[Beantworten]
(BK)§37 Abs. 2 Nr. 5 Satz 3 StVO gilt auch für Alte, Gehbehinderte und Rollstuhlfahrer. --Rôtkæppchen₆₈ 15:39, 28. Sep. 2015 (CEST)[Beantworten]
Ist diese kurze Grünphase nicht dermaßen einschüchternd, daß in der Folge Alte, Gehbehinderte und Rollstuhlfahrer das Recht, diese Strasse zu queren, subjektiv ( für sich genommen ) gar nicht mehr in Anspruch nehmen ?! 77.176.38.166 15:48, 28. Sep. 2015 (CEST)[Beantworten]
Nein. Das übliche Verhalten ist Knopf drücken, warten, bei Grün losmarschieren. Abwarten oder Messen, wie lange Grün bleibt, ist vollkommen unüblich. --Rôtkæppchen₆₈ 15:51, 28. Sep. 2015 (CEST)[Beantworten]
Dazu musst du nicht abwarten. Einmal mit offenen Augen über die Straße laufen und am nächsten Tag wiederkommen reicht völlig. --Eike (Diskussion) 15:53, 28. Sep. 2015 (CEST)[Beantworten]
(BK) Mit dem "subjektiven Grundrecht" kann ich nichts anfangen, aber ich vermute, ich bin nicht der einzige, der sich bemüht, von der Straße zu sein, bevor's wieder rot wird. --Eike (Diskussion) 15:52, 28. Sep. 2015 (CEST)[Beantworten]
Artikel zum Thema: Zwischenzeit (Verkehrstechnik) --Rudolph Buch (Diskussion) 15:54, 28. Sep. 2015 (CEST)[Beantworten]
Auf der Adalbertstraße leider nein, auf der Heribertstraße aber ja. Allerdings nur Mittwochs, wenn Wasser drinne ist.
Jetzt mal im Ernst: Wie lange eine Fußgängerampel Grün zeigt, ist vor allem eine Frage der Straßenbreite. Man geht davon aus, dass sich ein durchschnittlicher Fußgänger im Schnitt mit 1,2 Metern pro Sekunde vom Fleck bewegt. Die Grünphase soll mindestens so lange dauern, dass man die Mitte der gegenüberliegenden Fahrbahn erreicht - also etwa drei Viertel der Straße überqueren kann. Die Intervalle sind folglich nicht darauf ausgelegt, die ganze Straße bei Grün hinter sich zu bringen.
Für jeden Fahrstreifen sieht die StVO eine Breite von 2,75 m und 3,75 m vor, je nach zulässiger Höchstgeschwindigkeit: von Spielstraße bis Autobahn.
Die Große Seestraße ist in jede Richtung einspurig ohne dass sonst noch irgendwas wäre. Mit großzügig gerechnet 3 m breiten Fahrstreifen ist die Straße also alles in allem 6 m breit; eine Grünphase von 5 Sekunden ist da, wenn man die obigen Kriterien zu Grunde legt, schon großzügig.
Die Adalbertstr. ist an der Stelle, an der die Fußgängerampel ist, in jede Richtung einspurig. In der Mitte liegen zusätzlich zwei Straßenbahngleise, deren Raumbedarf jeweils schmaler ist als ein Fahrstreifen. Wenn die Straße geschätzt ungefähr 10m breit ist, wären demnach mindestens etwa 6 Sekunden Grünphase nötig. --88.130.106.69 16:06, 28. Sep. 2015 (CEST)[Beantworten]
Interessant und einleuchtend. Kann man das mit dem dreiviertel der Straße irgendwo nachlesen? Und muss es statt "Straße" nicht "Fahrbahn" lauten? Der Bürgersteig gehört ja auch zur Straße... -- Ian Dury Hit me  20:17, 28. Sep. 2015 (CEST)[Beantworten]
In meinem Fall stammen die drei Viertel aus den Durchführungsverordnungen der Stadt München. Der Bereich, um den es dabei geht, ist der Abstand von Bürgersteig zu Bürgersteig. Im konkreten Fall also die Fahrbahn inklusive des Randstreifens (also hier inklusive der Entwässerungsrinne). --88.130.96.6 14:15, 29. Sep. 2015 (CEST)[Beantworten]

Filmt doch mal dort einen gehbedinderten Menschen so ca. 70jährigen Menschen beim Überqueren der Ampelphase in situ - also dort. (Hinweis: Aus zwei Perspektiven! Auflösung tut nichts zur Sache - Handy reicht) Die heutige Jugend ist doch sonst nicht so einfalllos. Besten Dank und dann ab zu youtube! --80.187.102.107 19:04, 28. Sep. 2015 (CEST)[Beantworten]

Die Übergangszeit ist nur ein Teil der Sicherungszeit oder Schutzzeit, in der RiLSA zum Sichern und Räumen definiert und muss dafür ausgelegt sein. In der Verkehrserziehung heißt es, nicht losrennen, wenn die Ampel auf rot springt, solange Du auf der Straße bist, muss die Querung möglich sein. Du darfst nur nicht mehr von Verkehrsinseln oder Gehsteigen auf die Fahrbahn wechseln, bzw. die Querung einer Fahrbahn (mit wievielen Spuren auch immer) beginnen. Andere Länder, andere Sitten. In USA sind die Signalgeben hinter der Kreuzung bzw. querender Straße wie hier bei den Fußgängern. Dadurch ist der Blick auf die Straße gerichtet und die Vorfahrt der räumenden Fahrzeuge ist zu gewähren. --Hans Haase (有问题吗) 19:07, 28. Sep. 2015 (CEST)[Beantworten]
Wie alt bist Du denn? Was spricht dagegen, dies mal (aus zwei Pesrspektiven) zu filmen? --80.187.102.107 19:49, 28. Sep. 2015 (CEST)[Beantworten]
Ich wüsste nicht, wozu das gut sein sollte. --88.130.96.6 19:59, 28. Sep. 2015 (CEST)[Beantworten]
Wieso nicht eine Genehmigung des Darstellers einholen und hier hochladen? Wäre doch ein schönes Video für den Artikel Zwischenzeit (Verkehrstechnik) oder Übergangszeit. BU: Beispiel für eine zu kurze/ausreichende [hier Lemma einsetzen]. -- Ian Dury Hit me  20:17, 28. Sep. 2015 (CEST)[Beantworten]

kündigung für tanzschule

--2A02:908:D411:1F80:2531:7A5C:94D6:27F5 16:18, 28. Sep. 2015 (CEST)[Beantworten]

Niemand darf gekündigt werden, weil er eine Tanzschule besucht.--JTCEPB (Diskussion) 16:20, 28. Sep. 2015 (CEST)[Beantworten]
Du besuchst während der Arbeitszeit eine Tanzschule. Multiple choice: (a) Dein Arbeitgeber freut sich darüber, dass du ordentlich tanzen lernst? (b) Dein Arbeitgeber kündigt das Arbeitsverhältnis? --Pp.paul.4 (Diskussion) 17:28, 28. Sep. 2015 (CEST).[Beantworten]
Multiple Choice (c): Dein Arbeitgeber bezahlt dich dafür, denn er ist die Tanzschule. --88.130.96.6 20:03, 28. Sep. 2015 (CEST)[Beantworten]
42. Ehmm,ich mein: Keine Frage, ne? Ansonsten: Kündigungsfrist "zum nächstmöglichen Termin", schriftlich, leicht paranoid: mit Zeugen abgeben. --88.130.106.69 16:22, 28. Sep. 2015 (CEST)[Beantworten]
(BK) Es fehlt die konkrete Frage. Es ist auch nicht klar, ob mit "für" gemeint ist, dass der Tanzschulenbesuch als Kündigungsgrund angegeben wurde, oder dass es um eine Kündigung für einen Vertrag mit einer Tanzschule geht. @JTCEPB: wenn man während der Arbeitszeit eine Tanzschule besucht, kann man dafür eventuell sehr wohl gekündigt werden. --MrBurns (Diskussion) 16:26, 28. Sep. 2015 (CEST)[Beantworten]
Das ist doch Haarspalterei. ein SmileysymbolVorlage:Smiley/Wartung/:p --JTCEPB (Diskussion) 07:31, 29. Sep. 2015 (CEST)[Beantworten]
"Kündigung" und "Tanzschule" schreibt man groß. Wenn die Tanzschule zuviel Lärm macht, darf der Vermieter ihr kündigen. --83.79.9.134 17:32, 28. Sep. 2015 (CEST)[Beantworten]
Der allererste Google-Treffer bei „kündigung für tanzschule“ könnte womöglich hilfreich sein. --Pp.paul.4 (Diskussion) 17:43, 28. Sep. 2015 (CEST)[Beantworten]

Hier gehts zur Eingabemaske ihrer Frage -- 141.30.146.65 18:10, 29. Sep. 2015 (CEST)[Beantworten]

Dieser Abschnitt kann archiviert werden. Keine Frage gestellt. --88.130.109.193 18:34, 29. Sep. 2015 (CEST)

Henne-Ei-Problem

Im Artikel Henne-Ei-Problem heißt es, Aristoteles habe das Problem formuliert, was mit einem Zitat von Fénelon auch belegbar ist; den Beleg hatte ich am 8. Februar 2011 eingefügt. Aktuell wird für das Zitat ein neuerer Beleg von Blavatsky angegeben. Die Stelle bei Blavatsky liegt mir nicht vor; der Wortlaut scheint auf Fénelon zu weisen. Es fehlt die Belegstelle bei Aristoteles. Kennt die jemand? --Pp.paul.4 (Diskussion) 17:13, 28. Sep. 2015 (CEST)[Beantworten]

Im englischen Artikel en:Chicken or the egg wird für Blavatsky diese Quelle angegeben (nach egg volltextsuchen). Sie bezieht sich auf Plutarchs Gastmahl der sieben Weisen. --Rôtkæppchen₆₈ 00:12, 29. Sep. 2015 (CEST)[Beantworten]
An anderer Stelle zitiert Blavatsky ausdrücklich Aristoteles, wörtlich nach Fénelon. Und tout le monde zitiert nach den beiden, sogar in philosophischen Aufsätzen, und keiner sagt, wo das bei Aristoteles zu lesen ist. Schon ärgerlich! Ich schreibe später mal einen Gräzisten an. Grüße Dumbox (Diskussion) 07:36, 29. Sep. 2015 (CEST)[Beantworten]

Wann hat man schon mal die Gelegenheit, die Werke von Aristoteles zu lesen?
Auf der Suche nach dem Wahrheitsgehalt der Aussage eines spirituellen Franzosen über das Werk eines klar denkenden Griechen:

  • The Athenian Constitution <= NICHTS mit egg
  • Categories <= NICHTS mit egg
  • On Dreams <= NICHTS mit egg
  • On the Gait of Animals <= 1 x egg
  • On Generation and Corruption <= NICHTS mit egg
  • On the Heavens <= NICHTS mit egg
  • The History of Animals <= Das überlasse ich Euch (habe bereits eine Meinung)
  • On Interpretation <= NICHTS mit egg
  • On Longevity and Shortness of Life <= NICHTS mit egg
  • On Memory and Reminiscence <= NICHTS mit egg
  • Metaphysics <= NICHTS mit egg
  • Meteorology <= 1 x egg (!)
  • On the Motion of Animals <= NICHTS mit egg
  • Nicomachean Ethics <= NICHTS mit egg
  • On the Parts of Animals <= Das überlasse ich Euch (habe bereits eine Meinung)
  • Physics <= NICHTS mit egg
  • Poetics <= NICHTS mit egg
  • Politics <= NICHTS mit egg
  • Posterior Analytics <= NICHTS mit egg
  • Prior Analytics <= NICHTS mit egg
  • On Prophesying by Dreams <= NICHTS mit egg
  • Rhetoric <= NICHTS mit egg
  • On Sense and the Sensible <= NICHTS mit egg
  • On Sleep and Sleeplessness <= NICHTS mit egg
  • On Sophistical Refutations <= VIEL b-egg-ing
  • On the Soul <= NICHTS mit egg <Enttäuschung>
  • Topics <= VIEL b-egg-ing
  • Virtues and Vices <= NICHT zugänglich
  • On Youth and Old Age, On Life and Death, On Breathing <= NICHTS mit egg
213.169.163.106 08:50, 29. Sep. 2015 (CEST)[Beantworten]
Der französischsprachige Artikel fr:Paradoxe de l'œuf et de la poule#Philosophie quillt geradezu über von Aristoteles-Zitaten. Im englischen Artikel hatte ich am 8. Februar 2011 den Fénelon-Beleg auch eingefügt; dort gilt er aktuell noch, obwohl der Artikel quasi täglich von Kreationisten bearbeitet wird und aktuell der Autodidakt und intelligenteste Mensch der Welt en:Christopher Langan (kein deutscher Artikel) dort als ultima ratio angegeben wird. Bei dem Erzbischof Fénelon würde es nicht verwundern, wenn er Gedanken, die nicht durch die christliche Schöpfungslehre gedeckt sind, bemäntelt, etwa indem er Aristoteles vorschiebt; andererseits ist das Henne-Ei-Problem anscheinend wirklich bereits antik formuliert worden, oder nicht? --Pp.paul.4 (Diskussion) 10:19, 29. Sep. 2015 (CEST)[Beantworten]
Direkte Quelle könnte sein: Benjamin Martin: Biographia philosophica, being an account of the lives, writings, and inventions, of the most eminent philosophers and mathematicians who have flourished from the earliest ages of the world to the present time. London, W. Owen, 1764 (hier). Es ist beinahe mit Sicherheit nicht von Aristoteles. Es widerspricht seiner Philosophie (vgl. Metaphysik Buch 9, 7. Für ihn wäre die finale Ursache entscheidend, d.h. die Henne würde dem Ei mit Notwendigkeit vorgehen.--Meloe (Diskussion) 12:40, 29. Sep. 2015 (CEST)[Beantworten]
Sehr, sehr schönes Fundstück. Aber es liegt zeitlich nach Fénelon, oder?
Zur Eingangsfragestellung: "Blavatsky, die Fénelon zitiert, der Aristoteles zitiert", halte ich für ungünstiger als " Fénelon, der Aristoteles zitiert" - mit dem Zusatz-Hinweis, dass die Originalstelle (noch) nicht identifizierbar ist. 213.169.163.106 13:32, 29. Sep. 2015 (CEST)[Beantworten]
Vielen Dank an alle. Ich habe den Artikel entsprechend geändert. --Pp.paul.4 (Diskussion) 13:00, 30. Sep. 2015 (CEST)[Beantworten]

Zusatzfragen

Ich habe das schöne Goethe-Zitat mit Hinweis auf deinen Beitrag bei der Auskunft im Artikel ergänzt. Der Anatom Gerardus Blasius geht uns völlig ab, während nl:Gerardus Leonardus Blasius ein ordentlicher Artikel ist. --Pp.paul.4 (Diskussion) 16:28, 30. Sep. 2015 (CEST)[Beantworten]
Bin drübergeflogen: Da geht es nur um die Wirkursache bei der Entwicklung des Huhns, und speziell, in Auseinandersetzung mit Aristoteles und einem Fabricius (Girolamo Fabrizio, nehme ich an), ob die anima des Huhn im männlichen Samen ist. Grüße Dumbox (Diskussion) 17:48, 30. Sep. 2015 (CEST)[Beantworten]

Meine Versuche mich zu registrieren

Ich wollte mich gerne bei Badoo registrieren , habe schon mehrmals Fragen zur Registrierung ausgefüllt , an meiner E-Mail Adresse scheiterte die Registrierung , da entweder meine E-Mail Adresse oder mein Passwort nicht akzeptiert wurde. Wobei Adresse und Passwort schon einige Jahre alt sind . --176.0.6.148 18:03, 28. Sep. 2015 (CEST)[Beantworten]

Wenn "E-Mailadresse und Passwort schon einige Jahre alt" sind, dann bist Du also schon registriert und kannst Dich nur nicht einloggen?--2003:57:EA79:314D:8516:6F9B:93D9:2E17 20:29, 28. Sep. 2015 (CEST)[Beantworten]

E-Mail Adresse ändern

Ich möchte meine E-Mail Adresse ändern wie mache ich das. auf meiner Adresse erscheint im Handy gmx.derz esmuß aber gmx.de geschrieben sein.

--188.105.186.245 19:18, 28. Sep. 2015 (CEST)[Beantworten]

Wahrscheinlich irgendwo in den Einstellungen im Handy. Wo genau und wie, kommt auf das Handy an (und auf das Betriebssystem, das da drauf ist). --88.130.96.6 20:05, 28. Sep. 2015 (CEST)[Beantworten]

Mein Schwein Rechner pfeift.

Guten Abend!

Seit ungefähr einer Viertelstunde pfeift mein Rechner fast ununterbrochen. Auch ein Neustart hat nicht geholfen. Das einzige, was hilft, ist ein Ausschalten des Lautsprechers, was aber wohl nicht die Lösung sein kann. Weiß jemand Rat?

Gruß

Altſprachenfreund, 20:27, 28. Sep. 2015 (CEST)[Beantworten]

Ist das der kleine Piepslautsprecher? Dann könnte es ein Temperaturproblem sein. Oder der normale Lautsprecher, über den auch Audio ausgegeben wird? Schon mal runtergefahren und neu gestartet? lesen hilft --Optimum (Diskussion) 20:33, 28. Sep. 2015 (CEST)[Beantworten]
Neugestartet ja. Ich vermute, es ist der normale Lautsprecher. Ich kenne mich aber kaum aus. Von einem Piepslautsprecher höre ich das erste Mal. Altſprachenfreund, 20:36, 28. Sep. 2015 (CEST)[Beantworten]
Temperaturwarnung (mal auslesen), Spannungsversorgung (Netzteil überfordert?), oder irgendeine blöde Taste ist festgepappt (Nicht mit Pizzafingern tippen). 77.176.20.184 20:39, 28. Sep. 2015 (CEST)[Beantworten]
Wenn du alle externen Lautsprecher, Kopfhörer und zur Not auch den Monitor ausgestöpselt hast und es immer noch piepst, ist es der interne Pieper. --Eike (Diskussion) 20:48, 28. Sep. 2015 (CEST)[Beantworten]
(BK) Temperaturen (laut PC-Wizard 2015) im Überwachungschip 26,8 °C, im Prozessorkern 1 51 °C, im Prozessorkern 2 61 °C, bei der Festplatte 53 °C. Sind das ungewöhnlich hohe Werte? Altſprachenfreund, 20:51, 28. Sep. 2015 (CEST)[Beantworten]
PS: Lautsprecher aus- und wiedereinstecken hilft auch nichts. Altſprachenfreund, 20:59, 28. Sep. 2015 (CEST)[Beantworten]
Kommt das Pfeifen aus dem Lautsprecher? --Hans Haase (有问题吗) 21:06, 28. Sep. 2015 (CEST)[Beantworten]
Pfeifen erst einmal weg. Schauen wir mal, für wie lange. Altſprachenfreund, 21:07, 28. Sep. 2015 (CEST)[Beantworten]
PS: Hans, ja, wenn man ihn absteckte, war das Pfeifen weg, bis man ihn wieder ansteckte. Altſprachenfreund, 21:09, 28. Sep. 2015 (CEST)[Beantworten]
Die Werte wären für einen uralten AMD 586 Pentium-1-kompatiblen Prozessor zu hoch. Die Festplatte sollte unter 50° C sein. Sind Kabel davor? Sorge für besseren Luftstrom. Sind die Lüftungsschlitze vorne mit Staub zugesetzt? Hast Du mehrere Festplatten im RAID? Schau die SMART-Wert der Platte(n) durch, ob eine grade am sterben ist? --Hans Haase (有问题吗) 21:18, 28. Sep. 2015 (CEST)[Beantworten]
Manchmal lügen auch nur die Messprogramme. --Rôtkæppchen₆₈ 21:28, 28. Sep. 2015 (CEST)[Beantworten]
(quetsch)"manchmal werden die Messprogramme ausgetrickst" wolltest du vermutlich eigentlich sagen... --Benutzer:Duckundwech 08:20, 29. Sep. 2015 (CEST)[Beantworten]
Ist ein einige Jahre alter Intel-P8700-2,53GHz-Prozessor. Und das Pfeifen ist ja mittlerweile zum Glück seit über zwanzig Minuten weg. Altſprachenfreund, 21:31, 28. Sep. 2015 (CEST)[Beantworten]
Hardwareprobleme sollte der Rechner m. E. über den eingebauten Pieper mitteilen; dann würde das Piepen nicht verschwinden, wenn die die Lautsprecher abziehst. Daher würde ich auf ein Softwareproblem tippen. --Eike (Diskussion) 08:32, 29. Sep. 2015 (CEST)[Beantworten]
Ein PC hat verschiedene Komponenten, die pfeifen, zirpen, piepsen oder schrillen können: Lautsprecher, Piepslautsprecher, Lüfter, Festplatten. Ein geschultes Ohr kann die Quelle leicht ermitteln. Häufig ist es ein Lüfter, der auszutauschen ist. --Pp.paul.4 (Diskussion) 10:31, 29. Sep. 2015 (CEST)[Beantworten]
Aber wieviele davon hören mit hoher Wahrscheinlichkeit auf zu piepen, wenn man den Lautsprecher abzieht, und fangen nach dem Einstöpseln wieder an...? --Eike (Diskussion) 10:38, 29. Sep. 2015 (CEST)[Beantworten]
Zum Beispiel der Lüfter, wegen einer unmessbar geringen Lageveränderung. --Pp.paul.4 (Diskussion) 10:49, 29. Sep. 2015 (CEST)[Beantworten]
Möglich? Klar. Mit hoher Wahrscheinlichkeit? Glaub ich nicht. --Eike (Diskussion) 10:50, 29. Sep. 2015 (CEST)[Beantworten]

Asklepios

Wer ist/war Asklepios Frau? (nicht signierter Beitrag von 2003:45:4B69:E280:9DAA:E218:BF57:69B1 (Diskussion | Beiträge) 20:40, 28. Sep. 2015 (CEST))[Beantworten]

Epione kommt in einigen Traditionen vor. Grüße Dumbox (Diskussion) 20:44, 28. Sep. 2015 (CEST)[Beantworten]

Welche Bedeutung hat dieses Kreuz?

Kann mir eine(r) sagen, wie dieses Kreuz heißt und welche Bedeutung es hat ?

--Calluna (Diskussion) 20:56, 28. Sep. 2015 (CEST) [Kreuz][Beantworten]

Es ist das christliche Kreuz mit der Dornenkrone, eines der Leidenswerkzeuge.--Heletz (Diskussion) 20:59, 28. Sep. 2015 (CEST)[Beantworten]
Da steht doch was unten drauf (zwei Daten 1991, 1993). Wird also ein Gedenkkreuz für irgendein Ereignis sein. Freudiges, oder böses (Unfall etc.). Wo steht es denn? Ich erkenne da im Gegensatz zu Heletz keine Dornenkrone, ich sehe 12 (Friedens-)tauben als freie/künstlerische Dekorationszutat. --AndreasPraefcke (Diskussion) 21:01, 28. Sep. 2015 (CEST)[Beantworten]
Laut EXIF-Daten wurde das Foto bei 51°19'43,69"N 8°05'00,72"E gemacht, das ist am westlichen Rand von Hellefeld. Gemäß Openstreetmap befindet sich an der Stelle ein Parkplatz und ein Picknickplatz. --Rôtkæppchen₆₈ 21:27, 28. Sep. 2015 (CEST)[Beantworten]
@Calluna: Ich würde mal ganz freundlich bei der katholischen Kirchengemeinde St. Martinus in Hellefeld nachfragen, Kontaktdaten hier. --Rôtkæppchen₆₈ 22:17, 28. Sep. 2015 (CEST)[Beantworten]

Taubensymbol und Zwölffachheit tauchen auch beim Hugenottenkreuz auf, welches hier aber kaum Pate gestanden haben dürfte. Als Unfallhäufungsstelle der Landesstrasse 839 insbesondere für Motorradfahrer gilt seit 2000 der nördlich gelegene Abschnitt Hellefelder Höhe http://www.gruene-hsk-fraktion.de/uploads/media/Antwort_an_Gruene_Motorradraser_Hellefelder_Hoehe_v._26.6.13.pdf Rosenkohl (Diskussion) 00:46, 29. Sep. 2015 (CEST)[Beantworten]

Die Plakette ist leider unscharf. Ich lese etwa: „Friedenskreuz / Frieden am 13.11.1991, aufgestellt am 13.11.1993“. --Pp.paul.4 (Diskussion) 10:55, 29. Sep. 2015 (CEST)[Beantworten]
Ich lese „FIRMK EUZ“. --Rôtkæppchen₆₈ 12:35, 29. Sep. 2015 (CEST)[Beantworten]

Firmkreuz“, siehe http://www.sauerlandkurier.de/vermischtes/kartoffeln-fuer-alle/ (Unterschrift zum Foto). --Vsop (Diskussion) 16:24, 29. Sep. 2015 (CEST)[Beantworten]

Rund um Hellefeld führt ein Wanderweg am "Füstes Kreuz" vorbei. Falls Anfrager und Hochlader des Bildes identisch sind, soll er bitte Näheres zur Inschrift mitteilen. --Pp.paul.4 (Diskussion) 10:52, 30. Sep. 2015 (CEST)[Beantworten]
Der eingezeichnete Wanderweg führt nicht an dem angegebenen Ort der Exif-Koordinaten der Bildes vorbei. Der Wanderweg trifft allerdings an einem Punkt ca. 4 km weiter nord-östlich an einer Kreuzung im Wald auf das Ende der Strasse "Fusthof". Im Hintergrund des Bildes sieht man eine Schnellstrasse mit Begrenzungpfeilern und dahinter Gartenhäuser. Es handelt sich dabei wahrscheinlich um die L 839 und Gartenhäuser zu den Grundstücken der Strasse "Auf der Heide". Sonntage waren der 17. November 1991 und 12. November 1995. Womöglich haben die Firmlinge von 1991 nach ihrer Firmung das Kreuz selbst entworfen, finanziert oder gebaut und 1995 "Aufgestellt", Rosenkohl (Diskussion) 13:53, 30. Sep. 2015 (CEST)[Beantworten]

öffentliches W-Lan

Wie kann man beim öffentlichen W-Lan in Hotels verhindern, dass der W-Lan-Betreiber oder ein Dritter Daten abgreift und man ausspioniert wird, wenn man sich mit dem Notebook ins W-Lan einwählt. --176.2.127.189 22:03, 28. Sep. 2015 (CEST)[Beantworten]

Das geht mit einem VPN. --Rôtkæppchen₆₈ 22:13, 28. Sep. 2015 (CEST)[Beantworten]
Danke, scheint geklappt zu haben. :-) --2607:F358:21:6A:219F:4211:2E6E:5646 22:40, 28. Sep. 2015 (CEST)[Beantworten]

Sozialistisches Unternehmen gründen

Für Frieden und Sozialismus - Seid bereit!
Soundtrack

Hallo. Ich habe die Gelegenheit ein Unternehmen zu gründen. Nun gibt es ja Unternehmen wie Mondragon in Spanien bei der die Belegschaft sehr viel vom Unternehmen entscheidet. Ich würde gerne mein Unternehmen genau gleich aufbauen, weil ich auch denke das diese Unternehmensformen die Zukunft gehört. Leider habe ich überhaupt keine Ahnung von den Sachen die man dabei wissen muss. Ich würde mich gerne an ein Unternehmen in De wenden das eine ähnliche Struktur hat. Ich glaub das nennt man auch Genossenschaft in Deutschland. Kennt jemand solche Unternehmen an die ich mich wenden könnte?--85.181.199.102 22:32, 28. Sep. 2015 (CEST)[Beantworten]

Wenn Du ein Unternehmen mit mehreren Mitarbeitern gründen willst und selbst keine Ahnung hast, brauchst Du auf jeden Fall einen Steuerberater. Der kennt sich meistens auch mit Unternehmensgründung und den Unterschieden zwischen den Unternehmen aus.--Optimum (Diskussion) 22:40, 28. Sep. 2015 (CEST)[Beantworten]
Es gibt mehrere Modelle, die Mitarbeiter zu beteiligen, zum Beispiel: Aktiengesellschaft, bei dem ein Teil der Aktien von den Mitarbeitern oder einer Mitarbeitervereinigung gehalten wird; Kommanditgesellschaft, bei der die Mitarbeiter Kommanditisten oder Komplementäre sind. Ein Unternehmen gründen, ohne Ahnung von Betriebswirtschaftslehre zu haben, würde ich jedoch nur, wenn ich das eingesetzte Kapital nicht benötige. --BlackEyedLion (Diskussion) 22:47, 28. Sep. 2015 (CEST)[Beantworten]
Wenn du "überhaupt keine Ahnung von den Sachen" hast, ist es wirklich eine gute Idee, ein sozialistisches Unternehmen zu gründen. Die spezifischen Voraussetzungen erscheinen erfüllt. --83.79.9.134 22:48, 28. Sep. 2015 (CEST)[Beantworten]
+1!--Antemister (Diskussion) 22:55, 28. Sep. 2015 (CEST)[Beantworten]
In sozialistischen Unternehmen hat die Belegschaft trotz Betriebsrat nicht mitzuentscheiden. Selbst die Betriebsleitung hat nichts zu entscheiden, sondern nur die Plankommission. Gründe einfach eine Plankommission, deren Aufgabe es dann sein wird, die ihr anvertrauten Unternehmen planvoll kaputtzuwirtschaften und bei reichen Kapitalgebern nach Krediten zu betteln, um den Kollaps abzuwenden. Die DDR hat erfolgreich vorgemacht, wie man eine Volkswirtschaft auf sozialistische Art und Weise ruiniert. --Rôtkæppchen₆₈ 22:54, 28. Sep. 2015 (CEST)[Beantworten]
Ja, das sah dann so aus. --2003:76:E6F:E435:4D52:5FC0:F791:29FF 23:04, 28. Sep. 2015 (CEST)[Beantworten]

Wiki Auskunft ist wohl nicht erste Adresse für sowas! Oder was soll dies? --80.187.96.213 23:02, 28. Sep. 2015 (CEST)--80.187.96.213 23:02, 28. Sep. 2015 (CEST)[Beantworten]

@Rotkäpp: Du beziehst Dich auf Unternehmen in einem mow totaliären sozialistischen Staat - das ist etwas anderes, als ein Unternehmen, in dem "sozialistische" Strukturen in Form von gemeinsamer Entscheidungsfindung bestehen, was offenbar gemeint ist. Ein Beispie für umfassende Arbeitnehmermitbestimmung wäre Elbdudler. Der Unternehmensgründer redet viel und gerne über die Struktur seines Unternehmens. --94.219.10.213 23:14, 28. Sep. 2015 (CEST)[Beantworten]

Wenn man keine Ahnung hat sollte man erstmal nichts anderes tun als lernen (und ganz sicher nicht einen Betrieb gründen, bis man weiß, was man da gerade macht). Für selbstverwaltete Betriebe gibt es seit mehr als 30 Jahren eine erste Adresse: die Zeitschrift Contraste --91.44.83.12 23:49, 28. Sep. 2015 (CEST)[Beantworten]

Es gibt erfolgreiche genossenschaftliche Unternehmen, zum Beispiel Mondragón Corporación Cooperativa. Wenn es etwas kleiner sein darf: Orbea (Unternehmen). --Pölkkyposkisolisti 23:50, 28. Sep. 2015 (CEST)[Beantworten]

Solche Träumereien hatten schon öfters welche, sogar mit Ahnung. Der letzte, der damit baden gegangen ist, war ein Hannsheinz Porst. Es gibt betriebswirtschaftliche Grundsätze die eingehalten werden müssen, ob da sozial, sozialistisch oder kapitalistisch geführt wird. Und dann werden schnell die Relationen vergessen. Wenn ein 500-Mannbetrieb eine Million am Jahresende übrig hat und die nicht in die Rücklagen nimmt, weil die soziale Idee im Vordergrund steht, dann sind das gerade mal 2000,-- Euro pro Mitarbeiter. Die haben die schnell verjuckt und der Betrieb ist ohne Reserven. Wenn ein Bill Gates 50 Millarden in USA verteilt, sind das gerade Mal 20 Dollar pro Amerikaner. --2003:68:ED61:E400:9D6:D769:4A58:E78B 23:56, 28. Sep. 2015 (CEST)Ja falscher Fehler, es sind 200,-- --2003:68:ED19:5F00:20BE:C052:9D95:6BF1 09:01, 29. Sep. 2015 (CEST)[Beantworten]
Womit sich die IP 2003... mit ihren Rechenkünsten auch gleich als sozialistischer Finanzchef empfielt. --83.79.9.134 00:04, 29. Sep. 2015 (CEST)[Beantworten]
Du meinst die andere IP2003, ich habe gar nicht gerechnet. Nimm zukünftig die letzten 4 Ziffern/Buchstaben, dann weiß man, wer gemeint ist. --2003:76:E6F:E435:4D52:5FC0:F791:29FF 00:13, 29. Sep. 2015 (CEST)[Beantworten]

Es wäre mal ein Anfang wenn der Fragesteller der Welt verkünden könnte, in welchem Land und in welcher Branche dieser volkseigene Betrieb entstehen soll und was er unter sozialistischem Unternehmen versteht. Mitbestimmung? Wobei? Finanzielle Beteiligung? Gleicher Lohn für alle? 17-Stunden-Woche? Hauseigene Kampfgruppe der Arbeiterklasse? Umweltzerstörung à la Bitterfeld? Solidarische Preisgestaltung bei sozialistischen Kunden und Lieferanten? --83.79.9.134 00:02, 29. Sep. 2015 (CEST)[Beantworten]

Jenseits der hier verbreiteten Fixiertheit auf stalinistische und poststalinistische Gesellschaften empfehle ich dem Fragesteller, sich das jugoslawische Wirtschaftskonzept (sogenannter Dritter Weg) genauer anzusehen. --91.44.83.12 00:31, 29. Sep. 2015 (CEST)[Beantworten]
Das war in der CSSR mit Ota Šik. --83.79.9.134 00:40, 29. Sep. 2015 (CEST)[Beantworten]
Das konkurriert nicht. Jugoslawien – Der „dritte“ Weg? Auch Bahro verfolgte da bestimmte Ansätze... Jugoslawien hat aber eine langjährige praktische Erfahrung, auch mit Streiks und der Wahl und Abwahl von Direktoren. Relikte davon sind heute noch in Serbien wirksam. Nicht alle Betriebe wurden durch die Oligarchen gestohlen. --91.44.83.12 00:45, 29. Sep. 2015 (CEST)[Beantworten]
Hier noch ein Einstieg in die Geschichte der 1936 nach anarchistischen Prinzipien organisierten Fabriken und Kollektive in Spanien. Über die Literaturangaben kommt man weiter, ein guter Anfang wäre vielleicht: Kaminski, Hans Erich: Barcelona. Ein Tag und seine Folgen 1.Mai 1937, edition tranvia, Berlin 1986.--91.44.83.12 00:39, 29. Sep. 2015 (CEST) Ergänzend: Juan A. Gamero Vivir la Utopia Arte-TV, 1997. Der Film über den Anarchismus in Spanien mit 30 überlebenden Anarchisten der Spanischen Revolution und des Bürgerkriegs lief im deutschen Fernsehen auf Arte unter dem Titel: "Die Utopie leben! Der Anarchismus in Spanien." --91.44.83.12 00:59, 29. Sep. 2015 (CEST)[Beantworten]
Sozialdemokratisch geht einfacher und die usgaben für MIlitär und Stasi sind nicht so hoch. Wenn Du bei einer Wohnbaugenossenschaft Miete zahlst und Anteile hast, württembergischen Genossenschaftswein trinkst und bei der klassischen Volksbank Dein KOnto und einen Anteil hast, bist Du schon mal ohne viel Umstände dabei. Serten DiskSkeptisch : Kritik 07:48, 29. Sep. 2015 (CEST)[Beantworten]
Stiftung / Stiftungsunternehmen[24]? --Eike (Diskussion) 08:36, 29. Sep. 2015 (CEST)[Beantworten]
Hier wissen einige Leute nicht was Sozialismus ist. In der UDSSR und DDR usw. gab es nie Sozialismus. Die nannten es so,aber das heißt nicht das es so war. Ich kann auch sagen ich bin Superman, deswegen muss es ja nicht stimmen. Aber danke für die Leute die hier geantwortet haben und Ahnung haben. Ich meine die Firma soll im Sinne von Mondragón Corporación Cooperativa aufgebaut werden, die ja sozialistisch ist und Millionen verdient. --85.181.199.102 16:18, 29. Sep. 2015 (CEST)[Beantworten]
Da liegt ein großes Missverständnis vor. Sozialismus bedeutet, dass die Produktionsmittel im Kollektiveigentum des Proletariats sind (d.h. zumeist, in der Hand einer Arbeiterpartei). Im Sozialismus wird auch prinzipiell kein Gewinn erwirtschaftet. Produziert wird nach einem zentral aufgestellten Plan und die produzierten Güter werden an die Arbeiter nach diesem zentral aufgestellten Plan wieder verteilt. Unternehmen gibt es im Sozialismus nicht. Unternehmen sind in Privateigentum, im ursprünglichen Wortsinn war der Eigentümer sogar grundsätzlich auch der Unternehmer, also der Geschäftsführer. Unternehmen agieren auf dem Markt und sind gewinnorientiert. Selbstverständlich kann in einem kapitalistischen Unternehmen der Unternehmer bestimmen, dass innerhalb des Unternehmens genossenschaftliche Prinzipien angewendet werden (das ist der Unterschied zum Sozialismus, wo der Leiter eines volkseigenen Betriebs nicht entscheiden kann, dass im Betrieb kapitalistische Prinzipien angewendet werden). Aber auch ein nach genossenschaftlichen Prinzipien organisiertes Unternehmen in einer kapitalistischen Wirtschaft ist immer noch ein kapitalistisches Unternehmen und eben kein "sozialistisches". --194.113.41.2 16:54, 29. Sep. 2015 (CEST)[Beantworten]
Das ist total nett, dass du uns den Sozialismus erklärst. Aber dein Sozialismus ist merkwürdigerweise nicht das, was ich unter Sozialismus verstehe. In meinem Sozialismus gibt es z.B. keinen zentral aufgestellten Plan, keine volkseigenen Betriebe und schon garnicht eine Partei. Warum sollten meine Kollegen und ich den Betrieb, in dem wir arbeiten, irgendjemand überlassen, sei es einem Direktor, einem Bürokraten oder einem Volk? Und dass Parteien die Menschen verderben kann man doch täglich in der Zeitung lesen. Ich habe nicht ohne Grund weiter oben mal den Dritten Weg in Jugoslawien ins Spiel gebracht... Etwas frei adaptiert bemerke ich also zu deiner (veralteten und schon längst überkommenen) Vision: Wenn ich nicht tanzen kann ist es nicht mein Sozialismus. --91.44.83.12 17:56, 29. Sep. 2015 (CEST)[Beantworten]
Bitte sehr, habe ich doch gerne gemacht, dir zu erklären, was Sozialismus bedeutet. Wenn dir das nicht gefällt, was Sozialismus bedeutet, darfst du dir natürlich gerne selbst etwas ausdenken und dies dann Sozialismus nennen. Das ändert aber nichts daran, das Sozialismus eben das bedeutet, was ich dir erklärt habe. --194.113.41.2 18:05, 29. Sep. 2015 (CEST)[Beantworten]
Naja, ich sehe schon, du hast dich tief in die Materie eingelesen... Lassen wirs lieber, wenn du dir da so sicher bist. :-) --91.44.83.12 18:17, 29. Sep. 2015 (CEST)[Beantworten]
Musste ich ja, mich da tief einlesen. Hab es ja schließlich studiert. Damit auch du den Unterschied zwischen Sozialismus (Verstaatlichung der Produktionsmittel sowie zentrale Planung und Lenkung des Wirtschaftsprozesses) und Genossenschaft (Unternehmensform mit dem Ziel, den Erwerb und die Wirtschaft ihrer Mitglieder durch gemeinschaftlichen Betrieb zu fördern) verstehst, kannst du die verlinkten Artikel lesen. --194.113.41.2 18:36, 29. Sep. 2015 (CEST)[Beantworten]
Genossenschaft hat doch mit Sozialismus garnix zu tun, was willst du denn damit? Ich habe eher den Eindruck, du hast vor 50 Jahren einfach die Neue Linke und die dortigen Diskurse innerhalb des letzten halben Jahrhunderts verpaßt. Aber wie geschrieben: Du bist dir so sicher, dass mir da eher langweilig ist. Also lassen wir das. Wenn ich nicht tanzen kann... --91.44.83.12 19:47, 29. Sep. 2015 (CEST)[Beantworten]
Studiert haben viele der hier Mitschreibenden, offenbar mit unterschiedlichem Erfolg. Deine Ausführungen zum Sozialismus beschränken sich auf einen Sonderfall, wie Du unserem Artikel leicht entnehmen könntest, wenn Du den mal studieren magst. --178.4.107.221 19:44, 29. Sep. 2015 (CEST)[Beantworten]
Ich hab zum zweiten mal das Unternehmen Mondragon erwähnt, aber ok. Nicht jeder hat eine große Aufmerksamkeitsspanne. Und ich rede natürlich von klassischem Sozialismus, nicht den ganzen pervertierten arten die von totalitären regiemen so genannt wurden, weil Diktatur hört sich halt nicht so schön an. Und es ist auch klar das mein Unternehmen dann trotzdem im unserem semi-kapitalistischen, staats-kapitalismus unterkommen muss und darum etwas eingeschränkt ist.--85.181.199.102 20:37, 29. Sep. 2015 (CEST)[Beantworten]
@IP 194: Lass mich nur kurz dein Weltbild zerstören: Es gab/gibt auch Sozialismus ohne Staat --> booom heads explode.--85.181.199.102 20:40, 29. Sep. 2015 (CEST)[Beantworten]
"Klassischer Sozialismus" fordert laut Wikipedia-Artikel die Enteignung der Produktionsmittel sowie die zentrale Verteilung der Güter (siehe Sozialismus#Klassischer_Sozialismus). Das ist genau das, was ich weiter oben geschrieben habe. Du darfst ja gerne deine eigene Utopie entwickeln und diese dann "Sozialismus" nennen. Die kannst du dann in entsprechenden Foren zur Diskussion vorschlagen. Hier ist jedoch die Auskunft der Wikipedia und hier werden Wissensfragen beantwortet. Das, was du suchst, hat nichts mit Sozialismus zu tun. Mondragon ist eine Genossenschaft, die auf dem Markt agiert. Wenn du eine Genossenschaft gründen willst, dann kannst du über den Artikel Genossenschaft die entsprechenden Informationen finden. Deine Frage ist somit beantwortet.
Nachdem ich deine Frage beantwortet habe, beantworte mir bitte noch die Frage, wo es Sozialismus ohne Staat gab/gibt? --194.113.41.2 11:10, 30. Sep. 2015 (CEST)[Beantworten]
Eigentlich wollte ich auch nur wissen ob es in Deutschland ein Unternehmen gibt das so arbeitet.--85.181.199.102 20:46, 29. Sep. 2015 (CEST)[Beantworten]
Ich hatte ja oben schon auf die Zeitschrift Contraste verwiesen. Dort gibt es auch seit 30 Jahren die Bunten Seiten. Wenn es das gibt, was du suchst, dann steht es dort. Vielleicht kannst du da auch über eine Bibliothek Einblick nehmen. --91.44.83.12 21:08, 29. Sep. 2015 (CEST) Addendum: Von größeren Betrieben in Westdeutschland weiß ich wenig. Historisch interessant ist sicher die Betriebsbesetzung des Zementwerkes Seibel und Söhne 1975 in Erwitte. Von Bedeutung, weil es in der Geschichte der Arbeitskämpfe in der BRD die erste Betriebsbesetzung ist, hier aber angeführt, weil die Produktion eine Zeit lang in Eigenregie weitergeführt wurde. (Google-Suche mit Erwitte Seibel Zement besetzt und Buchtipp: Dieter Braeg (Hg.): Erwitte – »Wir halten den Betrieb besetzt«. Geschichte und Aktualität der ersten Betriebsbesetzung in der Bundesrepublik. Die Buchmacherei, Berlin 2015 , 258 Seiten, vgl. auch hier und den 1975/76 gedrehten Dokumentarfilm von Edith Schmidt, David H. Wittenberg). In dem Artikel Kollektive Selbstverwaltung finde ich noch einen Hinweis auf den Fahrradhersteller Strike-Bike in Nordhausen. Hier wurde ebenfalls der Betrieb besetzt und zwischen 2006 und 2010 selbstverwaltet produziert. Und bedeutend war natürlich als erster selbstverwalteter großer Betrieb die Glashütte Süßmuth in Immenhausen, die 1970 vom Eigentümer kurz vor dem Bankrott den Arbeitern übereignet wurde, wobei da, wenn ich recht erinnere, der DGB auch etwas die Finger drin hatte (siehe hier und hier Der Spiegel 14/1970 und Fabian, Franz (Hrsg.): Arbeiter übernehmen ihren Betrieb oder der Erfolg des Modells Süssmuth, Reinbek, Rowohlt 1972, ISBN 3499116057). Grundlegend ist für dein Vorhaben auch empfehlenswert: Schwendter, Rolf (Hrsg.): Grundlegungen zur alternativen Ökonomie. 2 Bände. Teil 1: Die Mühen der Berge. Teil 2: Die Mühen der Ebenen., AG SPAK 1986, 292 / 267 S., ISBN 3923126379. Der Verlag hat zu dieser Thematik auch sonst einige interessante Texte zu bieten. --91.44.83.12 22:00, 29. Sep. 2015 (CEST) Achso: Und nicht vergessen (frei nach Rosa L.): Wenn ich nicht tanzen kann ist es kein sozialistischer Betrieb. --91.44.83.12 22:21, 29. Sep. 2015 (CEST)[Beantworten]
Voith in Bremen 1983 war der Versuch eines selbstverwalteten Maschinenbaubetriebs, Contraste-Artikel von 1999 [25] hier; die lukrative Windkraftsparte landete schließlich bei Siemens. Viele Details über arbeitnehmergeführte Unternehmen hier. --Aalfons (Diskussion) 11:31, 30. Sep. 2015 (CEST)[Beantworten]

Bouillabaisse zubereiten

Weiß jemand, wo man die benötigten Fische, insbesondere Drachenkopf, in Berlin kaufen kann? Oder online? Fällt wohl eher weg... @Rainer Zenz:, @Oliver S.Y.: - habt ihr oder sonst jemand ne Idee? Ich würde erstmal im KDW fragen. Oder gibts da bessere Quellen? --Pölkkyposkisolisti 23:45, 28. Sep. 2015 (CEST)[Beantworten]

Ich würde es beim Fischhändler versuchen. --83.79.9.134 00:06, 29. Sep. 2015 (CEST)[Beantworten]

KDW kannst bestellen --Graf Umarov (Diskussion) 00:15, 29. Sep. 2015 (CEST)[Beantworten]

Also neben dem KDW würde ich eher auf Selgros/Metro tippen, notfalls besorgen die es Dir dort per Bestellung. Ich fand den Fisch dort auch generell besser als bei vielen Fischtheken. "Mein Fischhändler" besorgt ihn mir auch so, und nicht in Hamburg :) Oliver S.Y. (Diskussion) 00:41, 29. Sep. 2015 (CEST)[Beantworten]
Eventuell im Fischmarkt? Außer dem Drachenkopf, bekommt man auch alle Fische bei Kaufland in den Fischtheken (dort kann man auch Bestellungen aufgeben) --178.2.78.6 06:06, 29. Sep. 2015 (CEST)[Beantworten]
Vergesse bitte den Fischmarkt, mag er am Anfang vieleicht noch überdurchschnittlich gewesen sein für das Umfeld, ist er heute die Fahrt nicht wert, wenn es um Frischfisch geht. Da entspricht die Qualität maximal denen vergleichbarer innerstädtischer Angeboten. Lohnt sich eher, wenn man Räucherfisch will, und Fisch zum Mittag essen.Oliver S.Y. (Diskussion) 10:54, 29. Sep. 2015 (CEST)[Beantworten]

Begründung für unterschiedliche Steuersätze von Diesel und Benzin

Gibt es außer einer (unterstellten) versteckten Subvention des Straßengüterverkehrs noch andere Gründe für die unterschiedlichen Steuersätze von Dieselkraftstoff und Ottokraftstoff? Ich frage insbesondere vor dem Hintergrund, daß die Abgase von Diesel gegenüber Benzin als problematischer eingestuft werden.--Fit (Diskussion) 23:51, 28. Sep. 2015 (CEST)[Beantworten]

Warum Diesel-Steuern niedriger sind als Benzin-Steuern. Berücksichtigen muss man bei der Kalkulation, dass für Diesel-Fahrzeuge erheblich mehr KFZ-Steuer fällig werden. Ein Diesel dient unterm Strich der Entlastung für Vielfahrer/Langstreckenpendler, Wenigfahrer fahren oft mit Benzin günstiger. --2003:76:E6F:E435:4D52:5FC0:F791:29FF 00:09, 29. Sep. 2015 (CEST)[Beantworten]
Vor allem muß man auch mal berücksichtigen, daß Dieselmotoren niedrigere spezifische Verbräuche (in kg Kraftstoff pro kWh Motorarbeit) haben und dadurch umweltschonender sind, da sie weniger CO2 emittieren. Zudem tragen sie zu einer besseren Ausnutzung des Vorprodukts Erdöl bei, da mit Dieselkraftstoff (= leichtem Heizöl) eine für Vergaserkraftstoffmotoren nicht verwertbare Ölfraktion verwendet wird. Ferner können Dieselmotoren relativ einfach mit Pflanzenöl gefahren werden. Das "problematischere" Emissionsverhalten ist teilweise eine direkte Folge der höheren Umweltfreundlichkeit: Die bessere Energieeffizienz ergibt sich teilweise aus höheren Verbrennungstemperaturen und höheren Verdichtungsverhältnissen, die eben auch die Stickoxidbildung begünstigen. Und hier muß man nicht pauschal verurteilen, sondern abwägen. (nicht signierter Beitrag von 92.224.153.62 (Diskussion) 04:24, 29. Sep. 2015 (CEST))[Beantworten]
Neben Kinderliebe ist das eine weitere Leiche, die die frühen Grünen im Keller haben. Die haben den Diesel - damals noch ohne Filter - als umweltschonend propagiert, gefördert und durchgesetzt. Ganz Unrecht hatten sie nicht, wie der Schreiber davor genau ausführt. An den feinen Ruß und sonstigen Aromaten hat da (noch) keiner gedacht, das war recht unbekannt und wenn jemand darauf hingewiesen hat, wurde das als politische Gegnerschaft abgetan. Die heutigen Grünen sind da auch schlauer (geworden) und fordern nun wenigstens den Rußfilter. --2003:68:ED19:5F00:20BE:C052:9D95:6BF1 08:46, 29. Sep. 2015 (CEST)[Beantworten]
"Die Grünen" haben den Diesel "als umweltschonend" "gefördert und durchgesetzt"? Wann soll das gewesen sein, in welcher Parallelwelt? Citation needed! (nicht signierter Beitrag von 92.224.72.200 (Diskussion) 09:37, 29. Sep. 2015 (CEST))[Beantworten]

Die IP hat geschrieben "die frühen Grünen" und da hat die IP recht. Unser Bio- und Chemielehrer (Gründer des Bund Naturschutzes im Lkr und eben dann ein Grüner) z.B. hat schon 1977 den Diesel als das Umweltfreundlichste angepriesen und selbst einen Mercedes so. ca. Baujahr 1953 gefahren. Logischerweise ohne Kat o.ä. weil es das damals noch nicht gab. Mit Überzeugung. --Heletz (Diskussion) 09:54, 29. Sep. 2015 (CEST)[Beantworten]

An 92.., das Gedankengut der Grünen ist so ziemlich von allen Parteien, wenn auch manchmal widerwillig übernommen worden, oft nicht aus Überzeugung, sondern aus wahltaktischen Gründen. Diese Grünen haben tatsächlich recht viel im Lande bewegt, ohne dass sie direkt am Ruder waren. Das muss man ihnen lassen.--2003:68:ED19:5F00:F91E:34A:2994:4EC7 10:34, 29. Sep. 2015 (CEST)[Beantworten]
Die frühen Grünen haben überhaupt nichts "gefördert und durchgesetzt", weil die nämlich nichts zu melden hatten. Und Dieselkraftstoff war schon immer billiger als VK. (Sogar sehr viel mehr billiger, als es nach dem Lieterpreis den Anschein hat: Er hat nämlich eine höhere Dichte, deswegen kriegt man mehr Gewicht und damit mehr Brennwert pro Liter, denn der gravimetrische Brennwert von Heizöl und VK ist fast exakt gleich.) Und Ahnung von naturwissenschaftlichen Zusammenhängen zu haben und sich daran zu orientieren finden höchstens Deppen verwerflich, ideologisch oder "grün", Normale halten das einfach nur für vernünftig. (nicht signierter Beitrag von 92.224.242.201 (Diskussion) 19:12, 29. Sep. 2015 (CEST))[Beantworten]


Die Lobby, die der Diesel hat, ist sein Wirkungsgrad von bis zu 46 %, der Benziner auf bis zu 43 %. Europa ist mit dem Steuervorteil von Diesel ein Einzelgänger. --Hans Haase (有问题吗) 14:22, 29. Sep. 2015 (CEST)[Beantworten]
"Bis zu 46 %" ist hohles Werbegeblubber - bei einem Funkgerät, einer Taschenlampe o. ä. will ich doch auch nicht wissen, wie weit sie maximal strahlen (was wörtlich bedeutet, daß sie darüber hinaus nicht wahrnehmbar sind), sondern natürlich, welche Mindestreichweite sie haben, also, bis wohin ich mich auf sie verlassen kann. Richtig ist: Dieselmotoren haben keine Drosselverluste, die bei "Ottos" im relevanten Teillastbereich den Wirkungsgrad stark reduzieren, weil sie immer mit voller Luftladung fahren und die Leistung über die Einspritzmenge regulieren. (Wobei sich das relativiert: Mit modernen Motorkonzepten gibt es eigentlich bis auf die Fremdzündung gar keinen Unterschied mehr zwischen Otto und Diesel: Die Gemischbildung im Otto erfolgt auch durch Einspritzung, und das Lambda-Verhältnis wird dadurch gesteuert, daß zunächst ein sehr fettes Gemisch in einem kleinen Bereich entflammt wird, was die Zylinderwände thermisch schont, und sich anschließend das brennende Gemisch mit der restlichen Luft im Zylinder vermischt; das ist ziemlich ähnlich wie beim Diesel.) Und damit kann man auch mal das Märchen vom "spritschluckenden Vollgas" beerdigen: Wie fährt man einen, sagen wir: Käfer, denn am sparsamsten? Antwort: Mit Vollgas (im 4. Gang natürlich) auf ca. 90 km/h beschleunigen, Leerlauf einlegen, Motor ausmachen, rollen lassen, bis die Geschwindigkeit auf ca. 60 km/h abgefallen ist, Zündung einschalten, vierten Gang einlegen, Gaspedal voll durchtreten, Kupplung kommen lassen und wieder auf 90 hochziehen. (Natürlich hatten wir das damals, als wie es gelesen hatten, auch sofort ausprobiert: Man kam auf Verbräuche von weit unter 5 l, was damals als sensationell galt. Und der "Trick" dabei ist eben, daß man den Motor so immer im Bereich des besten Wirkungsgrads (Vollgas ein Stück unter Nenndrehzahl) betreibt und so die Drosselverluste vermeidet, die aufträten, wenn man mit 2/3 Gas oder so konstant mit 70-80 km/h fahren würde. (nicht signierter Beitrag von 92.224.242.201 (Diskussion) 19:12, 29. Sep. 2015 (CEST))[Beantworten]

29. September 2015

Wieso dürfen die Autos nicht verkauft werden?

Sollte Volkswagen diesen Plan nicht liefern und auch keine akzeptable Lösung präsentieren, würde die "letzte Konsequenz", die das KBA angekündigt hat, die Besitzer der betroffenen Autos ebenso treffen wie den VW-Konzern: Denn dann würde das Bundesamt den Fahrzeugen die Zulassung entziehen. Die Pkw mit Dieselmotoren dürften dann weder bewegt noch verkauft werden. [26]

Das sie nicht mehr fahren dürfen ist klar, aber warum dürfen sie nicht verkauft werden? Man kann doch mit seinem Eigentum machen was man will. Notfalls kann man das Auto in ein Ausland verkaufen, wo es keine Umweltvorschriften gibt. Das Verkaufsverbot käme doch einer Enteignung gleich. --2003:76:E6F:E435:C7:1B67:7105:EE6A 09:38, 29. Sep. 2015 (CEST)[Beantworten]

Von "Enteignung" spricht man, wenn man vorher etwas hatte und es nachher nicht mehr hat. Für den Verkauf mancher Sachen gibt es gesetzliche Bedingungen, und es könnte sein, dass die hier nicht erfüllt sind. --Eike (Diskussion) 09:47, 29. Sep. 2015 (CEST)[Beantworten]
Ich weiß schon was Enteignung bedeutet. Wenn ich aber ein Auto habe, das nicht mehr fahren darf und das ich auch nicht verkaufen darf, um einen Erlös zu erzielen, dann kommt das einer Enteignung gleich. Ich habe dann etwas unnutz- und unveräußerbares, was im schlimmsten Fall noch Geld für die Unterbrigung kostet, weil ich ein abgemeldetes Auto nicht einfach auf die Straße stellen darf. --2003:76:E6F:E435:C7:1B67:7105:EE6A 09:52, 29. Sep. 2015 (CEST)[Beantworten]
Da müssten sich die Betroffenen vertrauensvoll(?) an den Verkäufer/Hersteller wenden, der sie beschissen hat (wobei die Händler das ja nicht wussten). Und ich kann mir nicht vorstellen, dass einem jemand verbietet, das Auto im Ausland zu verkaufen. Im Inland wird dir eh keiner viel Geld bezahlen für ein Auto, mit dem er dann ja genausowenig fahren darf. --Eike (Diskussion) 09:56, 29. Sep. 2015 (CEST)[Beantworten]
Mich interessiert in erster Linie die Grundlage für die Behauptung im verlinkten Artikel "Die Pkw mit Dieselmotoren dürften dann weder bewegt noch verkauft werden." Bewegen ist klar, aber auf welcher Grundlage könnte der Verkauf verboten werden. Ich muss das Auto ja nicht zum fahren verkaufen sondern weil es sich jemand als Blumenkübel in den Vorgarten stellen will. Kann so ein Verkauf tatsächlich untersagt werden? --2003:76:E6F:E435:C7:1B67:7105:EE6A 10:01, 29. Sep. 2015 (CEST)[Beantworten]

Das mit dem "dürften nicht verkauft werden" ist wahrscheinlich eine kreative Erfindung des Autors. Schließlich darf man nichtzugelassene Autos jederzeit kaufen und Verkaufen. man darf sie halt nicht im öff. Straßenverkehr bewegen. --Heletz (Diskussion) 10:08, 29. Sep. 2015 (CEST)[Beantworten]

Das ist wohl wieder ein Fall, in dem ein Journalist mit nicht mal Halbwissen die ihm vorliegenden Informationen nach Gutdünken umformuliert und mit dem eigenen Unwissen über die sich ergebenden Konsequenzen vermengt bis zum Schluss keiner mehr weis worum es geht. --Berthold Werner (Diskussion) 10:13, 29. Sep. 2015 (CEST)[Beantworten]
Es kommt da auf den Konsens an. Vermutlich ging es um den Entzug der Typgenehmigung. Solche Autos dürfen dann weder gefahren noch als fahrbare Autos verkauft werden. Als nackter Satz ist das "nicht verkaufen dürfen" sicher missverständlich. Auch diese nicht fahrbereiten Autos dürfen natürlich den Besitzer wechseln, wie man alles verkaufen kann, was einem gehört. (Die Oma vielleicht nicht, da sind aber dann auch die Eigentumsverhältnisse ungeklärt. SCNR, hatte heut einen Lachsack zum Frühstück.)--2003:68:ED19:5F00:F91E:34A:2994:4EC7 11:03, 29. Sep. 2015 (CEST)[Beantworten]
Letzteres glaube ich nicht. Beispiel Medikamente, Waffen, ... Es gibt Dinge, die darf man besitzen, aber nicht (beliebig) verkaufen. --Eike (Diskussion) 11:12, 29. Sep. 2015 (CEST)[Beantworten]
Mit dem "beliebig" in Klammern hast du das ja selbst relativiert. Manche Verkäufe sind mit Regeln behaftet, die muss man dann einhalten, aber verkaufen darf man. --2003:68:ED19:5F00:F91E:34A:2994:4EC7 11:28, 29. Sep. 2015 (CEST)[Beantworten]
Das glaube ich nicht. Wenn ich sagen wir Psychopharmaka verschriben bekomme, bin ich mir ziemlich sicher, dass ich die nicht verkaufen darf. --Eike (Diskussion) 12:10, 29. Sep. 2015 (CEST)[Beantworten]
Warum niicht? Wenn Du die Regeln einhältst (Apothekerlizenz, Käufer weist eine ärztliche Verschreibung vor ...) darfst Du die sicher verkaufen. --192.164.215.244 13:02, 29. Sep. 2015 (CEST)[Beantworten]
Ja , er hat ja recht. Keine Regel ohne Ausnahme und die hat er gefunden. Da haben wir tatsächlich das AMG (Arzneimittelgesetz) das den Verkauf von Arzneimittel, nicht nur von LMA-Pillen von Privatpersonen an Privatpersonen einschränkt. Da müsste ein AMG-konformer Mittelsmann eingeschaltet werden. Auch das ist schwierig bis vielleicht unmöglich.--2003:68:ED19:5F00:F91E:34A:2994:4EC7 19:58, 29. Sep. 2015 (CEST)[Beantworten]
Da hat der Journalist B******t geschrieben. Wenn das KBA die Typgenehmigung entzieht, darf das Auto selbstverständlich weiterhin verkauft und gefahren werden, allerdings nicht im öffentlichen Straßenverkehr. Es ist weiterhin möglich, das Auto auf nichtöffentlichem Grund ohne Zulassung zu fahren. --Rôtkæppchen₆₈ 12:02, 29. Sep. 2015 (CEST)[Beantworten]
Wenn z. B. der Flughafen Fahrzeuge fürs Rollfeld benötigt, könnte er nun einen Preisvorteil rausholen. --Hans Haase (有问题吗) 13:13, 29. Sep. 2015 (CEST)[Beantworten]
Falls eine erhebliche Umweltbelastung davon ausgeht (gnhihihihi), kann auch der Betrieb auf Privatgrund rechtswidrig sein. ;) —[ˈjøːˌmaˑ] 13:20, 29. Sep. 2015 (CEST)[Beantworten]
Oder auch nicht: §325 Abs. 7 StGB. --Rôtkæppchen₆₈ 13:26, 29. Sep. 2015 (CEST)[Beantworten]
Abs. 2 wäre doch anwendbar? —[ˈjøːˌmaˑ] 13:32, 29. Sep. 2015 (CEST)[Beantworten]
Vermutlich. --Rôtkæppchen₆₈ 14:55, 29. Sep. 2015 (CEST)[Beantworten]

Wobei ihr immer von Gebrauchtwagen ausgeht die verkauft werden sollen, vermutlich weil zwei Sätze vorher steht, dass Besitzer der Autos betroffen sind. Kann aber auch sein, dass nur Neuwagen gemeint sind. Woher soll man dem Artikel wissen? --Berthold Werner (Diskussion) 07:54, 30. Sep. 2015 (CEST)[Beantworten]

--2003:51:4960:1173:4870:943B:C6BE:DEC 09:50, 29. Sep. 2015 (CEST) suche stadesamt in skjern kommune in Dänemark[Beantworten]

http://www.rksk.dk/borger/familie--boern-og-unge/vielse/trauung --Vsop (Diskussion) 10:21, 29. Sep. 2015 (CEST)[Beantworten]

Gewinnwarnung - merkt denn niemand, wie bescheuert dieser Ausdruck ist?

Weiss jemand, woher dieser abstruse Ausdruck "Gewinnwarnung" kommt? Ist es ggf. eine dieser vielen schlechten und falschen Übersetzungen aus dem Englischen - wie z.B. "es macht Sinn"?

Der Logik des Ausdrucks "Gewinnwarnung" folgend, müsste eine Hochwasserwarnung eigentlich "Niedrigwasserwarnung" heissen, d.h. man warnt vor dem nun nicht mehr vorkommenden Niedrigwasser, eine Hitzewarnung im Sommer müsste dann ebenfalls "Kühlewarnung" heissen, d.h. man warnt von der dann nicht mehr herrschenden Kühle.

Meine Vermutung: Der Begriff wurde den Marketingabteilungen geschaffen, da Gewinnwarnung einfach besser tönt als "Verlustwarnung".

Ehrlicherweise muss man jedoch anerkennen, dass es ja bei negativen Ereignissen für eine Firma noch nicht absehbar ist, ob der Gewinn schlussendlich in einen Verlust dreht. Es ist zum einen eine schwarze Null, ein tieferer Reingewinn oder eben ein Verlust möglich. Trotzdem stört mich der Begriff und er läuft meinem Sprachgefühl zuwider.

Korrekterweise müsste man wohl von einer "Gewinneinbruchswarnung" sprechen. Hat es früher vielleicht einmal so geheissen und der Begriff wurde als zu lang empfunden? --Henry II (Diskussion) 10:14, 29. Sep. 2015 (CEST)[Beantworten]

es ist mehr wie „Wasserstandswarnung“, „Temperaturwarnung“, „Gewinnwarnung“, ... die können ja trotzdem noch Gewinn machen... also mit Verlust hats nix zu tun... --Heimschützenzentrum (?) 10:39, 29. Sep. 2015 (CEST)[Beantworten]
BK Eine Gewinnwarnung ist nicht gleichbedeutend mit Verlustwarnung. Es ist ein Begriff aus dem Aktienrecht, wo auch Gewinnprognosen gemacht werden. Die Gewinnwarnung soll bedeuten dass es eine Abweichung (nach unten) zu der (erwarteten) Prognose geben könnte. Gewinn steht hier auch eher im Sinne von (nicht quantifiziertem) Ertrag, also Ertragswarnung.--2003:68:ED19:5F00:F91E:34A:2994:4EC7 10:48, 29. Sep. 2015 (CEST)[Beantworten]
Der Wikipedia-Artikel Gewinnwarnung hilft hier weiter (hätte ich nur früher nachgeschaut), der Begriff hat durch die Wahl als Unwort in Österreich quasi den halbamtlichen Stempel als "bescheuert". Meines Erachtens kann meine Anfrage somit als erledigt betrachtet werden. Danke! --Henry II (Diskussion) 11:06, 29. Sep. 2015 (CEST)[Beantworten]
Genauso bescheuert ist übervorteilen statt benachteiligen. --Pölkkyposkisolisti 16:33, 29. Sep. 2015 (CEST)[Beantworten]
Also: Wenn ein Unternehmen für ein Geschäftsjahr einen Gewinn von 10 Mrd. EUR prognostiziert, aufgrund geänderter Bedingungen aber zu einem späteren Zeitpunkt nur noch 3 Mrd. EUR zu erwarten sind, muss eine Gewinnwarnung ausgegeben werden, denn der Gewinn wird nicht so hoch ausfallen wie ursprünglich prognostiziert. Nichts desto weniger wird das Unternehmen noch Gewinn machen. Verlustwarnung wäre also falsch. Ob man Gewinnwarnung besser Gewinnminderungswarnung oder so genannt hätte, naja.
Abseits: Sinn machen ist weder falsch, noch eine Übersetzung. Diesen Mythos hat wohl vorrangig ein gewisser Bastian Sick in die Welt gesetzt; es wäre nicht das einzige Mal, dass er daneben gelegen hat. Seine Argumentation in der Sache war jedenfalls lächerlich (mit dieser wäre bspw. auch der Ausdruck es macht Spaß falsch). Zum Ausdruck Sinn machen gibt es eine ausführliche, sehr interessante Abhandlung eines Professors für Linguistik (?) einer Hochschule (ich weiß keine Details mehr, könnte Bremen gewesen sein). Yellowcard (D.) 16:56, 29. Sep. 2015 (CEST)[Beantworten]
Das Wort Gewinnwarnung ist eine von Wirtschaftswissenschaftlern erschaffene Lehnübersetzung eines von Wirtschaftswissenschaftlern geschaffenen Wortes. Was will man da mehr erwarten, wenn man keine Linguisten ranlässt. --Rôtkæppchen₆₈ 17:22, 29. Sep. 2015 (CEST)[Beantworten]
Mit „Sinn machen“ befasst sich unter anderem Anatol Stefanowitsch im Bremer Sprachblog und im Sprachlog. ireas (Diskussion) 21:09, 29. Sep. 2015 (CEST)[Beantworten]
Ich sehe schon einen Unterschied zwischen "Sinn haben" und "Sinn machen". Nicht alles, was Sinn hat, macht auch Sinn. Kein Sprachpurist zweifelt an der Richtigkeit von "Sinn ergeben" - aber "ergeben" ist sicher näher am "machen" als am "haben" - und genau in der Bedeutung von "Sinn ergeben" hat (!) "Sinn machen" auch einen Sinn. "A steht vor B, B ist rot" - das hat Sinn, macht aber keinen. "A steht vor B, B steht hinter A" - das hat und macht Sinn. "Alle Quigliche sind griesbig, alle Griesbigen sind brätisch, daher sind alle Quiglichen brätisch" - macht Sinn, hat keinen.--Alexmagnus Fragen? 23:03, 29. Sep. 2015 (CEST)[Beantworten]
Dieser Abschnitt kann archiviert werden. --Eike (Diskussion) 13:29, 29. Sep. 2015 (CEST)

Rückgabe-/Ersetzungsrecht Fahrrad

Hallo zusammen,

ich habe vor etwa 3 Monaten bei einem Fachhändler ein Rad gekauft, genauer ein Diamant Opal (800€). Das war nötig, nachdem mir das Vorgängermodell gestohlen wurde. Diese Reihe zeichnet sich insbesondere durch eine extreme Wartungsarmut aus, so habe ich beim erwähnten Vorgänger in anderthalb Jahren etwa 6000 Kilometer runtergefahren und dafür nur ab und zu die Kette nachölen müssen.

Das jetzige Rad war – wegen der Kette und dem Chainglider – direkt nach dem Kauf mehrmals beim Händler, der es erst nach dem dritten Mal hinbekommen hat, das alles so einzustellen, dass die Kette nicht mehr springt und nur noch minimal am Chainglider schleift. Letzteres war beim 2013er Modell nie vorgekommen.

Am Wochenende hat sich nun der Chainglider gelöst, ich hab ihn wieder festgezurrt, als ich heute morgen losfahren wollte, war die Kette rausgesprungen. Ich musste den Glider also abmachen, um die Kette wieder einzusetzen. Alles wie gesagt bei der Baureihe nie erlebt.

Meine Frage: Was ist an der Stelle mein Recht? Ich möchte ein funktioinierendes, wartungsfreies Fahrrad – und zwar dieses Modell (oder von mir aus den Vorgänger, wenn man den irgendwo aufgetrieben bekommt). Aber mit dem konkreten Exemplar unter meinen Füßen bin ich sehr unzufrieden. Kann ich vom Händler ein neues Fahrrad oder die Rückzahlung des Kaufbetrags verlangen (natürlich abzüglich der Abnutzungsgebühr für drei Monate fahren)? Muss ich ihm auf ewig und drei Tage die Möglichkeit geben, den Chainglider immer wieder draufzusetzen und zu sagen, jetzt müsse aber alles in Ordnung sein?

Danke, --vigenzo (Diskussion) 11:15, 29. Sep. 2015 (CEST)[Beantworten]

Eigentlich hast du wohl von Anfang an ein Recht auf ein neues Exemplar.[27] Üblich scheint mir zu sein, dass der Händler erstmal zwei bis drei Versuche zur Reperatur hat. Danach kann man eine Wanldung/Rücktritt verlangen, oder halt ein neues Exemplar. --Eike (Diskussion) 11:42, 29. Sep. 2015 (CEST)[Beantworten]
BK Nach drei Monaten besteht noch Gewährleitung und wenn sich nun herausstellt, dass das Rad nicht die zugesicherten Eigenschaften hat, musst du beim Händler erst ein mal eine Behebung der Beanstandung verlangen. Erst wenn das nicht gelingt, kannst du Wandlung (Kaufpreis zurück, Umtausch) verlangen. Das sollte im Guten gehen. Wenn der Händler nämlich nicht mitmacht, musst du klagen. Dann wird ein Sachverständiger bestimmt, der deine Beanstandungen bestätigt oder auch nicht. Das kostet mehr als das Fahrrad, also machen auch die Händler meistens vorher mit. Wichtig ist, dass du dem Händler die Beanstandungen in einem Anschreiben übergibst, damit ist ein Vorgang geschaffen und die Sache wird auch ernst genommen.--2003:68:ED19:5F00:F91E:34A:2994:4EC7 11:48, 29. Sep. 2015 (CEST)[Beantworten]
Die Chainglider sind ein dauerndes Ärgernis. Das ist völlig unabhängig von der Fahrradmarke. Sieht zwar schick aus, ist aber bis heute nicht zu Ende erfunden. Bitte den Händler um eine Lösung, es gibt da einige Tricks. --Pölkkyposkisolisti 16:30, 29. Sep. 2015 (CEST)[Beantworten]

Snapshot auf Dateisysteme

Gibt es ein Land, wo ein Staatsanwalt in einem unter Verdacht stehenden Wirtschaftskonzern einen Snapshot auf alle vom Unternehmen verwalteten Dateisysteme zwecks Beweissicherung veranlassen kann ? --89.12.27.144 13:10, 29. Sep. 2015 (CEST)[Beantworten]

der Staatsanwalt kommt mit seinen speziellen Spezial-Experten angeritten und die machen dann, was nötig ist, um die Beweise zu sichern... egal ob mit oder ohne Snapshot (unterstützen das eigentliche alle Dateisysteme? nö, oda?)... --Heimschützenzentrum (?) 13:30, 29. Sep. 2015 (CEST)[Beantworten]
ach so: ich frag mich da immer, warum ein „Wirtschaftskonzern“ wohl ihn belastende Daten überhaupt auch nur temporär speichern sollte, anstatt sie schnellstmöglich zu vernichten... der Staatsanwalt hat wahrscheinlich immer die besten Chancen mit soner Art Steuersünder-CD vom unzufriedenen Angestellten... --Heimschützenzentrum (?) 13:42, 29. Sep. 2015 (CEST)[Beantworten]
Wie stellst du dir das vor? Schummelsoftware wird entwickelt, ohne sie jemals zu speichern, z. B.? --Eike (Diskussion) 14:00, 29. Sep. 2015 (CEST)[Beantworten]
die „Schummelsoftware“ findet der „Wirtschaftskonzern“ vllt vor dem „Staatsanwalt (m/w)“...? auch wenn irgendn schelmischer Studi die noch so gut vor der Code Inpection versteckt hat... oder wird sowas noch ordentlich formal verifiziert, damit das Ding nich plötzlich den Rückwärtsgong reinlegt? --Heimschützenzentrum (?) 14:11, 29. Sep. 2015 (CEST)[Beantworten]
Wer sagt eigentlich, dass die Daten in dem Land liegen, in dem die Staatsanwaltschaft gerade ermitelt? Sichere Datenverbindungen sind keine allzu große Zauberei und dass man auf einmal seine wertvollen Unternehmensgeheimnisse wegwirft, nur weil die irgendeine Behörde sehen möchte, glaube ich auch nicht. Die werden einfach an einen Ort geschafft, an den die Behörde nicht drankommt. In einen Staat ohne Abkommen zur Strafrechtshilfe. Das ist heute noch viel einfacher, als es das vor 50 Jahren noch war: Damals brauchte man noch massenweise LKWs, die fuhren dann über die Straße, es gab Grenzkontrollen, das alles kostete einen Haufen Zeit und Geld. Heute braucht man nur einen Server. --88.130.96.6 14:32, 29. Sep. 2015 (CEST)[Beantworten]
dann braucht man aber wieder Verschlüsselungs- und Signaturverfahren... falls im ausländischen Datenlager mal n Putsch passiert... --Heimschützenzentrum (?) 14:41, 29. Sep. 2015 (CEST)[Beantworten]
Ab einer gewissen Unternehmensgröße sollte man es sich auch leisten können, kompetente Mitarbeiter in der IT einzustellen. --88.130.96.6 14:54, 29. Sep. 2015 (CEST)[Beantworten]
na dann können die doch von ihrem „Kompetenz-Team“ die Daten im Inland verschlüsselt und signiert speichern... und nur wenn der Kopf der Köpfe seinen essbaren USB-Stick reinsteckt, dann kann man die Daten entschlüsseln und auf Echtheit überprüfen... --Heimschützenzentrum (?) 15:24, 29. Sep. 2015 (CEST)[Beantworten]
Die Daten sollten ja schon aus dem Land - sicher ist sicher. --88.130.109.193 18:18, 29. Sep. 2015 (CEST)[Beantworten]
Der Staatsanwaltschaft Zugriff auf die gesamten Unternehmensdaten zu geben, geht in wohl jedem denkbaren Fall zu weit - wahrscheinlich häufig deutlich zu weit. Und selbst, wenn das in irgendeinem Land der Welt möglich sein sollte, dann wird es daran scheitern, dass es schlicht praktisch nicht funktioniert. Du wirst einfach nicht jeden USB-Stick von jedem der zig-tausend Mitarbeiter bekommen. Das klappt nicht. Und schließlich hat praktisch jedes Unternehmen Betriebsgeheimnisse, die es nicht in einem öffentlichen Strafverfahren herausposaunt haben möchte. Wahrscheinlich würden Unternehmen ein Land, in dem sie eine unbegrenzte Offenlegung ihrer kompletten Daten fürchten müssten, schlicht meiden. --88.130.96.6 14:05, 29. Sep. 2015 (CEST)[Beantworten]
Der Snapshot auf alle Dateisysteme des inkriminierten Unternehmens würde der Staatsanwaltschaft als Trumpf in der Hinterhand dienen für den Fall, daß die Manager der Staatsanwaltschaft in der Einvernahme die Hucke volllügen. Der Snapshot kann ja bei Nichtgebrauch wieder leicht und behende gelöscht werden. 89.12.27.144 14:11, 29. Sep. 2015 (CEST)[Beantworten]
Ja und? Du willst also, nur weil ein Unternehmen "unter Verdacht steht", den Datenschutz abschaffen? Unter Verdacht wovon denn überhaupt? Dass mal ein Mitarbeiter im eingeschränkten Haltverbot geparkt hat? Dann möchte ich demnächst bitte bei dir zu Hause vorbeischauen - ich kann die Fotos aus deinem Schlafzimmer ja leicht und behende wieder löschen, nachdem ich sie mit meinen Freunden und der Cloud geteilt hab... Werd ich übrigens bestimmt machen - also das Löschen. Ehrenwort. Kurz und gut: So funktioniert Datenschutz nicht. Ein Staat, der sowas macht, wird ganz schnell ohne Unternehmen dastehen. --88.130.96.6 14:24, 29. Sep. 2015 (CEST)[Beantworten]

Walter Frese, Industriearchitekt, vor allem: Schlachthöfe

Wer findet eventuell die Lebensdaten? Danke! -- Neusser (Diskussion) 15:16, 29. Sep. 2015 (CEST)[Beantworten]

Nebenfrage: Wirklich "Frese"? Weil: Im Text und den Links auch "Freese". 213.169.163.106 17:00, 29. Sep. 2015 (CEST)[Beantworten]
Googlebookse mal => 1949 Herr Architekt Walter Frese kurz vor Vollendung seines 77. <= Kurzer Nachruf. 213.169.163.106 17:04, 29. Sep. 2015 (CEST)[Beantworten]

schule gründen

weiß jemand wie viel es ungefähr kosten würde, eine schule zu gründen? 194.166.117.17 16:03, 29. Sep. 2015 (CEST)[Beantworten]

[28] nennt Beispiele fuer vergleichsweise geringen Kapitalaufwand, allerdings knapp 10 Jahre alt. -- 195.132.129.70 16:09, 29. Sep. 2015 (CEST)[Beantworten]
Möglicherweise gibt es da bedeutsame Unterschiede zu beachten zwischen Grundschulen, Hochschulen, Baumschulen, Tanzschulen, Surfschulen, Gitarrenschulen usw. Außerdem könnte der Ort der Schule eine Rolle dabei spielen. Also ob ich die Schule in Manhattan aufmachen will oder in der Sahara. Nicht zuletzt spielt es vermutlich auch eine Rolle, ob man die Schule für 2-5 Schüler plant oder für 3000-5000. --Fogelfau (Diskussion) 18:18, 29. Sep. 2015 (CEST)[Beantworten]
Man sollte auch auf die Voraussetzungen achten, wenn man z.B. eine Delfinschule gründen möchte, bietet es sich an selbst Delfin zu sein. --Benutzer:Duckundwech 20:24, 29. Sep. 2015 (CEST)[Beantworten]
Die plastische Chirugie vollbringt da Wunder! -- Janka (Diskussion) 21:26, 29. Sep. 2015 (CEST)[Beantworten]
Das mit den Delfinen ist Blödsinn. Man muss doch auch nicht hoch sein, um eine Hochschule zu gründen. Auch habe ich selten Bäume getroffen, die eine Baumschule betreiben. --2003:76:E6F:E435:6594:8AE3:B6F:F718 21:29, 29. Sep. 2015 (CEST)[Beantworten]
Aber mittlerweile hat Hochschule doch viel mit Schwindel zu tun. --91.44.83.12 23:11, 29. Sep. 2015 (CEST) [Beantworten]

Das mit den Delphinen ist kein Blödsinn. Obs aber in dem Artikel hier hilft bezweifle ich eher. --80.187.107.41 21:45, 29. Sep. 2015 (CEST)[Beantworten]

Es geht hier um Lehranstalten, nicht um Gemeinschaften von Meeressäugern, also ist es Blödsinn. --84.144.116.10 21:55, 29. Sep. 2015 (CEST)[Beantworten]
Es kann durchaus sein, dass es bei der Frage in Wirklichkeit gar nicht primär um Delphine geht, sondern um etwas anderes. Die Kosten liegen jedenfalls irgendwo zwischen acht Euro und 300 Millionen Euro - je nachdem, worum es geht.--Fogelfau (Diskussion) 00:59, 30. Sep. 2015 (CEST)[Beantworten]
Vermutlich meint der Fragesteller so ein neumodisches Institut, in dem Kindergartenkinder aufs Universitätsstudium vorbereitet werden. --Rôtkæppchen₆₈ 12:52, 30. Sep. 2015 (CEST)[Beantworten]

Kein Auslandsversand

Wenn man mal im Ausland ist, dann fällt einem erst auf wie viele Onlinehändler nicht ins Ausland versenden, noch nicht einmal in EU-Staaten (bzw. allenfalls nach Österreich, in die Schweiz dann nicht mehr). Was ist der Grund dafür? Mehrwertsteuerbürokratie?--Antemister (Diskussion) 19:42, 29. Sep. 2015 (CEST)[Beantworten]

Das wissen am besten diese Händler! --Heletz (Diskussion) 20:03, 29. Sep. 2015 (CEST)[Beantworten]
Wieso sollten Leute, die kein Deutsch sprechen, in deutschsprachigen Internet-Shops einkaufen? -- Liliana 20:04, 29. Sep. 2015 (CEST)[Beantworten]
Wieso sollten Menschen im Ausland kein Deutsch sprechen? Ist das neuerdings verboten? --178.4.107.221 20:06, 29. Sep. 2015 (CEST)[Beantworten]
Über eine Mio. Deutsche leben im EU-Ausland, die prinzipbedingt gute Kunden von Onlinehändlern wären, dazu kommen Schweizer und Südtiroler, ebenfalls sind in Skandinavien und den BeNeLux-Staaten Deutschkenntnisse weit verbreitet. Auch kann man Online-Shops auch dann nutzen wenn man die Sprache äußerst rudimentär beherrscht, und dass kann sich lohnen, wenn sie billiger sind (habe mir sagen lassen dass Schweden viel bei Amazon.de kaufen, weil es Amazon in Schweden nicht gibt).--Antemister (Diskussion) 21:04, 29. Sep. 2015 (CEST)[Beantworten]
Im Falle der Schweiz ist es klar: die Schweiz gehört nicht zur europäischen Zollunion und alles, was rein kommt, muss verzollt werden. -- Liliana 21:05, 29. Sep. 2015 (CEST)[Beantworten]
Blöööööööödsinn.. und selbst wenn es so wäre, ist das ein Problem, das der Empfänger mit dem Postboten löst und der Versender gar nicht mitbekommt. --83.79.9.134 23:43, 29. Sep. 2015 (CEST)[Beantworten]
Horst Lüning dazu: [29]. --Komischn (Diskussion) 22:58, 29. Sep. 2015 (CEST)[Beantworten]
Der Versand ist teuer und unzuverlässig. Ein verlorenes Paket im Ausland nachzuzverfolgen ist nahezu unmöglich, außer man versichert es zusätzlich und läßt es gesichert zustellen. So kommen schon nach Frankreich, Grenznähe, schnell 100 Euro zusammen. --84.187.79.252 18:23, 30. Sep. 2015 (CEST)[Beantworten]

Haustierfutter

Wieso gibt es kein Schweinefleisch als Haustierfutter (Konserve)? --77.4.244.247 19:58, 29. Sep. 2015 (CEST)[Beantworten]

Fehlerhafte Annahme: Es gibt Schweinefleisch in Haustierfutterkonserven.

Wieso erledigt? Hinweis: Schweinefleisch ist durchaus Bestandteil von Katzen- oder Hundefutter. Das Kleingedruckte lesen - auch wenn man dazu eine Brille braucht. :-) Grüße --80.187.107.41 20:45, 29. Sep. 2015 (CEST)[Beantworten]

Das, was da auf dem Etikett des Heimtierfuttermittels beworben wird, ist nur die Geschmackszutat, nicht aber der Hauptbestandteil des Futters. Hauptbestandteil des Futters sind Schlachtnebenerzeugnisse, überwiegend vom Schwein, die nicht als Fleisch verkauft werden dürfen. --Rôtkæppchen₆₈ 21:33, 29. Sep. 2015 (CEST)[Beantworten]

Ich habe mich ja immer gefragt, warum es Katzenfutter eigentlich nicht in der Geschmacksrichtung Maus gibt. Grüße Dumbox (Diskussion) 21:36, 29. Sep. 2015 (CEST)[Beantworten]

[30] ein SmileysymbolVorlage:Smiley/Wartung/8)  --Rôtkæppchen₆₈ 21:51, 29. Sep. 2015 (CEST)[Beantworten]
Kreisch! Ich bin nicht allein! For the record: Ich heiße im RL nicht Franz Harrer und kenne diesen Seelenverwandten auch nicht. Grüße Dumbox (Diskussion) 21:54, 29. Sep. 2015 (CEST)[Beantworten]

Schweinefleisch hat einen schlechten Ruf als Hunde- und Katzenfutter, weil es ungekocht das Aujeszky-Virus übertragen kann! --80.219.124.55 13:29, 30. Sep. 2015 (CEST)[Beantworten]

Rohes Schweinefleisch ist in der Tat problematisch wegen der Aujetzky-Krankheit, auch wenn in Deutschland schon viele Jahre keine AK mehr auftrat. In Fertigfutter, egal ob nass oder trocken, wäre das Virus allein durch die thermische Behandlung inaktiviert. --Uwe G. ¿⇔? RM 14:29, 30. Sep. 2015 (CEST)[Beantworten]

Software herunterladen- Fernabsatzvertrag

Wenn ich mir entgeltlich Sofware heruntergeladen habe, habe ich dann einen Anspruch auf Rückgabe aufgrund der gesetzlichen Regelung von Fernabsatzverträgen?

--24.134.88.250 20:29, 29. Sep. 2015 (CEST)[Beantworten]

Du solltest die AGBs lesen.Da steht eigentlich was drinn. Nach den deutschen Gesetzen hast Du wenig Möglichkeiten, wenn Dein Gegenüber im Ausland sitzt. Innerhalb von D sieht es anders aus. Vermute mal aber Ausland. Nun: Um was geht es, um welchen Betrag geht es? Schlichte Kosten-Erfolg Analyse überlegen. Also schlechtem Geld muss man gutes Geld nicht unbedingt hinterherwerfen. --80.187.107.41 20:39, 29. Sep. 2015 (CEST)[Beantworten]
Kurze Antwort: In der Regel nein. Es gilt zwar die übliche vierzehntägige Widerrufsfrist ab Vertragsschluss. Sobald du die Software aber tatsächlich herunterlädst und benutzt, erlischt das Widerrufsrecht (jedenfalls dann, wenn die entsprechenden Informations- und Zustimmungspflichten eingehalten werden). Geregelt ist das in § 356 Abs. 5 BGB, Details gibt es beispielsweise im „Social Media Recht Blog“. Wie immer gilt: Im Einzelfall hilft hier nur der Anwalt. ireas (Diskussion) 21:16, 29. Sep. 2015 (CEST)[Beantworten]
Üblicherweise wird das ja so geregelt, dass der Anbieter eine Probeversion der Software anbietet, die entweder vom Funktionsumfang oder der Benutzbarkeitsdauer eingeschränkt ist. Dann kann man je nach Anbieter 14 bis 60 Tage kostenlos und unverbindlich testen und erst bei Gefallen die Software kaufen. --Rôtkæppchen₆₈ 21:30, 29. Sep. 2015 (CEST)[Beantworten]
Sowas war mal. Paypal. Sollange der Fragesteller aber nicht schreibt worum es eigentlich geht: Kann man ihm auch nicht helfen. --80.187.107.41 21:48, 29. Sep. 2015 (CEST)[Beantworten]
Das gibt es immer noch, schau mal die gängigen Antivirenprogramme, Microsoft Office oder Spezialsoftware an. --Rôtkæppchen₆₈ 21:55, 29. Sep. 2015 (CEST)[Beantworten]
Sinnvollerweise (und üblicherweise) verlangt der Anbieter vom Käufer vor dem Download aktiv eine Zustimmung gemäß § 356 Abs. 5 BGB - damit verliert der Käufer sein Widerrufsrecht (genauer gesagt: er verzichtet darauf). Versäumt es der Verkäufer, den Verkauf auf diesem Wege durchzuführen, oder macht er dabei Fehler (zum Beispiel bei der Reihenfolge von Belehrung und Verzicht), besteht ein Widerrufsrecht. --Snevern 23:54, 29. Sep. 2015 (CEST)[Beantworten]

Fielmann

Fielmann hat mir zwei Gläser offeriert. Die, welche ich beim letzten Mal bereits hatte und solche, die bis zum Rand hin scharf abbilden. Lohnt sich der Aufpreis wirklich?--85.4.233.141 21:36, 29. Sep. 2015 (CEST)[Beantworten]

Dir ist aber schon klar, was ein Augenarzt ist! --80.187.107.41 21:41, 29. Sep. 2015 (CEST)[Beantworten]
Dies ist kein Fall für den Augenarzt. Wieviel Aufpreis? Stets bedenken: auch Fielmann will (lukrative) Geschäfte machen; sachlich kann ich da nix zu sagen. 84.153.83.194 22:38, 29. Sep. 2015 (CEST)[Beantworten]
Beitrag eines ehem. Fielmann Kunden: Ich habe das letzte Mal (zugegebenermaßen für eine teurere Brille mit 8 Dioptr. und dünnen Kunststoffgläsern) mal beim lokalen kleinen Optiker nachgefragt, und der war günstiger. Und mit der Qualität bin ich mehr zufrieden als mit meinen letzten Fielmann-Gläsern. Grund für den Wechsel war dass bei meine beiden letzten Brillen nach ca .2 Jahren die Beschichtung so verkratzt war dass die Brille nicht mehr brauchbar war, diese hier hält jetzt auch schon zwei Jahre und ist noch OK.
Zur eigentlichen Frage: Als meine Gläser noch schwach waren, hat mich die Verzerrung nicht gestört. Bei stärkeren Gläsern nervt das (evtl) auch mehr. Hat es Dich denn bei Deiner letzten Brille gestört? Wenn nicht, warum dann wechseln? Edit: Ich fand Entspiegelung und minimale Tönung immer ganz nett, aber da mindestens ersteres eine Beschichtung ist, ist es auch wieder was das zerkratzen kann... -- Windharp (Diskussion) 08:18, 30. Sep. 2015 (CEST)[Beantworten]
Vater Fielmann: "Mein Sohn, heute lehre ich dir, wie man eine Brille verkauft. Du zeigst dem Kunden die Brille, wenn er nach dem Preis fragt, sagst du 100 Euro und beobachtest genau ob er zuckt. Zuckt er nicht, sagst du schnell", "... das einzelne Glas". Zuckt er immer noch nicht, dann sagst du schnell .. "und dann kommt noch das Gestell dazu". Daran muss ich immer denken, wenn ich beim Optiker bin und so ähnlich läuft das auch immer ab, wenn ich mal wieder eine Brille brauche. Jedes Mal habe ich ein schlechtes Gewissen, meinen Augen gegenüber, die mir wichtig sind und ich auch was gutes tun will. Jedes Mal das enttäuschte Gesicht der Optikerin, dass es nicht geklappt hat mit den breit geschliffenen Gläsern, Einfärbungen und Beschichtungen. Auch musste Mal bei Fielmann die Geschäftsführerin gerufen werden, weil ich darauf bestand die 120,-- Euro Gleitsichtsonnenbrille aus der Zeitungsanzeige zu bekommen. Die Augenärztin meinte, dass das ganze Gerede um das verbreiterte Sichtfeld Humbug sei und nur Verkaufsargumente sind. Medizinisch sei das nicht notwendig.--2003:68:ED16:C100:6987:8DB3:F930:D4E9 10:53, 30. Sep. 2015 (CEST)[Beantworten]

Da ich demnächst ne neue Brille brauche, hänge ich mich mal an. Bisher hatte ich Kunststoffgläser, entspiegelt und gehärtet. Kunststff sollte es schon sein, doch ist Entspiegelung sinnvoll? Und die Härtung? Rainer Z ... 11:43, 30. Sep. 2015 (CEST)[Beantworten]

Nur ganz selten stört es mich, dass von hinten was auf die Gläser gespiegelt wird. Das mit der Härtung sehe ich so: Es ist von der Kälte her egal, ob ich meine Hand in 8 oder 10°C kaltes Wasser halte. Kunststoff ist weicher als Glas und die Härtung ist Augenwischerei, weil das nur minimal einen Effekt hat.--2003:68:ED16:C100:6987:8DB3:F930:D4E9 12:52, 30. Sep. 2015 (CEST)[Beantworten]
Entspiegelung hilft eigentlich nur dem Fotografen, der mich knipst, brauche ich nicht. Plastegläser will ich nicht, die zerkratzen zu schnell und sind wegen niedrigerem Brechungsindex dicker. Ich habe aber Prisma, Heliomatic, Polarisation und Gleitsicht, womit die Gläser auch auf 500 Euro kommen. --Pölkkyposkisolisti 13:23, 30. Sep. 2015 (CEST)[Beantworten]

Die alten Kunstoff-Gläser, die er wieder offeriert hat (mit Entspiegelung etc. - weiss nicht was alles) liegen bei CHF 308.- Das Update für bessere RAndschärfe bei CHF 419.- Und die Fielmannzeitungsinseratgläser (die er mir absolut nicht empfehlen will - er hat mir dazu ein Anschauungsmodell mit doppelt so dicken Gläsern gezeigt) bei CHF 136.- Ich schätze die Preise gelten für beide Gläser zusammen. Bei 3.75 ist Glas vermutlich zu schwer. Ansonsten kann ich die Brillen nur bedingt vergleichen, meine alte Brille war einiges grösser. Insofern könnte die Randschärfe mir schon auffallen, wenn es nicht nur ein Verkaufsargument ist.--85.4.233.141 15:23, 30. Sep. 2015 (CEST)[Beantworten]

Welches Objektivgewinde hat ca. 325,4 mm Durchmesser und ca. 0,75 mm Steigung?

Welches Objektivgewinde hat ca. 325,3 bis 325,5 mm Außendurchmesser und ca. 0,75 mm Steigung? Die zugehörige Kamera ist eine Hitachi GP-5E/K aus den 1970er-Jahren, das Objektiv ist von Canon. --84.161.101.16 22:13, 29. Sep. 2015 (CEST)[Beantworten]

Worauf zielt das Ganze ab? --Heletz (Diskussion) 07:58, 30. Sep. 2015 (CEST)[Beantworten]
Beschaffung eines Adapterrings, um das alte Objektiv an einer CS-Mount-Kamera weiterzuverwenden. --79.219.213.182 08:09, 30. Sep. 2015 (CEST)[Beantworten]
M 35,5 × 0,5 ist ein häufiges Gewinde bei Videokameras, auch bei modernen. Eine Lösung könnte ein Adapter mit falscher Steigung sein, der dann einmal draufgezwungen wird, muß ja nicht mehr ab. Ein Beispiel, nur falsch herum und für M49. --Pölkkyposkisolisti 08:19, 30. Sep. 2015 (CEST)[Beantworten]

Wer mißt misst Mist. Es waren 25,4 mm, also C/CS-Mount. --79.219.213.182 10:35, 30. Sep. 2015 (CEST)[Beantworten]

Dieser Abschnitt kann archiviert werden. 79.219.213.182 10:35, 30. Sep. 2015 (CEST)

Wann kommt Achselhaar bei Frauen wieder in Mode?

Gibt es dazu wissenschaftliche, evt. ethologisch-mathematische Untersuchungen? Vorwissenschaftlich scheint es mir auf der Hand zu liegen, daß das Natürliche einen evolutionären Vorteil gegenüber dem Modischen langfristig haben muß.

--93.204.98.29 22:45, 29. Sep. 2015 (CEST)[Beantworten]

Die Evolution ist gegenüber der Mode ein erheeeeeeeeeblich langsamer wirkendes Phänomen. Ich würde daher nicht darauf warten, dass evolutionäre Vorteile über sonnengebräunt vs. blass, gut genährt vs. schlank oder eben achselbehaart vs. rasiert entscheiden. Mein persönlicher Tipp: Warte trotzdem lieber nicht auf diesen Modewechsel - vermutlich ist Godot schneller da. --Snevern 23:41, 29. Sep. 2015 (CEST)[Beantworten]
Waren Achselhaare bei Frauen je in Mode? Das ist bei mir persönlich der Abtörner schlechthin, aber Geschmäcker mögen sich ja unterscheiden. 90.184.23.200 06:59, 30. Sep. 2015 (CEST)[Beantworten]
Eben, das ist eine reine Geschmacksfrage, dazu braucht man keine "ethologisch-mathematische Untersuchungen" (*lol*) um Wissenschaftlichkeit vorzutäuschen. Es wird genau dann wieder "Mode", wenn die Mehrheit genug hat von glatten Frauen-Achseln.--Heletz (Diskussion) 07:28, 30. Sep. 2015 (CEST)[Beantworten]
"Früher" war es ungewöhnlich, dass Frauen sich die Achselhaare rasiert haben. Ich würde behaupten, die meisten haben nicht einmal an die Möglichkeit gedacht. So, wie du nie über die Möglichkeit nachgedacht hast, dir die Augenbrauen wegzurasieren. (Die Wikipedia schreibt "Zumindest in Westdeutschland war natürliche Achselbehaarung auch bei Frauen vor 1980 noch üblich, in der DDR bis zu deren Ende.") --Eike (Diskussion) 07:36, 30. Sep. 2015 (CEST)[Beantworten]

Achselhaare sind bereits (angeblich) zurück. Holstenbär (Diskussion) 08:30, 30. Sep. 2015 (CEST)[Beantworten]

Igitt. Ich dachte das wäre neben Coca-Cola und Elvis eins der Vorteile gewesen, von den Amis besetzt worden zu sein. Vgl. auch Euterhaarentferner#Geschichte oder en:112 Gripes About the French. Serten DiskSkeptisch : Kritik 08:46, 30. Sep. 2015 (CEST)[Beantworten]
Anfang der Neunziger Jahre fand ich in Frankreich ein Plakat einer konservativ-katholischen Gruppe, die Stimmung gegen die Achselhaarentfernung bei Frauen machte. Für mir als Deutschen war das eher exotisch, weil das (die Haarentfernung) zumindest in Männerkreisen noch gar kein Thema war. --195.200.70.46 09:42, 30. Sep. 2015 (CEST)[Beantworten]
Ende der 90er ist meine Stieftochter zum Austauschjahr nach Argentinien. Damals hat man mir ganz verstohlen mitgeteilt, daß ich ihr das Rasieren zeigen soll, weil Achselrasur in Südamerika normal sei. Das war bei Frauen in Europa noch vollkommen ungewöhnlich. --Pölkkyposkisolisti 12:30, 30. Sep. 2015 (CEST)[Beantworten]

Hallo zusammen, ich hab eine Frage zu einer Erkrankung, nämlich Akute lymphatische Leukämie, an der ein Freund gestorben ist. Er ist letztes Jahr daran erkrankt und nach einer Chemotherapie aus dem Krankenhaus entlassen worden. Nach einigen Monaten wurden dann erneut Leukämiezellen entdeckt und er kam wieder ins Krankenhaus. Nach einer erneuten Entlassung wurden vor wenigen Wochen wieder Krebszellen im Blut entdeckt und er kam ein drittes Mal ins Krankenhaus. Er sagte zu mir, der Arzt würde meinen, es spräche nichts dagegen es noch ein drittes Mal zu versuchen. Drei Wochen später ist er nun tot. Was mich jetzt ratlos zurücklässt sind folgende Sachen. Da wir sehr oft, eigentlich täglich, telefoniert oder geschrieben haben, hat er mich immer auf dem neuesten Stand gehalten, wie es ihm geht. Das letzte was er mir geschrieben hat, ist

  • dass er bei einem MRT gewesen sei
  • und dass er sich erstmals konkret mit seinem Ableben beschäftigen musste. Der Arzt hat ihm von einer speziellen Situation erzählt und wie er in eben dieser Situation behandelt werden will. Es wäre ein rein hypothetisches Szenario und nichts würde aktuell darauf hindeuten.

Weiteres konnte ich nicht in Erfahrung bringen, weil besagter Patient tatsächlich kurz darauf gestorben ist.

Nicht zu wissen, was mit den beiden Sätzen gemeint ist, quält mich, weswegen ich hoffe, dass irgendeiner mit medizinischem Wissen hier ist, der mir was dazu sagen kann.

1) Was macht man als Leukämie-Patient beim MRT? Ich dachte Leukämien metastasieren nicht. Kann man Blutkrebs damit auch untersuchen oder was passierte dort?

2) Welches Szenario könnte der Arzt geschildert haben? Da der Patient Zeuge Jehovas ist, könnte ich mir vorstellen, dass der Arzt ihn nach der Verwendung einer Bluttransfusion gefragt hat, bzw ob er eine ablehnen würde. Allerdings kann ich mir auch noch einige andere Fragen vorstellen

3) Ist das plausibel, dass ein Arzt so optimistisch ist und der Patient höchstens drei Tage später verstirbt? Wäre es denkbar, dass der Patient die Krankheit nur mir gegenüber kleingeredet hat und eigentlich wusste, dass es jetzt nichts mehr bringt, bzw. die Behandlung einstellen ließ, um die, wie er mir in den letzten Monaten immer wieder beschrieb, heftigen Nebenwirkungen nicht mehr ertragen zu müssen? Denn wenn ich ehrlich bin, ich selber könnte nicht beschwören, vielleicht beim dritten Mal immer noch die Kraft oder die Hoffnung das nochmal durchzustehen, erneut aufzubringen.

4) Welch anderen Möglichkeiten sind gängig. Ich weiß, das ist alles im Nebel rumstochern, aber ich hab einfach niemanden sonst, den ich fragen könnte und diese Fragen quälen mittlerweile tatsächlich.

Grüße--88.75.5.66 22:47, 29. Sep. 2015 (CEST)[Beantworten]

AML birgt einige Todesursachen:
  • Ablehnen einer Bluttransfussion (Zeuge Jehovas)
  • Herunterfahren des Immunsystems durch die Chemo zusammen mit Infekten, die dadurch Lebensbedrohlich werden
  • Schäden am Rückenmark durch fortschreiten der Krankheit
  • Schäden am Rückenmark durch die Chemo
  • «Blutversagen» im übertragenen Sinn, da das Blut nicht mehr das erfüllt was überlebenswichtig ist
Das MRT wird wahrscheinlich zur Visualisierung des Zustands des Rückenmarks gemacht worden sein. --Hans Haase (有问题吗) 23:01, 29. Sep. 2015 (CEST)[Beantworten]
Nachtrag: Hier ist interessant wie das Blut und die Leukocyten gebildet werden, den die sind bei AML die Folge, die Ursache ist einen Schritt früher. Es mag ein Hinweis sein, dass Krebspatienten auch eine Leukämie entwickeln können. Ob sie die Köpereigene Abwehr gegen den Krebs ist oder ein Folge oder Mutation des vorhandenen Krebes wäre in Einzelfall zu erforschen und Zusammenhänge sind nicht belegt, der Verdacht existiert. --Hans Haase (有问题吗) 23:16, 29. Sep. 2015 (CEST)[Beantworten]
Andere Mögliche Begründung der MRT: "Bei der akuten lymphatischen Leukämie (ALL) und einigen Unterformen der akuten myeloischen Leukämie (AML) finden sich bei manchen Patienten Leukämiezellen auch in Gehirn oder Hirnhäuten. Symptome können Kopfschmerzen oder neurologische Ausfälle sein, zum Beispiel Lähmungen. [...] Eine Magnetresonanztomographie (MRT) kann bei Verdacht auf Absiedlungen im Gehirn ebenfalls sinnvoll sein. Auf den Bildern lassen sich Ansammlungen von Leukämiezellen nicht selten von gesundem Hirngewebe unterscheiden." [31] --Buchling (Diskussion) 23:22, 29. Sep. 2015 (CEST)[Beantworten]
Sehr viel Ähnlichkeit mit einem erst kürzlich die Familie getroffenen Fall. Zu 1. Die MRT ist heute das beste bildgebende Verfahren um Vieles sehr genau, wie hier vermutlich das zentrale Nervensystem darzustellen. Das ist relativ unkritisch für die Todesursache zu sehen. Zu 2. Bluttransfusionen, falls hier vorgenommen, Bluttransfusionen werden heute sehr kritisch gesehen und die Risiken aufgeklärt. Man weiß heute, dass eine Bluttransfusion gleichbedeutend mit einer kleinen Organtransplantation ist und viele, bis tödliche Gegenreaktionen im Körper hervorrufen kann, die Betonung liegt auf kann. Diese Krankheit hat aber auch nur eine Überlebensrate von 50% bei erwachsenen Menschen. Auch das könnte der Arzt angesprochen haben und könnte die Beschäftigung des Kranken mit dem Ableben begründen. Zu 3. Auch im Fall in der Familie war 5 Tage vorher alles optimistisch, dass dieser (x-te) Krankenhausaufenthalt glücklich ausgeht und der Patient zu den 50% der Überlebenden gehören wird. Zu 4. Innerhalb 3 Tage verschlechterte sich der Zustand und unser Angehöriger starb. Die von uns geforderte Obduktion zeigte, dass er eine Lungenentzündung mit Versagen der Lunge bekommen hatte. Begründung, das durch die Chemo geschädigte Immunsystem, konnte solche Keime und Viren nicht mehr abwehren. Ob das mit einer rechtzeitigen Antibiotika-Behandlung hätte verhindert werden können, mag dahingestellt sein. In deinem Fall solltest du mit den Ärzten sprechen, oder die Angehörigen animieren denen Fragen zu stellen. Der Tod hat viele Ursachen.--2003:68:ED16:C100:6987:8DB3:F930:D4E9 09:38, 30. Sep. 2015 (CEST)[Beantworten]

Kostenpflichtige Webseiten

Als Person, die das Internet erst ca. 2006 erobert hat, werde ich schonmal mit Merkwürdigen Warnungen seitens meiner Eltern konfrontiert. Das man irgendwelche E-Mailanhänge nicht öffnen soll und es irgendwelche Schadspripte gibt, ist mir bekannt... insbesondere als bei uns der Internetzugang noch jung war und es schier unendliche Weiten waren, warnte man mich davor, dass Kosten entstehen könnten, wenn man Internetseiten auch nur aufruft. Nach fast 10 Jahren Erfahrungsammeln ist mir aber kein solcher Fall bekannt geworden, nur frage ich mich immernoch woher diese Warnung kommt. Wisst ihr zufällig mehr oder wo dieser Irrglaube(?) seinen Ursprung hat? --178.2.78.6 23:32, 29. Sep. 2015 (CEST)[Beantworten]

Nur reine Vermutung: "Ich habe versehentlich eine Seite geöffnet und dabei ein Porno-Abo bekommen" geht einem VIEL leichter über die Lippen als "ich habe etwa zwanzig mal auf der Pornoseite auf zustimmen gedrückt und dabei das Kleingedruckte übersehen" ;-) Und gerade im Internet gilt: Je mehr Leute etwas behaupten, desto eher wird es als wahr angesehen. Eine Seite, bei der man allein durch den Besuch rgendwelche Kosten hat, ist mir auch als "Früh-Internetler" (so etwa als 28.8er-Modems aktuell waren) auch noch nie untergekommen. Gruß, --Benutzer:Apierta 23:38, 29. Sep. 2015 (CEST)[Beantworten]
Andere Möglichkeiten: Früher wollten die ISP teilweise nach übermittelter Datenmenge Geld haben. Die Medienberichte zu Takedowns von z.B. kino.to suggerierten teilweise, dass schon ein Aufruf davon strafbar sei. Gibt sicherlich noch mehr, woher jemand sowas glauben könnte. (So er's denn wirklich glaubt, und nicht einfach nur mehr Bandbreite für seine eigene Nutzung will…) --nenntmichruhigip (Diskussion) 23:52, 29. Sep. 2015 (CEST)[Beantworten]
Es gab in der Schmalbandzeit sogenannte Dialer, die den Modem-Internetzugang auf eine teure Premiumnummer umgeleitet haben und so per Telefonrechnung Geld gekostet haben. Die beworbenen Premiumwebsites haben nur funktioniert, wenn man diesen Premiumzugang nutzte. Aktuell lässt sich durch Anbieten kostenloser, illegaler Medien (Filme, Musik, etc) und nachfolgender Abmahnung Geld verdienen. Außerdem gibt es Schadsoftware, die die Festplatte verschlüsselt und für die Entschlüsselung Lösegeld verlangt. Andere Schadsoftware installiert vorgeblich vom BKA oder einer anderen Behörde (BSI, CIA, etc pp) stammende penetrant nervige Popups, die den Rechner unbenutzbar machen. Entfernung natürlich nur gegen Lösegeld. Außerdem gibt es immer mehr verseuchte Werbebanner, die teilweise schon beim Klick darauf unerkannt Schadsoftware installieren. --Rôtkæppchen₆₈ 00:10, 30. Sep. 2015 (CEST)[Beantworten]
Genau: Dialer --Eike (Diskussion) 07:22, 30. Sep. 2015 (CEST)[Beantworten]
es ist sogar denkbar, dass das Betrachten von Grafikdateien (insbesondere als Bild und nicht als Textdatei...) unerwünschte Wirkungen hat (nämlich wenn der Viewer auf falsche Eingaben falsch reagiert...)... --Heimschützenzentrum (?) 07:39, 30. Sep. 2015 (CEST)[Beantworten]
Es ist nicht lange her, daß Webseiten nur dann als vorbildlich und sicher galten, wenn der komplette Inhalt serverseitig bereitgestellt wurde. JS, Flash, Java waren verpönt und es gab die Empfehlung, das auszuschalten, weil jeder Code, der clientseitig ausgeführt wird, potentiell schädlich sein kann. Prinzipiell ist das heute nicht anders. --Pölkkyposkisolisti 07:56, 30. Sep. 2015 (CEST)[Beantworten]
@Homer Landskirty: Vor einigen Jahren gab es bei Windows eine GDI+-Sicherheitslücke, die es ermöglichte, mit präparierten JPEG-Dateien Schadsoftware einzuschleusen. Dazu musste die Datei nur irgendwie zur Anzeige gebracht werden, weitere Benutzerinteraktion war nicht erforderlich. --Rôtkæppchen₆₈ 11:39, 30. Sep. 2015 (CEST)[Beantworten]
Det hatter doch jesacht? --Eike (Diskussion) 11:46, 30. Sep. 2015 (CEST)[Beantworten]
ich hab ja mal gesagt, dass es möglich ist, dass es aus Versehen passiert, wenn ich mit Absicht sonen „Fehler“ in ein Programm einbauen kann... das hat aber auch nix geholfen (da kann man dann auch froh sein, wenn s einem nich als Drohung ausgelegt wird... „Morgen werde ich zu krank zum Arbeiten sein“ war total verboten... und „schwarz“, „weiß“, „ja“ und „nein“...)... es muss wohl erst in Windoze passieren und in ner Zeitung stehen, damit es als denkbar gilt... *rotfl* --Heimschützenzentrum (?) 13:16, 30. Sep. 2015 (CEST)[Beantworten]

30. September 2015

Welches Archiv ist sinnvoll?

Ich habe beim Entrümpeln (nicht bei mir) einem Dachbodenfund gemacht, und zwar zwei Kartons mit ca. 80 alten gerahmten Dias aus dem zweitem Weltkrieg. Offenschichtlich handelt es sich laut Beschriftung um Fotos eines offiziellen Kriegsfotografen der Wehrmacht, der Teile des Balkanfeldzugs und die Invasion Griechenlands/Kretas dokumentierte. Da ich selber jetzt nicht wirklich auf solche Bilder stehe, aber sie möglicherweise einen historischen Wert besitzen, würde ich sie gerne einem geeigneten Archiv übergeben. Was käme da sinnvollerweise in Frage? Benutzerkennung: 43067 08:49, 30. Sep. 2015 (CEST)[Beantworten]

Militärhistorisches Museum der Bundeswehr würde mir da einfallen. -- Gerd (Diskussion) 09:01, 30. Sep. 2015 (CEST)[Beantworten]
Das lokale Stadtarchiv, wenn der Fotograf eine Beziehung zu dieser Stadt hatte. Ggf. mit anderen Teilen des Nachlasses. --195.200.70.46 09:38, 30. Sep. 2015 (CEST)[Beantworten]
Im Bundesarchiv in Freiburg, Abteilung Militärarchiv gibt es Historiker, die da gerne einen Blick drauf werfen und die Sammlung auch übernehmen, wenn sie deren Relevanzkriterien erfüllt. Anrufen, Termin ausmachen und einen schönen Herbsturlaub im Breisgau machen.--2003:68:ED16:C100:6987:8DB3:F930:D4E9 09:49, 30. Sep. 2015 (CEST)[Beantworten]
Ich führte mit dem Bundesarchiv, Abteilung Militärarchiv einen Schriftwechsel in ähnlicher Sache und kann die Damen und Herren nur loben und weiterempfehlen. --Pp.paul.4 (Diskussion) 10:58, 30. Sep. 2015 (CEST)[Beantworten]
Vielleicht Commons? --Pölkkyposkisolisti 12:33, 30. Sep. 2015 (CEST)[Beantworten]
Gerne, nur leider steht dem ja wie so häufig das Urheberrecht etwas entgegen. Ich weiß nicht, ob der Fotograf schon 70 Jahre tot ist. Benutzerkennung: 43067 13:08, 30. Sep. 2015 (CEST)[Beantworten]
Dann sollte Wikimedia die Bilder digitalisieren und sichern, bis sie mit der jeweils gültigen Uploadvorschrift hier entsprechen. Was das Bundesarchiv erstmal hat, ist für uns verloren. --Pölkkyposkisolisti 13:17, 30. Sep. 2015 (CEST)[Beantworten]
Service: Das Projekt dazu (auf der WikiCon gelernt...) heisst Wikipedia:Medienschatz. --Concord (Diskussion) 15:17, 30. Sep. 2015 (CEST)[Beantworten]
Einen Diascanner habe ich ja selber, aber bislang die Arbeit für diese Motive gescheut. Bekommt man dafür WikiCup Punkte ;-)? Bundesarchiv hört sich gut an, ich werde da mal anfragen. Ich frage auch mal bei wp:Medienschatz an, ob Interesse besteht. Danke für den Hinweis, kannte ich noch nicht. Benutzerkennung: 43067 15:45, 30. Sep. 2015 (CEST)[Beantworten]

Da gibt's nur eine Möglichkeit: Das Bundesarchiv in Freiburg, Abteilung Militärarchiv ! --Heletz (Diskussion) 17:52, 30. Sep. 2015 (CEST)[Beantworten]

DNA-Abgleich im Krimi

Im Krimi sieht man immer wieder, dass die Lösung eines Falls dadurch zustande kommt, dass eine DNA-Analyse für ein nicht-menschliches Individuum gemacht wird, sei es nun ein Haustier, dessen Haare am Tatort gefunden worden, oder sogar eine Zimmerpflanze, deren Pollen auf der Leiche waren. Nur um es ganz klar zu sagen, ich meine nicht, dass mittels Genanalyse die genaue Rasse/Art/Spezies bestimmt wird, sondern dass genau ein bestimmtes Individuum erkannt wird: es war genau der Königspudel der Schwägerin (und nicht der des Nachbarn), es war genau die Yuccapalme des Hausmeisters (und nicht die der Geliebten). Kann das durchschnittliche deutsche (amerikanische) Kriminallabor das überhaupt? Sind die Leute dort technisch ausgerüstet und geschult, um eine solche Bestimmung auch bei nicht-menschlichen Individuen in der üblichen Zeit zu machen? --92.225.178.98 11:27, 30. Sep. 2015 (CEST)[Beantworten]

Für so etwas wird Vergleichs-DNA von den verdächtigen Lebewesen benötigt, dann wird exakt gleich wie bei menschlicher DNA vorgegangen. Als Ergebnis kommt ein Übereinstimmungsgrad der Spuren-DNA mit den einzelnen Vergleichsproben heraus, anhand dessen auf due Quelle der Spur geschlossen wird. --Rôtkæppchen₆₈ 11:31, 30. Sep. 2015 (CEST)[Beantworten]
Kein Problem mehr. Das machen Zoologen und Botaniker (auch nicht kriminologisch) routinemäßig, um Individuen und deren Verwandtschaft zu analysieren. Gängige Techniken wären random amplified polymorphic DNA (RAPD), amplified fragment length polymorphism (AFLP), inter-simple sequence repeat method (ISSR), es gibt haufenweise andere.--Meloe (Diskussion) 13:57, 30. Sep. 2015 (CEST)[Beantworten]

Daran, dass es technisch möglich ist und für ZoologInnen und BotanikerInnen zur Routine gehört, habe ich nie gezweifelt. :-) Aber kann das auch jedes kriminaltechnische Labor bzw. das Personal, das üblicherweise nur mit menschlicher DNA arbeitet? Ich erinnere mich dunkel, dass zumindest in Deutschland bei einem DNA-Abgleich nur ganz bestimmte Bereiche der DNA überhaupt geprüft werden, manche Informationen zum Beispiel gar nicht ausgelesen werden dürfen. Kann denn diese Technik einfach auch auf nicht-menschliche DNA übertragen werden?--92.225.178.98 14:31, 30. Sep. 2015 (CEST)[Beantworten]

Die verwendete Technik ist im Prinzip diesselbe, nur technische Details wie der verwendete Primer unterscheiden sich ggf. Normalerweise wird das dann wohl in einem biologischen Labor durchgeführt werden, der technische Ablauf dort ist aber fast derselbe. Die üblicherweise verwendeten Techniken werten nichtcodierende Sequenzen aus, eine Sequenzierung, die ggf. bestimmte Allele identifizieren könnte, ist nicht Standard (dadurch können beim Menschen ggf. Geschlecht, in bestimmten Grenzen auch Verwandtschaft, Herkunft, Hautfarbe und weitere Merkmale ermittelt werden, das hat aber mit dem üblichen DNA Fingerprinting direkt nichts zu tun). Sequenzanalysen im kriminologischen Zusammenhang bei Tieren oder Pflanzen sind wohl nicht üblich, ggf. wird aber DNA Barcoding eingesetzt werden, um zum Beispiel eine Pflanzen- opder Tierart zu bestimmen oder zu verifizieren.--Meloe (Diskussion) 18:22, 30. Sep. 2015 (CEST)[Beantworten]

K.F.Koehler Verlag

Meine Dammen und Herren

Ich habe schwierigkeiten mit findung eines Verlags, naemlich <<K.F.Koehler Verlag Gmbh>>. Dieses Verlag hoechstwahrscheinlich existiert nicht mehr, trotzdem ich wuerde gern erfahren wer heutzutage Urheberrechte schutzt, die frueher K.F.KoehlerVerlag geschuetzt hat. Es handelt sich ueber Gustav Radbruchs <<Rechtsphilosophie>> erschienen im Jahr 1973. Bitte helfen sie mir dieses Angelegenheit zu klaeren. Ich bedanke mich im voraus mit freundlichen Gruessen ihr Giorgi Jimsheladze--217.147.235.45 11:34, 30. Sep. 2015 (CEST)[Beantworten]

Der K.F.Koehler Verlag heißt heute offenbar Hase & Koehler Verlag GmbH. Inwiefern dieser Verlag Rechte an dem besagten Buch hält, weiß ich nicht. Ich finde das Buch hier bei Amazon. Erschienen ist es 2011 bei C.F. Müller. --88.130.95.57 11:44, 30. Sep. 2015 (CEST)[Beantworten]
Ich lese hier: Die Umbenennung des K. F. Koehler Verlages in Hase & Koehler Verlag fand bereits 1924 statt. Die Neugründung erfolgte demnach 1964. Im Artikel Karl Franz Koehler finde ich noch einen Literaturhinweis: Von-Hase-und-Koehler-Verlag (Hrsg.): K. F. Koehler, v. Hase & Koehler. Buchhändlerische Familientradition 1789–1989. von Hase und Koehler, Mainz 1989, ISBN 3-7758-1198-2. --91.44.83.12 14:12, 30. Sep. 2015 (CEST)[Beantworten]

Minianwendungen WIN 7

Hallo! Seit einiger Zeit habe ich Probleme mit der Minianwendung "Wetter", wo immer die Meldung "Verbindung mit dem Dienst kann nicht hergestellt werden" (Internetverbindung voll OK, keine Änderungen am Netzwerk, nur Browser mal aktualisiert - Firefox 4). Ist darüber etwas bekannt? Danke 2003:45:4801:1300:809C:A9BD:CAAF:B36 11:35, 30. Sep. 2015 (CEST)[Beantworten]

Diese App kannst du entfernen. Der Support für die Minianwendungen wurde eingestellt; die Wetter-App wird daher auch in Zukunft nicht mehr funktionieren. Ach ja und Firefox solltest du aktualisieren; aktuell ist Version 41... --88.130.95.57 11:46, 30. Sep. 2015 (CEST)[Beantworten]
Der Grund für die Einstellung diverser Minianwendungen/Gadgets (u.a. Wetter, Aktienkurse und Währung) unter Windows Vista und 7 waren wohl nicht behebbareSicherheitslücken. Diejenigen Minianwendungen, die ohne Internet auskommen, funktionieren noch, also Uhr, CPU-Last und ein paar andere. Ich habe das Wetter-Gadget immer gerne benutzt. Jetzt gibt es aber Windows 10, für Windows-7-Nutzer kostenlos, und da gibt es die Wetter-Kachel, die sogar noch ein bisschen mehr kann, aber leider nicht mehr auf dem Desktop ist. Als Ersatz für die Vista- und Windows-7-Uhr habe ich mir für Windows 8.0 wenige Tage nachdem es rauskam einen Ersatz selbstgeschrieben. --Rôtkæppchen₆₈ 15:36, 30. Sep. 2015 (CEST)[Beantworten]

Panikmache ?

Hallo. Ist dieser Brief Panikmache oder Tatsache ? Und sind dies ernstzunehmende Quellen ? Gefunden hier. Danke. 2001:7E8:C094:6201:F58C:A605:8225:14E9 12:11, 30. Sep. 2015 (CEST)[Beantworten]

Sexuelle Gewalt unter den Asylanten ist wirklich nichts neues, sondern eher Standard. Das fällt z. Z. einfach nur mehr auf, weil die absolute Zahl der Fälle größer ist. Der Brief ist sicher krawallig formuliert, aber sicher nicht frei erfunden.--Antemister (Diskussion) 12:22, 30. Sep. 2015 (CEST)[Beantworten]
"Standard?" Quelle? --Eike (Diskussion) 12:26, 30. Sep. 2015 (CEST)[Beantworten]
Ließt du denn keine Zeitung? Konkret habe ich diese Aussage aus der berichterstattung um diese Geschichte, wo das ein Fachmann so ausgeplaudert hat. Im Normalfall ist aber ja dass die absolute Zahl der Fälle klein sein dürfte, einfach weil es normalerweise so wenige Frauen unter den Asylanten sind. Z. Z. ist das ja anders, schlicht wegen der großen Zahl der Asylanten überhaupt und weil sich darunter ja auch Kriegsflüchtlinge befinden.--Antemister (Diskussion) 12:38, 30. Sep. 2015 (CEST)[Beantworten]
Da ist schon was dran von standart, wenn der Chef der deutschen Polizeigewerkschaft Rainer Wendt es gestern bei N24 live geschildert hat: http://www.spiegel.de/politik/deutschland/asyl-polizeigewerkschaft-ueber-machtkaempfe-unter-fluechtlinge-a-1055163.html & http://www.spiegel.de/politik/deutschland/asyl-polizeigewerkschaft-ueber-machtkaempfe-unter-fluechtlinge-a-1055163.html . 2001:7E8:C094:6201:F58C:A605:8225:14E9 12:48, 30. Sep. 2015 (CEST)[Beantworten]
Kann man doch auch positiv sehen: Sofern sie sich nur gegenseitig die Köpfe einschlagen, lassen sie die Deutschen (& Schweden) in Ruhe.--Antemister (Diskussion) 12:51, 30. Sep. 2015 (CEST)[Beantworten]
Werden sie aber wohl nicht (bei ihren Leisten bleiben). Wenn Siegmar Gabriel das Grundgesetz auf arabisch drucken lässt, sind das schon düstere Vorzeichen, in meinen Augen 2001:7E8:C094:6201:F58C:A605:8225:14E9 12:56, 30. Sep. 2015 (CEST)[Beantworten]
Ich kann in keinem deiner Links etwas von sexuellen Übergriffen (und schon gar nicht davon, dass sie standart wären) finden. --Eike (Diskussion) 13:02, 30. Sep. 2015 (CEST)[Beantworten]
Deine Quelle dafür ist also "Im Münchner Polizeipräsidium sieht man keine Anhaltspunkte für vermehrte sexuelle Übergriffe auf Frauen und Kinder in der Münchner Bayernkaserne." Alles klar, keine weiteren Fragen. --Eike (Diskussion) 13:01, 30. Sep. 2015 (CEST)[Beantworten]
"Doch Fälle sexueller Gewalt kämen meist gar nicht zur Anzeige, weil sich die Frauen nicht trauten, über das Erlebte zu sprechen"--Antemister (Diskussion) 13:04, 30. Sep. 2015 (CEST)[Beantworten]
... und aus "nichts Genaues weiß man nicht, aber wir haben keine Anhaltspunkte" machst du mal eben "ist eher Standard". --Eike (Diskussion) 13:11, 30. Sep. 2015 (CEST)[Beantworten]
Synonym für Standart ist Regel. Regelmäßig ist Standart. "Frauen berichten, dass sie, aber auch Kinder, vergewaltigt wurden oder sexuellen Übergriffen ausgesetzt sind. So schlafen viele Frauen in ihrer Straßenkleidung. Frauen berichten regelmäßig, dass sie nachts nicht zur Toilette gehen, weil es auf den Wegen dorthin und in den sanitären Einrichtungen zu Überfällen und Vergewaltigungen gekommen ist. Selbst am Tag ist der Gang durch das Camp bereits für viele Frauen eine angstbesetzte Situation." Jetzt hab ich mir die Frage erst beantwortet. Dadurch dass jemand mit dem Wort Standart daherkam. Also, keine Panik. Jetzt erledigt. Danke 2001:7E8:C094:6201:F58C:A605:8225:14E9 13:12, 30. Sep. 2015 (CEST)[Beantworten]
Jetzt hab ich eine Weile gesucht, wo du das Zitat her hast. Jetzt ist es klar: Die Quelle dafür, dass der Brief keine Panikmache ist, ist der Brief! Klingt "logisch". --Eike (Diskussion) 13:21, 30. Sep. 2015 (CEST)[Beantworten]
In einer vor ein oder zwei Tagen gesendeten Fernseh-Dokumentation über die Vorbereitungen der Stadt Darmstadt für die Ankunft der Flüchtlinge war das Thema jedenfalls Gegenstand und es wurde erwogen, den Frauen die Toiletten in der Halle zu überlassen und den Männern die Dixie-Klos außerhalb der Halle. --Pp.paul.4 (Diskussion) 14:56, 30. Sep. 2015 (CEST)[Beantworten]
Dieser Abschnitt kann archiviert werden. Nutzer beantwortet seine eigene Frage. Da war der Zweck wohl nicht das Fragen... --Eike (Diskussion) 13:05, 30. Sep. 2015 (CEST)

Altersbestimmung der Erde

Wir haben Leute gehabt, die hielten die Erde für rund. Wir haben Leute gehabt, die auch nicht daran glaubten, dass die Erde der Mittelpunkt des Universums sei. Wo finden wir etwas in der Wikipedia zur Altersbestimmung, denn die gängige Meinung ist ja mittlerweile, dass wir es mit Jahrmillionen und Milliarden zu tun haben? Wer beispielsweise in der Historie hat ein Alter über 1 Mio Jahre angenommen oder mehr? -- Sdfghjklökjhgfds (Diskussion) 12:16, 30. Sep. 2015 (CEST)[Beantworten]

Die einst seriöse Theorie der hohlen Erde folgte direkt aus Newtons principia.en:Age_of_the_Earth#Early_calculations, Alter der Erde wäre das Lemma. Physiker wie Helmholtz und Kelvin haben evolutionstaugliche Alter das ganze 19. Jahrhundert lang für völlig ausgeschlossen gehalten. Serten DiskSkeptisch : Kritik 12:33, 30. Sep. 2015 (CEST)[Beantworten]
Suchst Du Uran-Blei-Datierung? -- Iwesb (Diskussion) 12:46, 30. Sep. 2015 (CEST)[Beantworten]
Es geht hier nicht um eine "gängige Meinung" oder "den Glauben", die Erde sei sehr alt. Es geht hier um wissenschaftliche Erkenntnisse. Das ist ein grosser Unterschied. Erst gestern hat sich SPON damit befasst: http://www.spiegel.de/wissenschaft/natur/fossilien-wie-das-alter-bestimmt-wird-a-1053456.html 130.226.41.19 13:39, 30. Sep. 2015 (CEST)[Beantworten]
Geochronologie#Samarium-Neodym-Methode taugt auch. Der Sponartikel ist banales Skeptikergedöns und lässt die tatsächlich interessante Kontroverse aus. Die Evolutionstheorie wurde zu einer Zeit bekannt und breit anerkannt, in der sie - aus physikalischer Sicht bezüglich des Alters der Erde - von völlig abstrusen und unbelegten bzw. physikalisch unmöglichen Voraussetzungen ausging. Will heißen, Physik ist ganz nett, aber nicht wirklich wichtig. Serten DiskSkeptisch : Kritik 13:46, 30. Sep. 2015 (CEST)[Beantworten]
Skeptikergedöns? Der Journalist, der den Artikel geschrieben hat, kennt offenbar unsere Artikel Geschichte der Geologie (ab Kapitel 4: "Die Entdeckung der Erdgeschichte") und Stratigraphisches Prinzip und noch ein paar andere. Zur Frage: Wer beispielsweise in der Historie hat ein Alter über 1 Mio Jahre angenommen oder mehr? Das war Georges-Louis Leclerc de Buffon 1745, oder so. Serten hat allerdings Recht, dass die ersten Schätzungen eines deutlich höheren Erdalters, als die biblischen 6000 Jahre auf physikalischen Annahmen beruhten, die heute als widerlegt gelten. Handfester wird es erst nach der Entdeckung des radioaktiven Zerfalls. Aber selbst mit so schlichten und leicht verständlichen Methoden, wie Dendrochronologie und Warvenchronologie kommt man schon lockerflockig weit über 6000 Jahre hinaus. Geoz (Diskussion) 18:11, 30. Sep. 2015 (CEST)[Beantworten]

Angabe zur Kunststoffart an Produkten (speziell UK)

Kennt jemand (gesetzliche) Vorgaben zur Angabe des Kunststoffs bei Produkten? Speziell kam mir die Frage auf beim Betrachen von diesen Handwärmern und auch Tierzubehör. Ich kenne die Recycling-Codes, würde aber gerne wissen, ob diese auch in anderen europäischen Ländern (verpflichtend?) genutzt werden (speziell UK). --Degurina 12:52, 30. Sep. 2015 (CEST)[Beantworten]

Siehe hier und hier. --Rôtkæppchen₆₈ 18:15, 30. Sep. 2015 (CEST)[Beantworten]

Freistellungen zu Bewerbungen

Hallo, kennt jemand Urteile die diese These die die IHK Kassel hier auf die Webseite schreiben unterschreiben? https://www.ihk-kassel.de/auw_ausbildung_ausbildunga_z_freistellung_bewerbungsgespraech Gruß --GroßerHund (Diskussion) 15:18, 30. Sep. 2015 (CEST)[Beantworten]

Weitere Seiten siehe: https://www.das.de/de/rechtsportal/arbeitsrecht/arbeitssuche/freistellung-arbeitssuche.aspx http://www.arbeitsrecht.org/arbeitnehmer/bewerbung-einstellung/wann-ihr-arbeitgeber-sie-fuer-die-stellensuche-freistellen-muss/

Welche These genau? --88.130.95.57 15:24, 30. Sep. 2015 (CEST)[Beantworten]

Genau dieser Teil: Das bedeutet, dass Auszubildende auch keinen Urlaub nehmen müssen, sondern die Bewerbungstage zusätzlich zu ihrem Jahresurlaub "frei" sind. --GroßerHund (Diskussion) 15:34, 30. Sep. 2015 (CEST)[Beantworten]

Da das BGB 629 (http://dejure.org/gesetze/BGB/629.html) hier nur schwammig formuliert ist. --GroßerHund (Diskussion) 15:34, 30. Sep. 2015 (CEST)[Beantworten]
Für Bewerbungsgespräche - nicht zum Schreiben von Bewerbungen - bekommt der Arbeitnehmer unter bestimmten Voraussetzungen frei. Von einer Anrechnung dieser freien Zeit auf eventuell-bestehende Urlaubstage sagt § 629 BGB nichts und ich wüsste auch nicht, woraus die sich ergeben sollte: Der Anspruch auf Freistellung fällt ja auch nicht auf einmal weg, nur weil der Arbeitnehmer keine Urlaubstage mehr hat.
Andererseits ist die Aussage nicht ganz richtig, wenn dort von "Tagen" die Rede ist. Man bekommt "angemessen" lange frei. Das heißt so lange, wie für den jeweiligen Bewerbungstermin inklusive Hin- und Rückfahrt erforderlich ist. Wenn für Hin- und Rückweg praktisch keine Zeit gebraucht wird und das Bewerbungsgespräch sagen wir mal 2 Stunden dauert (was meiner Erfahrung nach schon arg lang wäre), dann steht dem Arbeitnehmer dafür definitiv kein ganzer 8-Stunden-Tag als Freizeit zu. --88.130.95.57 16:52, 30. Sep. 2015 (CEST)[Beantworten]
In meinem alten Handbuch (Meyer, gibt es nicht mehr aktuell) steht, dass es bis 2 Tage sein können, weil Anfahrtszeiten mit berücksichtigt werden müssen.--2003:68:ED16:C100:6987:8DB3:F930:D4E9 18:25, 30. Sep. 2015 (CEST)[Beantworten]
Dieser Extremfall widerspricht meinen Ausführungen ja nicht. In der Regel sind es halt deutlich weniger, weil Anfahrtszeiten, die nicht anfallen, nicht berücksichtigt werden. --88.130.95.57 18:42, 30. Sep. 2015 (CEST)[Beantworten]

Hilfe bei Notensatz gesucht

Hallo! Ich habe handschriftliche gemeinfreie Noten und den zugehörigen Liedtext, siehe [32]. Könnte jemand das für mich setzen? Oder hat jemand einen Kontakt dafür? --Genealogist (Diskussion) 17:54, 30. Sep. 2015 (CEST)[Beantworten]

Was für ein Schloss ist das?

Moin, kann mir jemand helfen bei dieser Datei?: http://www.gwexter.org/bilder/was%20f%C3%BCr%20ein%20schloss%20ist%20das.jpg. Tausend Dank. --Gwexter (Diskussion) 18:42, 30. Sep. 2015 (CEST)[Beantworten]